Purple Book NBCOT Prep

Ace your homework & exams now with Quizwiz!

A child avoids playground equipment that requires her feet to be off the ground. What does this behavior MOST likely indicate? a. difficulty modulating proprioception b. somatodyspraxia c. gravitational insecurity d. bilateral integration/sequencing deficits.

C. Gravitational insecurity. gravitational insecurity is described as fear response to movement. Tactile defensiveness (answer A) is a term used to describe discomfort with various textures with unexpected touch. Somatodyspraxia interferes with development of bodyscheme and awareness. Bilateral integration relates to ability to coordinate, sequence, and execute motor actions quickly and efficiently.

An OT is preparing to evaluate a toddler who has UE orthopedic concerns. How will the OT MOST likely obtain the majority of the initial assessment data? a. measurement tools that assess visual-motor skills. b. dynamometer and pinch meter readings. c. Observation of a child during activities in the child-care center. d. Functional independence measures.

C. observation of a child during activities in the child-care center. Through observation of the child, the OT can collect information about the child's motor performance skills and participation in activities that require UE/hand skill. Naturalistic observation is a method of ecologic assessment which is a "primary mechanism for obtaining data relevant to the child's performance context." The other answer choices are all appropriate choices after the child is old enough for these assessments.

an OT has been asked to design a social skills group for individuals functioning at a project group level. What is most appropriate activity to include? a. a game of social bingo b. an exercise group c. planning a birthday party d. attending a church service

a. a game of social bingo in a project group the focus is on common short-term tasks that require some interaction, cooperation, and competition. mutual interaction outside the task is not expected. An exercise group or church service does not require interaction and is more appropriate for parallel activity. C requires a higher level of interaction than the individuals in this group are ready for.

an OT is preparing a presentation for a business with a high incidence of workers diagnosed with cumulative trauma disorders. What are the most significant risk factors associated with cummulative trauma disorders? a. repetition, high force, and awkward joint postures b. progressive resistive exercise, joint mobilization, and weight bearing c. inflammation, swelling, and pain d. fatigue, muscle cramps, and parasthesias

a. repetition, high force, and awkward joint postures These are frequently associated with cumulative trauma disorders. B are interventions that OTs may use with a variety of conditions. C and D are all considered to be potential symptoms of cumulative trauma disorders.

An OT is making a home visit to an elderly individual who lives alone and has severe hand weakness resulting from arthritis. When planning to address safety in the home, the MOST important area to consider is the individuals ability to: a. work locks and latches on doors and windows b. use built up utensils while eating c. demo energy conservation techniques d. manipulate fasteners on clothing

a. work locks and latches on doors and windows This is a safety concern because the individuals may not be able to operate the door to let individuals in or prevent break-ins. The other answers are not safety issues.

A child with Autism engages in limited and repetitive acts during OT eval. How would the OT MOST likely document the child's behavior? a. the child rocks and bangs his head against the wall throughout the eval b. the child frequently cries, without an obvious reason that creates the tearfulness c. the child does not verbally respond to the questions initiated by the OT d. the child does not make eye contact with the OT throughout the eval

a. the child rocks and bangs his head against the wall throughout the eval This answer is most representative of restrictive-repetitive action. The other answers are most representative of behaviors related to socialization deficits frequently seen in children with autism.

The office doorway of an individual using a wheelchair has a clear opening of 28 inches. According to the ADA guidelines, which of the following recommendations would be the MOST appropriate to facilitate clear passage of the wheelchair through the doorway? a. the doorway width needs to be expanded to have a minimum clearance of 32 inches b. the client needs to obtain a wheelchair narrower than 28 inches c. the doorway width needs to be expanded to have a minimum clear opening of 45 inches d. the doorway width is satisfactory and needs no modification

a. the doorway width needs to be expanded to have a minimum clearance of 32 inches. The doorway rather than the wheelchair should be adapted.

A school-based OT works with a classroom teacher to develop strategies that promote good handwriting habits and readiness for written seat work. What is the most likely recommended strategy for postural and arm preparation (strength and stability)? a. provide phys ed instructor with UB strengthening activities for use in class b. begin writing periods with students completing chair push ups and elastic band exercises c. include paper and pencil tracing games just prior to written seat work periods d. have students trace letters in shaving cream smeared across the tabletop

b. begin writing periods with students completing chair push ups and elastic band exercises. Postural and arm preparation activities are important to a comprehensive writing program. A is not an optimal strategy, as students would only do this when in gym class. Answers C and D promote visual motor integration which does not influence hand and muscle strength and motor control.

an OT in the school system is developing transition activities for a group of 16-18 yr old students with DD. Which activities are best to address goals related to transition? a. role play ordering food in the classroom b. go out for lunch at a fast-food restaurant c. order a takeout lunch by the phone d. select lunch items from a picture menu in the classroom.

b. go out for lunch at a fast food restaurant This is correct because a key principle in intervention for effective transition includes using natural environments and cues. The other answers do not provide as many opportunities for the student to practice the community member role in activities that actually take place in the community.

an OT is working with a 5th grade child with spina bifida who will need to self-catheterize twice a day. What will this child need in order to perform the self-catheterization? a. assistance with self-awareness, set up , and sequencing b. good hand strength, ROM, and perceptual awareness c. a powered bidet d. reducer rings

b. good hand strength, ROM, and perceptual awareness A is typically indicated for a child with ID whereas C and D are toileting adaptations commonly used to assist with cleansing and positioning.

When working on cooking skills, an individual with a history of TBI exhibits moderate UE incoordination. Which of the following recommendations would be most beneficial for this individual? a. built up utensil handles b. heavy utensils, pots, and pans c. use of a high stool to work at counter height d. placement of commonly used items on shelves just above and below the counter

b. heavy utensils, pots, and pans using heavy kitchen items increases stability for individuals with incoordination. Answer A is useful for individuals with limited grasp, C benefits those who fatigue easily, and D is for individuals with limited reach.

A treatment plan for a child with a visual discrimination problem would most likely include which adaptation of visual materials? a. low contrast and defined borders b. high contrast and defined borders c. high contrast and shaded borders d. low contrast and shaded borders

b. high contrast and defined borders High contrast of the stimuli in relation to the background and defining important areas of the stimuli attracts the eye to provide clear input.

An OT witnesses a seizure in a child with hydrocephalus. What is the MOST relevant information to document? a. childs positioning during the seizure b. objective signs and duration of the seizure c. responsiveness during the seizure d. facial expression during and after the seizure.

b. objective signs and duration of the seizure this will allow the staff to monitor the efficacy of anti-convulsive meds during periods of gradual withdrawal. The other answers are important but not as significant.

An OT is educating a caregiver regarding manual wheelchair mobility. Which of the following is the best way to teach the caregiver to propel a wheelchair down a steep ramp? a. angle the wheelchair backward and guide it down the ramp backwards b. tip the wheelchair backward and guide it down the ramp forward c. allow the patient to propel the wheelchair independently d. obtain the assistance of a second individual

b. tip the wheelchair backward and guide it down the ramp forward. Tip it back until approximately 30 degrees. A would be difficult for the individual to see where she is going. C can only be performed by very strong individuals, D could be awkward and dangerous.

An OT documents that an individual exhibits elbow flexion strength grade of 1. According to the MMT system of letters and numbers, the word that would be the equivalent to grade 1 would be: a. absent b. trace c. good d. normal

b. trace a score of one is defined as contraction that can be observed or felt but not seen as motion. 4 is good, 5 is normal, 0 is absent.

an OT consults with the parents of a child with anxiety disorder and challenging behavior that interferes with homework completion. They consider modifications to the childs bedroom and homework routine. What recommendation would best help the child with self-organization skills? a. providing open storage space for clothing and school supplies for easy access b. painting walls and furniture bright colors to create a cheerful environment and hanging a picture schedule c. adding cabinets with labeled compartments to store items out of sight and creating a consistent nightly routine d. giving the child a choice about when he wants to complete homework

c. adding cabinets with labeled compartments to store items out of sight and creating a consistent nightly routine. A consistent routine will help alleviate the child anxiety about what is going to happen next. Placing unused items out of sight reduces visual distraction. Answers A and B add visual distraction. Answer D is contrary to consistency and predictability.

The OT is making recommendations to a community living site for a 13 y/o child with moderate intellectual disabilities. Which statement MOST accurately describes the functional ability of this child? a. the child requires nursing care for basic survival skills. b. the child can usually handle routine daily functions in a supervised home. c. the child requires supervision to accomplish productive, nonroutine work d. the child is able to learn academic skills at the third to seventh grade level

b. the child can usually handle routine daily functions in a supervised home. Children with a moderate ID have an IQ range of approximately 40-55. They can usually handle daily routine and unskilled or semiskilled work. This child would most likely by able to complete ADLs and live in a group home setting. Answer A describes a child with profound ID. C describes a child with severe ID, and D describes a child with mild ID.

An elderly individual who ambulates with a walker in the home states he does not like sponge baths and would prefer to resume taking showers but is afraid of falling. Which should the therapist do FIRST? a. suggest bathtub bathing instead of showering b. .encourage the client to purchase a shower chair c. demonstrate how using a shower chair improves safety d. explain that therapy will boost his confidence when showering

c. demonstrate how using a shower chair improves safety Educating the client is the first response the OT practitioner should make. Getting into the bathtub is even more dangerous. Explaining that the therapy will increase the client's confidence is subjective and does not address the fear of falling.

The OT is treating a child with an above the elbow amputation who is experiencing hypersensitivity of the residual limb. The therapist would MOST likely perform which intervention in the preprosthetic phase of treatment? a. engage the child in play activities that strengthen bilateral UEs b. include activities to increase the ROM in the shoulder on the affected side c. encourage play activities that incorporate tapping, application of textures, and weight bearing to the residual limb. d. practice dressing activities that include donning and doffing the UE prosthesis.

c. encourage play activities that incorporate tapping, application of textures, and weight bearing to the residual limb. Desensitization is a key preprosthetic intervention. Answers A and B will not affect hypersensitivity and D is incorrect because the child does not yet have access to the prosthetic.

An OT has been hired as a program manager to develop a community based program for individuals with severe and persistent mental illness. What is the FIRST step in the processes that the OT must complete? a. program planning b. program implementation c. needs assessment d. program evaluation

c. needs assessment in order to determine the need for services, a needs assessment must be conducted. It explores the needs of the targeted population from the perspective of those who work with the individuals as well as the individuals themselves. Program planning involves establishing goals and objectives based on the results of the needs assessment. Then the implementation occurs and the program is evaluated.

An OT is performing a functional ROM assessment on a n individual with arthritis. How should the OT eval IR? a. ask the individual to touch the back of his neck b. use a goni to measure IR in a supine position c. observe the individual touching the small of his back d. interview the individual regarding areas of pain and stiffness

c. observe the individual touching the small of his back touching the small of the back requires shoulder abduction and internal rotation. answer a requires external rotation. agoniometer would be used when formal joint measurement is required. an interview will provide useful information concerning pain, stiffness, and limitations, but it is not a reliable method for assessing ROM.

based on evaluation data, the IEP team has developed goals for a new student with autism and is now developing a service plan. which represents the most likely next step? a. team leader identifies the classrooms in the school district that are available for students with autism b. IEP team leader describes the services that school typically provides to students with similar needs. c. family and other team members discuss the service options to identify those that match established priorities and support students achievement d. IEP team approved services that are available in the school and reverse the family to community resources for those that they cannot provide

c. family and other team members discuss the service options to identify those that match the established priorities and support the students achievement. Answers A and B are incorrect as they do not address the students individual needs and do not include the parents in decision making. D is incorrect as it does not include parent participation.

An OT is demoing bathing techniques for a 2 y/o child with seizure disorder and hypertonic muscle tone. Which suggestion is MOST appropriate for the OT to recommend to the childs parents? a. place the child on his back in the tub filled with only several inches of water b. bathe the child quickly to avoid an increase in tone c. handle the child slowly and gently in warm water d. stand and lean over the tub to support and wash the child

c. handle the child slowly and gently in warm water. This will allow a child with hypertonicity to be most relax and easier to handle. A is incorrect because AE is frequently needed to provide a child with a sense of security during bath time and bathing without supports can be dangerous. B could increase hypertonicity. D uses poor body mechanics.

Which of the following methods is best for evaluating a hood grasp? a. direct the individual to hold a sewing needle while it is being threaded b. observe the individual lift a tall glass half filled with water c. have the individual hold a heavy handbag by the handles d. hand the individual a key to place in a lock

c. have the individual hold a heavy handbag by the handle a hook grass is strongly based on the use of digits 2 to 5. It is the only prehension pattern that does not include the thumb to supply opposition. this grass occurs when an individual is holding a shopping bag, pale, or briefcase. answer A would be held with a two-point pinch. answer B would be a cylindrical grass. And answer D would be a lateral pinch

An OT is providing instruction to caregivers in a long term care facility about how to assist a resident whose severe attention span deficits impair the ability to participate in self feeding. The therapist MOST likely recommends which method? a. demo the feeding process b. provide verbal feedback c. provide HOH assist d. use backward chaining

c. provide HOH assist These are most commonly used within the context of functional task engagement where the OT will physically move the body part for the client. Repetition is then promoted in the patterns of functional movement and frequently follow a developmental sequence. This method provides Max A while allowing the resident to still feel involved and connected to the task. Given the severity of the deficits, the other answers would be insufficient to sustain active participation.

an OT is fabricating an orthotic device for an individual who has carpal tunnel syndrome. Which of the following orthotic devices fabrication techniques best describes the position of the trim lines to allow for adequate digit motion? a. distal to the MCP crease b. proximal to the DIP joint c. proximal to the MCP crease d. distal to the ulnar fifth MCP crease

c. proximal to the MCP crease trim lines of an orthosis that extend proximal to the MCP crease allow for adequate MCP flexion and extension. All other answers restrict MCP movement

an individual who uses a wheelchair is being discharged from a rehab facility to home. In determining accessibility of the interior home environment, what areas should be the greatest concern to the OT? a. location of telephones and appliances b. arrangement of furniture in the bedrooms c. steps, width of doorways, and threshold heights d. presence of clutter in the environment

c. steps, width of doorways, and threshold heights this should be the first area of eval to determine if the individual can even enter or exit the home. The other answers would be addressed after.

A preteen with a hx of TBI is relearning how to prepare simple foods but has been having difficulties with sequencing. the OT has provided her with a chart of steps to follow and she has just learned to prepare her favorite sandwich without losing her place in the process. She continues to require occasional verbal cues to look at the chart and ensure safety. At this point, how would her level of independence be documented? a. independent b. independent with set up c. supervision d. min assist

c. supervision At this level, the child performs the task on his own but cannot be safely left alone or may need verbal cuing. At independent level, the child performs the complete task including set up without need for any form of assistance. Answer D signifies that the child performs 50-75% of the task.

An artist recently diagnosed with MS is interested in pursuing a leisure activity that will promote physical fitness. Because the individuals symptoms are limited to mild UE numbness and slight weakness in the dominant hand at this point, the BEST activity to recommend is: a. volleyball b. painting with the dominant hand c. swimming in a cool water pool d. jogging on a treadmill or track.

c. swimming in a cool water pool swimming is an excellent activity for promoting physical fitness. The cool water will prevent overheating that is contraindicated for MS. A and D will likely result in overheating. B would appeal to an artist, but would do very little to promote physical fitness.

Upon discharge, an individual will be performing sliding board transfers with assistance from family members. When ordering a wheelchair, which features will be MOST important to include? a. one arm drive device and low backrest b. reclining backrest and elevating footrests c. swing away footrests and removeable arm rests d. elevating footrests and removable arm rests

c. swing away footrests and removeable armrests This must occur before performing the slideboard transfer. Answers A and B include nothing that would facilitate this type of transfer. One arm drive is useful for individuals with the use of only one UE. D is incorrect because although removable armrests would make the transfer easier, elevating footrests would not.

an individual with low endurance complains of becoming too fatigued during sexual activity to enjoy it. The best strategy for the OT to recommend is for the individual to: a. time sex for the end of the day b. take the top, prone position c. take the bottom, supine position d. experiment with a variety of positions

c. take the bottom, supine position this requires the lease amount of energy and should be the primary recommendation. In addition the OT may encourage experimentation. Having sex at times when there is most energy is beneficial, but the individual would likely be fatigued at the end of the day.

An OT is working with an individual with ALS who developed a sacral decubiti and has recently become too weak to turn himself in bed. What should the OT plan to do NEXT in regard to client/caregiver instruction? a. begin a strengthening program b. suggest that the client and caregiver begin a wheelchair education program c. teach the caregiver how to position the client safely d. provide an environmental control unit to the client

c. teach the caregiver how to position the client safely The progressive nature of the disease necessitates that rehabilitation in ALS be compensatory. The OT helps the caregiver team to optimize safety by assessing positioning, performing safe transfers, and maintaining good skin integrity. Answer A and B most likely would have occurred prior and would not address the issue of decubiti.

an OT in a psych setting is documenting a clients responses to an activity. Which of the following should the therapist write in the chart in order to relay the objective portion of the note? a. the client did not want to finish her stenciling activity b. the client was hostile to another client in the activity group c. the client independently selected one of six craft designs presented d. the client demonstrated an appropriate level of frustration tolerance.

c. the client independently selected one of six craft designs presented This contains objective material validated by the therapist. A and B are interpretations of behavior and D reflects the practitioners judgement.

An individual is about to be discharged from outpatient OT after rehab for a hand injury. The individual has not been able to work for 3 months and is unable to perform all of the job requirements as a truck driver. Which of the following should the OT recommend at discharge? a. HEP b. home health OT c. work hardening d. no further services

c. work hardening work hardening programs are designed to include or simulate job-related tasks that gradually progress the individual to obtain the skills that meet the actual demands of a job. Continuing to perform a HEP and discontinuing the OT services would probably not enable the individual to return to the workforce after a 3 month absence. Home health is only appropriate for individuals who are unable to leave their homes to attend outpatient OT.

The OT staff in an outpatient facility are developing goals for a new, multidisciplinary work-hardening program. Which of the following goals is MOST appropriate for this program? a. ADL retraining to increase the ability to perform household skills independently b. progressive resistive exercise to increase endurance for self-care skills c. work simulation and conditioning to increase strength and endurance in work-related skills. d. vocational retraining to increase to ability to reenter the job market

c. work simulation and conditioning to increase strength and endurance in work related skills Work simulation is a goal of work hardening. In addition to increasing productivity and feasibility through work-simulated activities. Answers A and B are not typical goals. Answer D is incorrect because it is vital that a work hardening program be viewed as an adjunct to vocational retraining, not as a vocational training program in itself.

A 4 y/o with ADHD exhibits visual inattention and perceptual deficits that interfere with completing classroom worksheets. What activity would the OT MOST likely recommend for this child to train visual attention? a. playing a game in which images are matched by memory b. assembling a 200 piece puzzle c. finding waldo against a complex visual background d. blowing cotton balls into a target

d. blowing cotton balls into a target. This is a short term activity with immediate reward. The other answers require sustained visual vigilance and delayed gratification. They do not represent developmental expectations for a preschool aged child.

the OT is observing dressing skills in an individual with COPD. While putting a shirt, the individual becomes SOB, stops to rest before finishing with the shirt, and frequently elevates his shoulders while dressing prior to attempting to put on pants. This behavior most likely indicates he is experiencing issues in which areas? a. postural control b. muscle tone c. strength d. breathlessness

d. breathlessness A key feature of COPD is dyspnea. A would be correct if the client had been unable to maintain balance. B would have been evident if the client had demod spasticity. C would have been evident by inability to push arms through the holes.

an individual with joint changes that moderately limit finger flexion would be MOST comfortable using utensils with: a. regular handles b. weighted handles c. a univeral cuff atachment d. built up handles

d. built up handles this allows a comfortable grasp that regular utensils do not provide. Weighted utensils would cause faster fatigue.

an individual reports that back pain during sexual activity is so severe that it prevents any enjoyment. The best strategy to recommend is to: a. use a side-lying position b. time sexual activity for periods of high energy c. avoid discussing pain with the sexual partner because it might be a "turn off" d. encourage the client to identify alternative methods with the partner for meeting sexual needs that do not cause pain

d. encourage the client to identify methods with the partner for meeting sexual needs that do not cause pain communication is particularly important for this situation. The couple can discuss alternate positions and methods such as masturbation, fantasy, etc. C is inappropriate. B is an appropriate strategy for people with low endurance.

The OT investigator is attempting to gain insight into family beliefs related to children who experience blindness. The OT would like to explore attitudes, values, communication preferences, and overall expectations of the parents in relation to the children who experience blindness. Which of the following would be the most likely naturalistic design that would capture the desired info? a. participatory action research b. grounded theory c. phenomenology d. ethnography

d. ethnography. this is a primary research approach in anthropology concerned with the description and interpretation of cultural patterns of groups. A is an approach that directly involves study participants. B is naturalistic inquiry used to generate theory and comparison. C is uncovering the meaning of how humans experience phenomena by how they live through it.

the goal of a work program for homeless youth is to develop job skills that will improve housing status. Which must occur first? a. help clients feel safe and supported and explore the meaning of the worker role b. develop work skills, habits, and appropriate interpersonal and work behaviors c. emphasize quality and identify realistic work interests d. eval participants performance strengths and weaknesses

d. evaluate the participants performance strengths and weaknesses the first step in any intervention is almost always evaluation. Then the OT would build trust, followed by assistance with developing the clients skills. Finally, work quantity and quality would be addressed.

the OT is working with an individual with alzheimers. Eval revealed profound memory loss, severe intellectual deterioration, incontinence, and nearly complete loss of basic ADL performance. The family is concerned about the agitation the individual demos when getting undressed in the evening. What is the most appropriate intervention for the OT to implement? a. modifying the bathroom to ensure safety with toilet and tub transfers b. training the caregiver in the use of memory strategies such as labeling. c. working with the caregivers to help them utilize routines that will facilitate caregiving d. instructing the caregiver to use visual and verbal cues to enable the individual to be as independent as possible

c. working with the caregivers to help them utilize routines that will facilitate caregiving the individual clearly has late stage alzheimers and thus cannot learn new tasks. A and B should be done during the early and mid stages of the disease. D is no longer an appropriate or safe goal.

following the inclusion of OT in the students IEP, what is the priority for the school based OT? a. develop IEP goals related to the childs fine motor performance to handle classroom tools and learning materials. b. meet with classroom teacher to review methods for collecting progress-monitoring data to track students performance. b. readminister test used in initial eval to update scores for the educational record d. facilitate childs performance in student roles expected in the school setting.

d. facilitate childs performance in student roles expected in the school setting. answer A is incorrect as OTs work together with other team members to generate the students IEP goals. Furthermore, the goals should reflect functional outcomes related to the childs participation in the curriculum, rather than increase isolated performance skill areas. C is incorrect as many tests are not appropriate to measure change.

When transferring an individual from one seat to another, OTs can best protect themselves from injury by: a. stepping back from the individual b. keeping the back in a flexed position c. keeping the knees bent d. maintaining a narrow base of support

c. keeping the knees bent all of the other answers could potentially contribute to injury.

An OT is planning intervention for a student who has difficulty manipulating her pencil to erase errors on her papers. What activities should be included to improve her in-hand manipulation? a. finger to palm translation b. palm to finger translation c. simple rotation d. complex rotation

d. complex rotation This skill involves the rotation of an object 180 to 360 once or repetitively.

Which of the following actions should the OT practitioner instruct a patient to perform FIRST when initiating a safe wheelchair transfer? a. have the patient scoot forward to the front of the seat b. position foot plates in the up position c. swing away the leg rests d. lock the breaks

d. lock the brakes breaks should be locked first. The other answers involve movements that could cause loss of balance or wheelchair movement unless the brakes are locked.

A child with under reactive sensory processing has been referred to OT. Based on sensory integration FOR, intervention activities should have which faciliatory characteristics? a. arrhythmic and unexpected b. arrhythmic and slow c. sustained and slow d. unexpected and rhythmic

a. arrhythmic and unexpected For children with low arousal in the vestibular system, the therapist would use fast, irregular rotational movement. Although arrhythmic input is excitatory, slow sensory input is inhibitory, so B is incorrect. Sensory integration treatment is complex and highly individualized and must be monitored carefully to observe the effects of sensory input on individuals.

A child with autism spectrum disorder demos inadequate playground skills. She plays alone, generally repeating the same play each day. Which approach will the OT MOST likely select to promote her skills? a. OT joins group on playground, facilitating childs play and including other children b. OT works with child on the playground when other students are not using the playground c. recommend the parent enroll in the child in ballet lessons after school d. ask the phys ed teacher to include her in small groups during gross motor activity.

A. OT joins group on playground facilitating the Childs play and including other children Using the natural setting enables the OT to observe the child's typical behavior as well as supports and challenges that characterize the recess period. Answer B does not represent characteristics in a typical recess period. Answer C is not appropriate as a primary approach to address the need. D is incorrect as there are no individualize supports included to adapt the context, instructions, or supports in class.

a 7 y/o with limited pincer grasp wants to zip his own pants in school because he gets embarrassed when he has to ask his teacher for assistance after using the bathroom. Which should the OT recommend the child try FIRST? a. large key ring b. oversized fasteners c. colored zippers d. hook and loop fasteners

A. large key ring a large key ring attached to the zipper pull is an adaptation to enable the child with limited grasp to pull the zipper. Oversized fasteners would not address the child's need to become independent in zipping unless all of the child's zippers were removed and replaced with fasteners. Colored zippers would be most helpful for a child with visual discrimination problems. Hook and loop fasteners are something the OT may recommend if the more convenient and less expensive key ring adaption does not assist the child in meeting his goals.

A supermarket employee with OCD takes an hour to stock 24 soup cans on the shelf because once he has placed the cans on the shelf, he removes them and starts over stating "all the labels were not lined up exactly in the same direction." Which of the following methods would be MOST effective to evaluate this individual's work performance? a. on-site observation of performance skills b. formal cognitive assessment c. verbal interview focusing on the requirements of the job. d. task evaluation using a "clean" medium such as a puzzle.

A. on-site observation of performance skills Individuals with OCD often experience difficulties with work. Observation in situational context is likely to be more useful in fully understanding their behavioral success and is the preferred approach for assessing work function of people with psychiatric disabilities. There is nothing to suggest the individual has a cognitive deficit or need for cognitive evaluation. Interview is more useful for assessing an individual's readiness for work, occupational development, and interests. Task evaluation with a clean medium may be indicated for individuals with OCD of the washing type. However, this individual has OCD of the checking type.

A toddler with spina bifida has been referred for assessment. When collecting the initial data during interview with the child's parent, what should the OT focus on PRIMARILY? a. parent's concerns and goals for the child. b. Childs medical management c. equipment needs d. physical layout of the home.

A. parents concerns and goals for the child. Interviews enable the OT and parent to develop rapport and provide opportunity for the parents and children to identify and discuss issues that are important to them. The other answers are important issues as well but can be addressed at a later time.

The school-based OT meets with the classroom teacher to gain information to help plan services for a student with learning disabilities in the middle school general education curriculum. The IEP includes occupational therapy to support the student's participation in the language arts curriculum. which question is most likely to elicit useful information that will also promote collaboration for the students benefit? a. what are you most concerned about with the students' performance in the language arts curriculum? b. does the student perform better on spelling tests when answers are typed rather than written c. are the students parents in favor of moving to computer use for assignments that require a good deal of an expression? d. what is the student's approach when writing an outline that will guide a short research report

A. what are you most concerned about with the students performance in the language arts curriculum. Answer A is an open ended question that doesn't assume any particular difficulties. B and C are close ended questions. D will gather limited info only about short research reports.

An OT working in a long-term care facility needs to eval the long-term memory of a resident. Which of the following methods is BEST for evaluating memory of personally experienced events (declarative memory). a. show the person a series of objects and ask him to recall the objects within 60 seconds b. ask the individual how he spent New Years. c. have the individual state the place, date, and time. d. ask the client to remember to bring a specific item to the next therapy session.

B. ask the individual how he spent New Years Declarative memory is one aspect of long term memory and includes conscious memory of events, knowledge, or facts. It is commonly assessed through verbal interviews and informal testing such as asking a question about a persons recall of personal events. Working memory refers to the temporary storage of information while one is attending to it (answer A). Prospective memory involves the ability to remember intentions or activities that will be required in the future (answer D). Knowing time, date, and place is orientation.

An OT has been working with an individual who is recovering from a TBI. A standard pivot transfer has been successfully demod in the gym. The MOST appropriate way to assess generalization of this new learning would be to have the patient perform which activity? a. identify potential hazards in the patient's bathroom at home that could make transferring unsafe. b. select an appropriate tub bench and nonskid mat for the patient's bathroom at home. c. attempt a standard pivot transfer from wheelchair to bed in the patient's hospital room. d. attempt a sliding board transfer from wheelchair to tub.

C. Attempt a standard pivot transfer from the wheelchair to bed in the patient's hospital room. Giving the individual a functional task, then changing it, and observing the response will tell the therapist how well the patient can transfer learning to new situations. Generalization of skill is seen when the learner is able to spontaneously perform the task in different environments. None of the other answers assess the ability to transfer learning.

During an initial eval, the OT suspects that a child has somatodyspraxia. In what area should the OT focus on the eval? a. ability to print or write b. reading competency c. math calculations d. new motor task planning

D. New motor task planning Somatodyspraxia is described as a deficit in learning new motor skills, planning new motor actions, and generalizing motor plans. Inability to print or write is termed dysgraphia. Dyslexia means dysfunction in reading, and dyscalculia is the term for inability to perform mathematics.

A high school teacher diagnosed with a right CVA is given a paper with letters of the alphabet displayed in a random order across the page and is instructed to cross out every "M". The individual misses half of the "M"s in a random pattern. What type of deficit would cause such a response. A. left visual field cut b. right visual field cut c. functional illiteracy d. decreased attention

D. decreased attention selective attention is the ability to attend to relevant stimuli while inhibiting distractions and irrelevant information. Because the errors were random, it does not reflect a visual field cut. Illiteracy is unlikely because the individual is a high school teacher.

an individual being interviewed by an OT experienced repeated sexual abuse by a parent as a child. The individual states that parents actions were due to the stress of being fired from a job and that everyone finds different ways to manage stress. In the eval report, the OT should identify this as the use of what defense mechanism? a. identification b. projection c. denial d. intellectualization

D. intellectualization by employing this defense mechanism, the patient protects herself from the pain associated with her trauma. A results in the individual adopting the values and feelings of a person who causes anxiety in order to increase her own self worth. B one attributes personally unacceptable impulses on the external world. And C is a refusal to acknowledge the external source of anxiety

Upon completion of the initial interview and chart review, what is the next step to be taken in the OT process? a. analyze the data b. develop a treatment plan c. perform selected assessments d. select appropriate assessment tools or screening procedures

D. select appropriate assessment tools or screening procedures an OT eval begins with the initial interview and chart review. Then specific evals are chosen and performed. After information is gathered, the OT analyzes the data to identify strengths and weaknesses in order to develop a treatment plan. Then specific interventions are chosen.

During a discharge planning group, one individual states that she may as well stay in the hospital because she is such a burden to her family, even though they say they want her home with them. Using a cognitive-behavioral approach, which response should the OT give to promote cognitive restructuring? a. "what might someone who disagrees say?" b. "im sure your family wants you home with them very much" c. "how about writing a poem that expresses your feelings" d. "take a 15 minute break to do some mindfulness meditation"

a. "what might someone who disagrees say?" The goal of this therapy is for the client to identify beliefs and thoughts that interfere with desired occupational performance to learn new beliefs and ways of thinking. Therapists do not tell their clients what to do, rather, they collaborate with their clients in learning how to think and talk to themselves in ways that support them. Answer B is an example of encouragement. C is a projective technique and D is also a technique that can be used for depression but they are not forms of CBT.

The OT is assessing an individual who demos normal ROM when flexing the elbow, but hyperextends by 15 degrees when the elbow is extended. The practitioner will MOST likely record the measurement as: a. -15 to 0 to 140 b. 0 to 140 c. 15 to 140 d. -15 to 120

a. -15 to 0 to 140 Negative ROM may vary from one setting to another.

the parents of a 12 month old with significant motor dysfunction due to CP asked the OT when a means of assisted mobility should be introduced. What is the earliest age the OT should suggest to the parents? a. 18 months b. 2 yrs c. 3 yrs d. 4 yrs

a. 18 months research continues to substantiate that children as young as 18 months can achieve independent skills in powered mobility

a 17 y/o individual who wears a a hip brace is measured for a wheelchair. What are the correct seat dimensions for the OT to recommend? a. 2 inches wider than the widest point across the childs hips with the brace on b. 2 inches wider than the widest point across the childs hips c. 2 inches more than the distance from the back of the bent knee to the buttocks d. the same as the distance from the back of the bent knee to the buttocks

a. 2 inches wider than the widest point across the childs hips with the brace on This allows the child to easily get in and out of the chiar while wearing the brace while preventing pressure on the sides. B would not allow enough space when the child is wearing the brace. C and D are incorrect altogether.

OT and PT are working with an individual with Medicare benefits in outpatient OT following a THA. the OTs initial eval identifies limitations in lower body dressing and moving around the kitchen safely to make meals. Which G code should the OT use when identifying goals? a. G8988- self care b. G8979 mobility c. G8988 self care and G8985 moving, handling and carrying objects d. G8988 self care and G8979 mobility

a. G8988-self care given that the patient is also receiving PT and only one G code can be used at a time, the most appropriate G code is for self care. Mobility would most likely be used by PT.

An OT has been hired to work with clients with mental health conditions in an organization whose philosophy of care is based on the recovery model. Which is the MOST appropriate eval tool for this population? a. Making decisions empowerment scale b. comprehensive occupational therapy eval c. bay area functional performance eval d. worker role interview

a. Making decisions empowerment scale all answers include assessments that can be used with individuals with mental health conditions however, the recovery model specifically addresses the areas of hope, empowerment, and self-determination. This scale is one of the standardized assessments that is designed to measure empowerment. The comprehensive OT eval and the Bay area functional performance eval both measure occupational function performance. The worker interview addresses psychosocial factors that impact work performance.

an Individual with complete C7 tetraplegia demos fair plus (3+) strength in the wrist extensors. which of the following interventions would the OT introduce to most effectively increase strength in the wrist extensors? a. a craft using increasingly heavy tools b. mildly resistive activities that are halted as soon as the individual begins to fatigue c. electric stim of the wrist extensors d. mild resistance during AROM to the wrist

a. a craft using increasingly heavy tools progressive resistive exercise is the most effective method for increasing strength in a muscle with fair plus strength. mild resistance would be used initially, but resistance would be increased as appropriate. Avoiding fatigue is appropriate for individuals with conditions such as MS, ALS, and Guillian Barre. Estim is for weaker muscles.

an OT who is employed by a senior center has been asked to develop groups to address the motor needs of people with parkinsons disease. Which of the following activities would be MOST appropriate to include? a. a game of rhythmic exercises performed to music b. creating time capsules for their grandchildren c. wheelchair races performed in pairs d. taking turns reading out loud from the newspaper

a. a game of rhythmic exercises performed to music. Rhythmic activities facilitate motor performance in individuals with Parkinsons. Races are generally not a good idea for this population because they often have difficulty terminating ambulation. D is an effective intervention for addressing communication, not motor deficits. Answer B is for patients approaching end of life.

an individual admitted for alcohol abuse is preparing for discharge and has made it clear that he does not plan to completely abstain from alcohol. What type of community based self-help program would be most appropriate for this parent? a. a harm reduction program b. alcoholics anonymous c. narcotics anonymous d. a health promotion program

a. a harm reduction program this type of program acknowledges the difficulty of abruptly stopping usage. If terminating is unlikely, small steps are introduced to reduce usage and harm to the individual. B and C are programs that require complete abstinence from substances. D focuses on concepts such as nutrition, exercise, and participation in meaningful activity but does not directly address substance abuse.

an OT in a residential community mental health setting is planning activities for clients within a psychodynamic frame of reference. which of the following types of activities with a practitioner be most likely to use? a. activities designed to explore and express feelings b. activities match to the specific cognitive level of the crew members c. activities that can serve as reinforcers for desired behavior d. activities that can teach the use of problem solving strategies

a. activities designed to explore and express feelings This theory provides framework for expressive techniques.Answer B is most reflective of a cognitive disabilities approach. Answer C would be used with a behavioral FOR. D would be used within a cognitive-behavioral context.

in assessing the dressing skills of a 5 y/o child, the OT observes that the child is able to put on a jacket , zip the zipper, and tie a knot in the draw string but needs verbal cueing to tie a bow. At what level would the OT most likely describe the childs dressing skills? a. age-appropriate b. delayed c. advanced d. limited

a. age-appropriate A typical child at this age can dress unsupervised and can tie knots but generally does not know how to tie a bow independently.

During eval, an individual who had a severe MI 2 weeks ago displays good memory of the information processed before the MI, poor recall of the first week after the MI, but good recall of info from the past week. What would be the best way for the OT to document this deficit? a. disorientation b. impaired long-term memory c. anterograde amnesia d. retrograde amnesia

a. anterograde amnesia Is the ability to recall events after a trauma. Retrograde amnesia is the ability to recall events prior to trauma. Long-term memory is the ability to store information for recall later. Disorientation is a problem in awareness of person, place, and time.

an individual in an OT group has a history of monopolizing the group without giving others an opportunity to participate in the group process. What is the most effective approach for working on the individuals social conduct and group skills during a discussion about planning a community outing? a. appeal to the group and give the individual feedback about how this behavior affects them b. have the individual assign tasks to various group members c. have the individual observe the group while saying nothing d. pair the individual with another group member to investigate expenses for the outing.

a. appeal to the group and give the individual feedback about how this behavior affects them assigning a specific concrete task allows the individual to focus on the task at hand, while sending the message that the individual is a valued member. B feeds into the individuals issues. D will give the other group member little opportunity for involvement. C will only increase the individuals frustration and lead to feelings of isolation.

An OT is evaluating two-point discrimination in an individual who sustained a median nerve injury. What is the BEST way to administer this test? a. apply the stimuli beginning at the little finger and progress toward the thumb. b. start with the thumb area first, then progress toward the little finger. c. present stimuli in an organized pattern to improve reliability during retesting. d. allow the individual unlimited time to respond.

a. apply the stimuli beginning at the little finger and progress toward the thumb. The purpose of these principles is to eliminate non-tactile cues and to ensure that the responses from the patient accurately reflect actual sensation. General guidelines for sensation testing are that an individual's vision should be occluded, stimuli should be randomly applied with intermingled false stimuli (opposite of answer C), a practice trial should be performed before the test begins, and the unaffected side should be tested first (opposite of answer B). With a median nerve injury the ulnar side of the hand is the unaffected side. and thus should be tested first. Also the individual should be given a specific time in which to respond.

Using an evidence based approach, how will an OT use CIMT with a preschool child with hemiplegic CP? a. as an adjunct to intervention to promote childs funciton b. to promote hand dominance c. to restrain the childs dominant hand d. in an attempt to diminish hypersensitivity in the affected limb

a. as an adjunct to intervention to promote the childs function CIMT seems to be a potential adjunct to other interventions for children with CNS dysfunction. B and D are incorrect as studies of CIMT do not suggest these results. C is incorrect as it describes a characteristic of CIMT not an outcome.

An OT is planning intervention for an 11 month old child who was born three months prematurely and spent 2 months in the NICU. When thinking about expectations, how does the therapist need to consider the babys development? a. as an infant who is 8 months old b. as an infant who is 9 months old c. an infant who is 11 months old d. according to norms for premature infants

a. as an infant who is 8 months old three months are subtracted from the childs chronological age to adjust for prematurity.

While developing an OT profile of a child with autism, the OT meets with the childs primary caregiver. What should the OT do as a FIRST step? a. ask the caregiver about the childs typical day and nighttime routines b. explain that norm-referenced testing will be included in the eval c. interview the teacher to determine developmental milestones the child has reached d. observe the childs daily activities within the typical environment

a. ask the caregiver about the child's typical day and nighttime routines. asking the caregiver about the child's daily activities is the first thing in OT should do when compiling an occupational therapy profile. The caregiver can provide valuable information about how his or her child participates in home and community-based activities. answers B&D or something the OT would most likely do after speaking to the child's parent or caregiver. answer c is not considered to be part of the interview with the primary caregiver.

which of the following community activities provides an appropriate level of challenge for a client beginning assertiveness training? a. asking a store sales person for information about an item before buying it b. in a restaurant, requesting that food be sent back to be rewarmed c. returning an item to a department store for cash with the receipt d. questioning a waitress about whether a restaurant bill is accurate

a. asking a store sales person for information about an item before buying it This process is gradual and empowers clients to make their own decisions, request assistance, avoid being taken advantage of, and provide constructive criticism when warranted. The other answers each require the client to make a special request and question or criticize a service which requires more skill and confidence.

An OT is working with an individual who has COPD and has become severely deconditioned. During the OT eval, the patient identified a goal to be able to prepare meals for the family, along with a strong interest in woodworking. How can the OT incorporate this interest into a plan to increase strength and endurance for meal prep? a. assemble presanded pieces of a wooden ship while standing at the worktable for increasingly longer periods of time. b. sand, assemble, and stain parts of a wooden birdhouse, starting with 2-pound wrist weights c. select a small wooden kit activity that can be sanded and painted in one session, alternating between sitting and standing d. sand pieces for a three piece stool using an inclined board, gradually raining the angle of the incline

a. assemble presanded pieces of a wooden ship while standing at the worktable for increasingly longer periods of time. Paint or stain fumes and sanding particles can be detrimental to individuals with respiratory illness. If not for that, B would have been a good option. C would be appropriate for individuals with a need for immediate gratification. D is a good way to increase ROM.

An OT is developing a stress reduction and management program for an individual recently diagnosed with MS. one stressor that the individual has identified is that her teenage children are resistive to helping with chores that were previously her responsibility (eg. cleaning the bathrooms, taking out the trash). Which coping strategy could the OT recommend that would be most effective in dealing with this stressor? a. assertive communication skills b. time management techniques c. deep breathing and muscle relaxation d. laughter

a. assertive communication skills Clarifying expectations, honestly defining needs, and providing tactful and constructive feedback are communication skills that promote understanding. Successful communication with the clients children will most likely help them deal with their fears and concerns and elicit greater cooperation. Other answers are all useful and valid stress reduction techniques but they do not address the issue at hand.

The rehab team in a SNF is preparing to discharge an individual following rehab for a hip replacement. Although the individual lived independently prior to admit, at the time of discharge he is demoing mild confusion. The patient is able to walk safely indoors with a walker but needs meals prepared and housekeeping done for him. Which housing option is MOST appropriate for this individual? a. assisted living b. memory care facility c. long term care facility d. senior housing

a. assisted living. Assisted living facilities are housing complexes with efficiencies or one bedroom apartments in which services are included in the cost of the room, such as light house keeping, activities, transportation, meals, laundry, medication management, and security checks. This is the minimum level of care this individual will require. A senior housing development may offer meals, transportation, and light housekeeping services for a fee but would not be able to assist with medication management. B and C offer less autonomy and higher levels of skilled care which are not necessary at this time.

the OT is working with an individual though medicare Part B. Which of the following activities can the OT delegate to an aide? a. assisting with routine dressing, after training and competency have been demonstrated b. introducing a new piece of AE with a feeding activity c. completing a billable ADL training session with distant supervision d. selecting an appropriate tub bench from a variety of options in a catalog.

a. assisting with routine dressing, after training and competency have been demonstrated Aides must be very closely supervised and can perform client related tasks that the individual knows how to complete and that does not require skilled service. An aide could not determine which AE would be appropriate and could not bill for services.

when measuring elbow ROM with a goniometer, where must the axis of the goniometer be positioned? a. at the lateral epicondyle of the humerus b. at the medial epicondyle of the humerus c. parallel to the longitudinal axis of the humerus on the lateral aspect d. parallel to the longitudinal axis of the radius on the lateral aspect

a. at the lateral epicondyle of the humerus. The stationary arm of the goniometer should be positioned parallel to the longitudinal axis of the humerus on the lateral aspect. The movable arm of the goniometer should be positioned parallel to the longitudinal axis of the radius on the lateral aspect.

following a hip replacement, a patient is limited to TTWB on the operated extremity. While performing a transfer, the OT should instruct the client to do which of the following? a. attempt to bear 90% of their weight on the unaffected leg b. attempt to place 50% of their weight on the affected side c. try to judge how much weight they can tolerate when standing d. prompt the client to put 100% of their weight on the affected leg

a. attempt to bear 90% of the weight on the unaffected leg In TTWB, clients are instructed to imagine that an egg is under the foot. All other answers break the WBing precautions.

While working with an OT on bed mobility, a patient being seen for home health OT following spinal surgery complains of constant fatigue and difficulty getting a good nights sleep. Which suggestion is MOST appropriate for addressing this individuals sleep concerns from the perspective of habits and routines? a. avoid caffeine and get into bed only when ready to sleep b. use a white-noise machine c. make sure the room is dark and quiet d. exercise and take a warm bath prior to bed time.

a. avoid caffeine and get into bed only when ready to sleep The routine of a warm bath before bed can be relaxing however exercise should be done earlier in the day or evening as it is alerting. Answers B and C are environmental adaptations and may not be suitable for all people.

An OT group is planning an outing to a local horse show. Because many of the group participants take antipsychotic (dopamine-blocking) meds to reduce symptoms of schizophrenia, what is the MOST important precaution for the OT to take? a. avoid prolonged sun exposure b. limit access to sharp objects c. plan for frequent bathroom stops d. avoid flashing lights

a. avoid prolonged sun exposure common side effects of dopamine-blocking antipsychotics include sexual side effects, acute dystonia, akathisia, tardive dyskinesia, dry mouth, sedation, and photosensitivity. Lithium, which is a medication commonly used to stabilize mood, may cause polyuria, requiring frequent bathroom stops. Limiting access to sharp objects is a precaution for individuals who are suicidal. Flashing lights are a precaution for individuals with epilepsy.

an OT is working with a 3 y/o child who has spastic diplegia. What mobility device would be most appropraite to use in assisting this child to explore space? a. body length prone scooter b. aeroplane mobility device c. tricycle d. power wheelchair

a. body length prone scooter children with this diagnosis have abnormal tone affecting all 4 extremities, but more abnormal tone in the LEs. Therefore this child may use his UEs to propel the scooter while his LEs are positioned on the scooter. B is designed for children with good LE function who need support in the upper body. C is for individuals with good LE control. D is designed for individuals with limited UE and LE.

The OT is working with a client who sustained a polytrauma, secondary blast injury. Which of the following might the OT expect to see. a. brain injury and injuries as a result of fragments impacting the body b. trauma similar to what one might see in a car accident c. pulmonary and gastrointestinal issues d. multiple fractures of UEs and LEs

a. brain injury and injuries as a result of fragments impacting the body A secondary blast injury is the result of energized fragments flying through the air causing penetration. Answers B and D are most representative of a tertiary blast injury when an individual is thrown from the blast into a solid object. Answer C tends to be result of a primary blast injury due to exposure to the over pressurization wave or the complex pressure generated by the blast itself.

Impulse control difficulties limit a 12 y/o boy with conduct disorder in the classroom. Which is the MOST effectively written functional goal for his IEP? a. by end of marking period, student will participate in class activities for 1 hour without disruptive outbursts for 4 of 5 days b. by end of the marking period, student will attend to an activity for 30 min, demoing improved impulse control c.student will show a 50% reduction in the frequency of disruptive outbursts by the end of the marking period d. when presented with a new activity, the student will follow the directions without protest, 4 of 5 times, within 6 months

a. by end of marking period, student will participate in class activities for 1 hour without disruptive outbursts for 4 of 5 days This answer relates to the performance to be developed to a childs environment or life tasks, making it more meaningful. The other answers are measurable, but not functional goals because they do not address the context in which the skill is applied. C describes a reduction in negative behavior and focuses the team on measuring that negative behavior rather than the preferred functional performance.

when teaching children with moderate ID to feed, groom, and dress themselves, the OT is most likely to use which approach? a. chaining b. sensory stories c. demonstration d. role modeling

a. chaining Backward and forward chaining are among the behavioral approaches frequently used with children with cognitive limitations.. Answer B is generally used with children whose sensory processing interferes with participation in occupation. Answers C and D may be used together with different approaches, but by themselves do not address the cognitive and perceptual deficits and limited processing performance skills that are characteristics of this population.

While completing tasks on a standardized test, the child suddenly becomes uncooperative and complains that the test is too hard. What is the MOST appropriate response? a. change the pace of the test activities according to the test manual specification b. terminate the session and score the remaining items at zero points c. follow administration instructions without deviation d. adapt the remaining test items to ensure the childs success

a. change pace of test activities according to the test manual specification Most standardized tests have some flexibility about the order and arrangement of items. Although a test may have to end before completing all items, scoring incomplete items as zero does not accurately portray the childs performance. C and D may make the results invalid unless they are specifically addressed in the testing manual.

An OT measures an individual's elbow PROM three times and gets three different measurements, varying by up to 10 degrees. The BEST action for the therapist to take is: a. check the alignment of the goniometer b. use a larger goniometer c. use a smaller goniometer d. attempt to force the individual's arm further into flexion

a. check the alignment of the goniometer If the goniometer is not aligned correctly, the measurements will demo a discrepancy. Changing the size of the goniometer can make the discrepancy bigger. D would be painful to the individual.

When instructing the parents of a toddler in the use and care of a hand orthosis, on which instruction should the OT place the MOST emphasis? a. checking for irritation and pressure problems b. avoiding excessive heat exposure c. cleansing the orthosis regularly d. adhering strictly to the wearing schedule

a. checking for irritation and pressure Because a toddler cannot always effectively articulate discomfort, skin irritation may go unnoticed. Therefore younger children are more at risk for developing skin and pressure problems. Although the other answers are important factors, answer A is most important.

an OT needs to assess the performance skills in a 10 y/o child with autism. Which observation will provide the MOST info? a. a childs use of sandpaper, glue, and paint while constructing a birdhouse b. occurrence of self-stim behaviors during classroom circle c. childs ability to place a backpack in a designated cubby when entering the classroom d. teachers appraisal of the childs capacity in role of the learner in the classroom

a. childs use of sandpaper, glue, and paint while constructing a birdhouse Assessing whether the child can use tools in accordance with their intended purpose while constructing a bird house provides opportunity to observe goal directed, discrete purposeful actions that can be discerned. The other answers are most representative of performance patterns.

The OT is meeting with the family of an elderly individual who has just been diagnosed with early stage dementia. The client has had 3 car accidents within the past 3 months and the family does not feel that continued driving would be safe. They report difficulties with repeatedly getting lost, parallel parking, and driving more slowly that is reasonably safe. Which of the following areas would be MOST important to assess when determining this individuals ability to drive safely? a. cognition b. problem solving c. interpersonal skills d. orientation

a. cognition the individual appears to be functioning no higher than ACL 4.8. older individuals at this level tend to drive too slowly and reaction time is affected. therefore cognition is the most important part to assess. The therapist would not want to limit cognitive assessment to the area of problem solving. answer c may arise as cognition declines. answer D is not the focus of the situation.

an individual sustained a RUE soft tissue injury and is currently experiencing swelling of the hand and joint stiffness. Which of the following should be implemented first in the acute phase of edema management? a. cold modalities, elevation, and active assist or prom b. electrical stim and dynamic use of orthosis c. resistive exercises, weight bearing, and lifting d. joint mobs, serial casting, and dynamic use of orthosis.

a. cold modalities, elevation, and active assist or prom these are all remedial techniques of the biomechanical theory according to the phase of healing. D and B are all considered treatment techniques for joint stiffness. C would potentially increase joint stiffness and pain

In establishing long-term goals for an individual with a complete T4 paraplegia in a rehab setting, the OT would MOST likely predict that the patient will attain what level of independence in bathing, dressing, and transfers? a. complete independence with self-care and modified independence with transfers. b. independence with self-care and moderate assistance with transfers. c. minimal assistance with self-care and moderate assistance with transfers. d. dependence with both self-care and transfers

a. complete independence with self-care and modified independence with transfers An individual at this level will have sufficient trunk balance, UE strength, and coordination. Individuals with high cervical injuries are likely to be dependent in all aspects. Individuals with low cervical and high thoracic injuries require assistance with transfers and some self-care tasks.

which type of play is the OT most likely encouraging during a play period with blocks in the preschool? a. construction play b. symbolic play c. social play d. recreational play

a. construction play Symbolic play is associated with development of language and concepts. Creative play is characterized by refinement of skills in activities that allow construction, social relationships, and dramatic play. Recreational play is leisure experiences such as sports or arts and crafts.

during the eval, the OT observes than an individual is able to place dentures in his mouth, but has difficulty applying denture cream to the appropriate place on the dentures and attempts to place the cap on the tube backward of on the wrong end of the tube. How would the OT most likely interpret this observation? a. constructional apraxia b. ideomotor apraxia c. constructional impairment d. unilateral neglect

a. constructional apraxia this is a specific deficit in spatial-organization performance.

A lead OT researcher is selecting a sample for a study on the effects of a healthy diet and its influence on health and wellness. For the design, the researcher has relied on volunteers pulled from their own OT practice office as a way to recruit clients into the study. This form of sampling is most commonly referred to as what? a. convenience b. quota c. snowball d. purposive

a. convenience is also referred to as accidental sampling, volunteer sampling, or opportunistic sampling.

a patient diagnosed with parkinsons is being seen by an OT to develop a routine for performing self-care activities. How should the OT instruct the patient about the performance of self-care activities? a. coordinate with consistent timing of meds b. complete before meds are taken c. only attempt with the assistance of others d. complete at intervals throughout the day until completed

a. coordinate with consistent timing of meds. The most frequently used medical management for PD is a dopamine agonist, levodopa. A decrease in tremors and rigidity occurs during the on period after the administration of this medication. Timing the medication and periods of on-off are important in planning the clients activities.

when evaling an individual with dysphagia for MOTOR problems associated with swallowing, the OT should look for: a. coughing or choking b. disorientation or confusion c. pain while swallowing d. decreased smell and taste

a. coughing or choking these are motor problems that are commonly noted in patients with dysphagia. B are related to cognitive problems and C and D are sensory issues.

when developing a self-care program for a 3 y/o child with significant visual impairments, what would the OT most likely do? a. create a reliable route to the bathroom, encouraging the child to familiarize herself with the smells and sounds of the bathroom b. have the child practice various obstacle courses created in the family's living room to improve body image/awareness. c. practice fine motor skills with a light up activity peg board d. connect the family to other families with children who have visual impairments

a. create a reliable route to the bathroom, encouraging the child to familiarize herself with the smells and sounds of the bathroom a primary goal for OTs working with visual impairments is to support participation in the contexts in which they function on a daily basis. This establishes compensatory techniques for the visual deficit. B would be apropriate for the development of movement in space and body image or awareness, but does not directly support participation in self-care activities. C would be more for developing Fine motor coordination.

A 1 y/o boy with delayed motor development recently mastered independent sitting. what is the next significant motor performance his family should expect to see him development? a. creeping b. rolling from back to stomach c. cruising d. moving from supine to sitting

a. creeping creeping typically emerges after 7 months. Most 10-12 month old infants crawl rapidly across the room. B is typically accomplished by 6 months. C occurs just before the child walks.

An OT is treating a restaurant worker with pain resulting from a cumulative trauma disorder. The OT suggested using elastic taping to decrease pain in the UE while working. In addition to decreasing pain, this type of taping can also assist with: a. decreasing inflammation and edema b. potentially increasing muscle fatigue c. limiting ROM and strength d. desensitizing the UE

a. decreasing inflammation and edema. Taping does not increase muscle fatigue, rather it normalizes muscle tone by reducing overstretching of muscles. It does not decrease ROM, but potentially improves ROM by relieving pain. Desensitization is not a primary function of elastic taping.

An OT is evaluating an individual who has undergone a total hip replacement to determine awareness and adherence to hip precautions prior to discharge. What can the OT conclude when the individual is observed leaning forward and stopping at 90 degrees of hip flexion to use the long-handled shoe horn while donning shoes? a. demonstrates independence with precautions b. requires verbal cueing to observe precautions c. needs a longer assistive device d. demonstrates cognitive deficits

a. demonstrated independence with precautions precautions following total hip replacement include no hip flexion past 90 degrees, no IR, and no adduction past midline. Individuals with poor short-term memory, impulsivity, or poor judgement may require verbal cuing to remember these precautions. A long shoe horn may be necessary for an individual who cannot put on shoes safely with a shorter shoe horn.

an OT is planning a program for men with co-occuring disorders who are currently residing in a supported housing environment. The needs assessment should focus on issues common to individuals for which of the following conditions? a. depression and substance abuse b. mental retardation and sensory processing c. diabetes and hypertension d. post traumatic stress disorder and amputation

a. depression and substance abuse. Co-occuring disorders (CODS) are when a person has one or more substance related disorder paired with one or more mental disorders. The other answers do not include any substance abuse conditions and do not therefore meet the criteria

An OT is designing a series of group sessions for adolescents with eating disorders. What is a long-term goal for the group? a. develop healthy eating behaviors and meal prep skills b.. improve school performance c. develop independence in meal planning and awareness of portion size d. promote communication skills and assertiveness

a. develop healthy eating behaviors and meal preparation skills. The overarching goals of healthy eating and meal prep are essential to the health and safety of individuals. Menu planning and awareness of portion size are all aspects of healthy eating and meal prep. Although academic performance can be affected, it is of secondary importance. Communication and assertiveness training is also helpful, but again is secondary.

An OT observes that a 10 month old child is able to sit alone by propping himself forward on his arms, but consistently losses his balance when reaching for a toy. What does this behavior MOST likely indicate? a. developmental delay b. protective extension c. typical development d. advanced development

a. developmental delay typically developing children should be able to sit on supported for several minutes by 6 to 7 months. protective extension is a postural reflex to protect against falling. sitting on supported earlier than 6 months may indicate advanced development.

An individual who is considered modified independent for functional mobility consistently leaves her cane in another room. When asked where the cane is, the client replies, "oh, that cane-it's just so ugly". Which of the following actions is MOST important to take? a. discuss issues related to self-concept with the individual b. eval the individual's short term memory c. assess the individual's long-term memory d. devise strategies to address time management

a. discuss issues related to self-concept with the individual This individual has demod competence in using the cane but does not desire to use it. Her response most likely indicates her discomfort with the cane related to looks.

A client with Parkinsons reports recently having an accident when unable to make it to the bathroom in time. When the Home health TO recommends a bedside commode, the idea is immediately rejected by the client. Which of the following actions would the OT take first? a. discuss options and the consequences of each option b. document the reasons for rejection of the adaptation c. practice transferring to a chair at the bedside d. allow time for the individual to think about the options.

a. discuss the options and consequences of each option The combination of the right equipment, behavior changes, and environmental modifications depend on the individuals needs. The following steps can be implemented: identify options and consequences, allow for time for reflection and consideration, practice with demo device, reassess the decision, if acceptable order equipment. If unacceptable, remove it and document why.

Which of the following activities would BEST represent an expected outcome for an individual who completes an energy conservation program? a. getting dressed without becoming fatigued b. lifting heavy cookware without pain c. doing handicrafts without damaging his or her joints d. dusting and vacuuming more quickly.

a. getting dressed without becoming fatigued Prevention of fatigue is the primary purpose of energy conservation. The other answers do not address energy conservation, instead they address pain management, joint protection, and speed.

the OT is introducing a new member to a group at a day program for individuals with dementia. Using an errorless learning approach, which one of the following is the BEST way to begin teaching the new group member the names of other group members? a. each group member holds up a sign with his or her name on it, and the new member reads the names of each b. group members create an association between the names and characteristics c. each group member introduces hi or herself and the new group member is then asked to identify one or two names he or she remembers d. each group member states his or her name while tossing a balloon to another group member

a. each group member holds up a sign with his or her name on it, and the new member reads the names of each In errorless learning, the person learns something by saying or doing it rather than being told or shown by someone. This technique minimizes likelihood of making a mistake. Individuals with further stages of dementia are often incapable of new learning so the other answers would provide much room for error.

In the middle of a wheelchair to bed transfer, a morbidly obese patient begins to slip from the grasp of an average size OT. The BEST action for the OT to take is to: a. ease the patient onto the floor b. reverse the transfer, getting the patient back into the wheelchair c. continue the transfer, getting the patient back into the bed d. call next door for assistancee

a. ease the patient onto the floor. Whenever a situation occurs in which the OT cannot manage a transfer safely, the therapist should consider lowering the patient to the floor so that the patient is not injured. Although calling for assistance is an appropriate action, the higher priority action is easing the patient to the floor to prevent injury.

an OT making a bedside visit finds the patient poorly positioned with an edematous UE caught between the mattress and bed rail. the most appropriate intervention to address the edema in the UE is to: a. elevate the arm on pillows so it rests higher than the heart b. massage the arm gently, stroking toward the fingers c. instruct the patient to avoid active ROM d. suggest the patient avoid PROM.

a. elevate the arm on pillows so it rests higher than the heart When massaging an edematous extremity, stroking should be performed distal to proximal, not the reverse. Because ROM can be beneficial to edema management, answers C and D are incorrect

an individual demos flaccidity in the LUE following a CVA. while performing PROM to the affected arm, the OT notes marked pitting edema on the left hand. What should the OT do next to decrease edema? a. elevate the affected extremity b. fabricate a resting splint for the affected extremity c. provide the mild resistive exercises d. have the individual attempt to squeeze a ball

a. elevate the extremity positioning, use of compression gloves, massage, and PROM are all effective methods for reducing edema. B is an effective positioning tool, but it is not as effective as elevation and compression. C could result in more damage. D is inappropriate because the left arm is flaccid.

An OT is seeing an individual with stage 4 cancer for palliative care. What will the emphasis of intervention be for this individual? a. emphasizing QOL and engagement in meaningful activity. b. improving strength and endurance and managing pain. c. maximizing independence in perineal activities d. dealing with the psychological issues associated with preparing for death.

a. emphasizing QOL and engagement in meaningful activity. The palliative approach focuses on providing clients with relief from the symptoms, pain and stress of serious illness. Hospice care, unlike palliative care, is carried out only in the last weeks or months of life and D would be more related to that setting. Goals related to strength and endurance and maximizing independence are consistent with a rehabilitation approach.

a teacher in the child-care program asked the OT to help plan ways to include a preschool child with Down syndrome in parallel play. Which is the OT MOST likely to suggest? a. encourage 2 or more children to play next to each other with a collection of books b. initiate a game of go fish with children seated in a circle on the floor c. have the child work in a distraction-free environment while coloring a picture d. encourage several of the children to play a soccer game in the centers outdoor space

a. encourage 2 or more children to play next to each other with a collection of books. Parallel play is when a child plays beside others but play remains independent. Answers B and D are most representative of cooperative play. Answer C is a form of solitary play.

during a group session for individuals in an inpatient facility, one of the members repeatedly interrupts others, grabs tools and supplies and states this group is a waste of time. What is the best approach for the OT? a. encourage other group members to express their frustration with the individual by using I feel statements b. tell the individual she may not return to the group until she is able to treat others with dignity and respect c. ignore the behavior during the group and speak to her about it afterwards d. explain to her that this type of behavior is unacceptable.

a. encourage other group members to express their frustration with the individual by using I feel statements the individual dominated the group with disruptive behaviors. Ignoring this disregards the needs of the other group members. D may not be enough. B may be last resort.

An individual with a history of COPD has limited endurance. The long-term goal for this individual is to prepare three meals a week. The most relevant short term goal for the OT practitioner to focus on is independent use of: a. EC techniques b. work hardening activities c. graded activities to increase strength d. safety skills in the kitchen

a. energy conservation techniques EC techniques should be taught early for individuals with COPD so that they can be implemented in all life activities. Answer B and C would not address the need to perform meal prep in the most efficient manner. D would be more relevant to individuals with sensory or balance loss rather than deficits with endurance.

when evaling an adolescent with an ADHD the OT will be particularly observant for which behaviors? a. excessive talking and inability to remain seated and wait for his turn b. frequent references to oneself as the authority and the smartest in the group c. use of profanities and displays of aggressive behavior d. expressions of anxiety about working on a project and fear of failure

a. excessive talking and inability to remain seated and wait for his turn these behaviors correctly exemplify the excessive fidgeting and restlessness, inattention, and impulsiveness characteristic of ADHD. B consists of behaviors typically associated with grandiosity and self-esteem related to mood disorders. C exhibits symptoms of conduct disorder. D is typical of a child with anxiety disorder.

a 4 y/o with spina bifida has a lesion at the lumbar level resulting in a flaccid bladder. The parents are requesting a bladder training program. What is the OTs most appropriate response? a. explain to the parents that the toilet training is not a feasible option b. recommend waiting until the child is 5 y/o c. develop a toilet training program together with the parents d. assess the childs ability to remove LE garments

a. explain to the parents that toilet training is not a feasible option When the lesion is in the lumbar region or below, the reflex is no longer intact and the bladder is flaccid. Bladder training will be ineffective because the bladder has insufficient tone and requires assistance in emptying. These children are commonly provided with some type of catheterization. D is an appropriate goal but it is not relevant to the scenario.

An OT is evaluating the sensation of an individual who recently sustained a CVA and has adapted the method of response to include nodding and pointing to a card with a picture of the correct answer. This method of response would be MOST appropriate for an individual who has: a. expressive aphasia b. receptive aphasia c. agnosia d. ataxia

a. expressive aphasia This interferes with an individuals verbal or written expression, but not comprehension of verbal or written information. Individuals with receptive aphasia cannot comprehend spoken or written words and they do not understand instruction. Individuals with receptive aphasia may be able to imitate or follow a demonstration, but these techniques do not work for a sensory eval. Answers C and D would be able to understand directions, but be unable to accurately indicate an area because of impaired recognition of the body part or impaired coordination.

the OT is instructing an individual how to perform sit to stand transfers after a THR (posterolateral approach). Prior to sitting down from a standing position, the therapist should instruct the individual to: a. extend the operated leg forward, reach back for the armrests, and slowly sit b. extend the nonoperated leg forward, reach back for the armrests, and slowly sit c. extend the operated leg forward, hold the walker securely, and slowly sit d. extend the nonoperated leg forward, reach back for the armrest with one hand while holding the walker securely with the other, and slowly sit

a. extend the operated leg forward, reach back for the armrests, and slowly sit extending the operated leg forward allows maintenance of surgical precautions. All other answers are contraindicated.

The mother of an 18 month old child with hypertonia reports that dressing is extremely difficult . When teaching the parent how to put on the toddlers shoes and socks, what should the OT suggest to the mother FIRST? a. Flex the childs hips b. flex the childs knees c. flex the childs neck d. flex the childs shoulders

a. flex the childs hips. Flexing the hips breaks up the extensor pattern and when combined with knee flexion reduces the tone in the LEs facilitating dressing. Flexing more distal joints first is ineffective in reducing tone and may lead to excessive stress on the joints involved.

The OT has been working with a group of individuals with co-occurring mental illnesses and substance abuse in a clubhouse model for several months. The group is functioning at a mature level. Which is the BEST way for the OT to be involved in this group? a. functioning as a peer in the group b. functioning as a group leader c. functioning as a group advisor d. functioning as a group facilitator

a. functioning as a peer in the group A mature group is heterogenous in composition characterized by members taking those task and social emotional roles. The therapist interacts as an equal group member. The OT would function as a leader in parallel, project, and egocentric cooperative level groups. The OT would function as an advisor or facilitator in cooperative level groups.

a 6-month-old infant being seen for an OT assessment shows a strong preference for the left hand when reaching for a rattle at midline. considering the development of dominance in normal children, what is the OT most likely to conclude? a. further observation and evaluation of right side of dysfunction is indicated b. development of hand dominance is proceeding in a typical manner c. hand dominance will not develop until the child is one year old d. unilaterally proceeds bilaterally and typical development

a. further observation and evaluation of right-sided dysfunction is indicated. infants usually use a bilateral approach at this age. research reflects at 60% of the children were inconsistent in hand use for a simple grass task. It is worrisome and a typical when it infants less than 7 months old consistently use one hand, and these infants should be further evaluated for possible neurological impairment. answer B is incorrect because unilaterally at age 4 months is not typical development. answer c is incorrect because research has shown that consistent hand preference is seen from 2 to 4 years. answer D as in correct because bilaterally precedes unilaterally in the course of infant development.

an individual with guillian Barre acute syndrome demos poor to fair strength in the UEs. Which is the most appropriate approach for the OT to use when planning treatment for the EARLY stages? a. gentle nonresistive activities b. progressive resistive exercise c. fine motor activities d. AROM against moderate resistance.

a. gentle, nonresistive activities the initial phase of treatment for this diagnosis includes PROM and the use of an orthosis or positioning to protect weak muscles and prevent contractures. This should be followed by gentle, nonresistive activities and light ADLs as tolerated. Resistive exercises should be implemented after strength begins to improve. Activities within later treatment sessions should alternate between gross and fine motor and resistive and non resistive types to avoid fatigue.

during group activity in a mental health setting, one client, joe, grabs a tool from another client, sam. The OT employs a modeling approach to encourage appropriate behavior. Which of the following best illustrates a modeling approach? a. get joes attention, then return the tool to sam and ask him whether you may use the tool when he is finished with it. b. remind joe about the way he had asked sam to pass the sugar for his coffee that morning and praise him for it c. ask joe to identify another method he might have used to obtain the tool d. tell joe that you an see how frustrating it is to have to wait his turn for the tool

a. get joes attention, then return the tool to sam and ask him whether you may use the tool when he is finished with it. modeling is behavioral intervention that recognizes that learning occurs through observation and imitation. A is an example of modeling. B is an example of positive reinforcement. C is an example of a problem solving approach. and D is an example of an empathizing approach.

An individual with Alzheimer disease becomes confused with multistep instructions during self-care activities. Which is the MOST effective method that the OT practitioner can recommend to the caregiver for giving directions to the individual at home? a. give simple, step by step instructions and physical guidance b. provide three-step instructions with gestures for demonstration c. offer written instructions for the individual for any tasks that contain more than 3 steps d. have the individual verbally repeat the instructions after the caregiver gives them

a. give simple, step by step instructions and physical guidance. Answer B can be confusing because it provides too much stim. C is unlikely to be retained in the individuals working memory. Also, written instructions could be lost if the individual puts them down. D does not enable a person with this disease to retain information in the memory, and he or she may not repeat the instructions properly.

An OT is educating a client who presents with weakness in the left hand due to a CVA about the possibility of using a dynamic orthosis, such as a SaeboFlex, during feeding and grooming tasks. The SaeboFlex would assist the client to do which of the following? a. grasp and release objects such as a fork or toothbrush b. lift heavy cookware without pain c. reach above (over 90 degrees of shoulder flexion) to place dishes into overhead cabinets d. flex and extend the elbow in order to brush hair.

a. grasp and release objects such as a fork or toothbrush this device assists individuals with hypertonia in the hand to place the hand in an open, functional position. The other answers would not be the primary objective of issues related to weakness of the hand.

an OT provides a leather working activity to an individual with complete C8 tetraplegia to increase current grip strength. Which component of this activity would be MOST effective in promoting this goal? a. holding the hammer b. grasping the stamping tool c. squeezing the sponge to wet the leather d. lacing the leather with a needle

a. holding the hammer this is the only activity listed that requires gripping with the entire hand. B and D requires pinch patterns. C offers less resistance than holding the hammer and therefore would be less effective for strengthening.

The goal for an individual in the later stages of Parkinsons disease is to dress independently. The BEST adaption to compensate for this persons physical deficits would be: a. hook and loop closures on the front-opening clothing b. large buttons on front-opening clothing c. larger clothing slipped on over the head with no fasteners. d. stretchy fabric clothing with tie closures in the back.

a. hook and loop closures on front-opening clothing Parkinsons has 5 stages: 1-resting tremor appears and symptoms are mild and unilateral, stage 2- problems develop with trunk mobility and postural reflexes, symptoms are bilateral. Stage 3- results in mild to moderate functional disability with postural instability. Stage 4- difficulty with manipulation and dexterity increases . Individual with stage 5 are confined to bed. A would require the least amount of dexterity. Answer B might be easier than small buttons but would still require more manipulation than Velcro closures. C would eliminate the need for dexterity but having to raise the arms would be problematic because of the stiffness and rigidity of the arms. Closures on the back would be too difficult for an individual with decreased flexibility.

An OT working with a community mental health agency has a caseload of individuals planning on moving into supportive housing apartments. The role of the OT with these individuals is to work on skills related to budgeting, interpersonal relationships, domestic routine, and leisure/social practices. Which sis the BEST time and place for the OT to work on the skills? a. in the apartments after they have moved in b. in the partial hospitalization program before they move into the apartments c. in the adult day program after they have moved into the apartments d. in the clubhouse model environment after moving into the apartments

a. in the apartments after they have moved in Training is most effective when provided in a real-world context. Answer D provides an option for daytime social participation but would not allow for training in the natural environment.

task-specific training is being used to train an individual with severe cognitive limitations to put on a t-shirt. How will this training be most effective? a. in the individuals home b. in the OT department c. at least twice a week d. when the client is feeling cooperative

a. in the individuals home this is the natural environment which helps carryover and learning. The other answers would all be less effective or ineffective.

the most appropriate goal for an individual who sustained a C7 to C8 SCI is to be able to perform wheelchair to commode transfers at which of the following levels? a. independent with transfer board b. min to mod assist with a transfer board c. dependent in all transfers d. assisted, using a stand pivot transfer

a. independent with transfer board Stand pivot transfers are appropriate for individuals who have a lower level of injury that can put some weight through the LEs. Individuals with injuries C4 or higher require dependent physical assistance but may provide verbal direction.

an individual with COPD has identified an LTG of being able to shop independently for groceries. Which statement is the BEST STG for this individual? a. individual will purchase 10 items at the supermarket with supervision b. individual will cook a one-dish meal with items purchased at the supermarket c. individual will identify food items needed for developing a shopping list d. individual will learn EC techniques that apply to grocery shopping.

a. individual will purchase 10 items at the supermarket with supervision This is the best goal because it relates to the long term goal and is measureable unlike answer D.

an individual with a history of anger self-control issues has met the goals of independently identifying anger-provoking stressors in his home and work life and identifying his typical behaviors in anger-provoking situations. What action should the OT take next? a. initiate training in conflict resolution strategies b. utilize creative media to explore situations that trigger anxiety c. implement use of journaling to identify stressors and responses d. recommend discontinuation of OT services

a. initiate training in conflict resolution strategies Anger management programs typically address perspective taking, problem solving, recognizing emotions, and conflict resolution. Answer B is designed to provide a means for self expression and realization. Discontinuation of services is incorrect, because the client has not mastered his anger management.

an OT working on laundry scales with an adult with cognitive disabilities has determined that the individual is unable to recognize or judge when clothing is dirty.what is the next step that should be taken to maximize the individual's independence in doing laundry? a. check the individual to wear clothes for two days and then launder those items b. assess the individual's ability to recognize dirty clothing c. recommend that the individual take the clothes to the dry cleaner rather than wash them at home d. recommend that the staff do the individuals laundry from now on

a. instruct the individual to wear clothes for 2 days and then launder those items In answer A, the patient relies on a schedule rather than judgment. Answers C and D removes independence from the individual. Answer B would have already been performed.

at a weekly meeting of a responsive social skills group, one individual states he has a job interview the next day and is concerned about how much to reveal about his personal history. What is the next step the OT should take in the problem-solving process? a. involve the group in generating possible solutions for the problem b. select the best solution or combination of solutions c. instruct the group in how to role-play for this clients situation d. identify the pros and cons of each possible solution

a. involve the group in generating possible solutions for the problem responsive social skills traing programs follow a specific sequence. First, participants are asked to report on their progress since the previous session and describe problems they have recently experienced or are anticipated. Then problem solving begins with defining the problem, generate possible solutions, identify pros and cons, and select the best solution and discuss how to carry out the solution in a social interaction. Once the problem solving sequence is complete, the group would move onto the social skills training sequence where role-playing would occur

an extremely withdrawn individual has developed the ability to tolerate interaction with one other group member while making greeting cards using rubber stamps. Which of the following steps should be taken next in order to develop this individuals ability to interacting with others? a. involve the individual in a 3 member task group b. progress the individual from rubber stamping to stenciling c. instruct the individual in how to make stamps from various materials d. encourage the individual to choose stamps and colors

a. involve the individual in a 3 member task group in this case, the activity should be upgraded slightly by increasing the size of the group, challenging the client to use and develop greater personal interaction skills. B adds a level of complexity to the task which can help to develop problem solving skills. The other answers can all be effective adaptations for other goals.

a patient with a fractured radial head has the arm immobilized from above the elbow to below the wrist. The patient has requested exercises that will help maintain strength in the elbow and forearm muscles while awaiting permission to move the arm. In response, what types of exercises should the OT provide instructions to perform? a. isometric b. isotonic c. progressive resistive d. passive

a. isometric during isometric contraction, no joint motion occurs and the muscle length stays the same. The limb is set or held taught as agonist and antagonist muscles stabilize the joint. Isotonic movements shorten muscle length, Progressive resistive exercises are a type of isotonic exercises. With the arm immobilized the individual would be unable to perform these exercises. D results in joint motion but not muscle contraction and cannot be performed by a person who is immobilized.

an individual with a UE fracture has asked how to maintain arm strength until the cast is removed. What activities would best accomplish this goal? a. isometric muscle contractions b. isotonic muscle contractions c. progressive resistance movements d. passive movement

a. isometric muscle contractions isometric muscle contractions involve contracting the muscle without moving the joint. Isotonic shortens the muscle length. Progressive resistive is a type of isotonic contraction. D are performed by an outside force for joint movement with no contraction.

An OT is discharging a 4 y/o child with cerebral palsy from a rehab setting to home. What are the MOST appropriate instructions for the OT to provide to the family for maintaining the correct jaw control while feeding the child from the side? a. jaw opening and closing are controlled with your index and middle fingers; place your thumb on the child's neck. b. jaw opening and closing are controlled with your index and middle fingers; place your thumb on the child's larynx for support c. jaw opening and closing are controlled with the palm of your hand on the child's jaw, cupping it gently. d. jaw opening and closing are controlled with your index and middle fingers; place your thumb on the child's ear for stability.

a. jaw opening and closing are controlled with your index and middle fingers; place your thumb on the child's neck. Thumb placement should be near the fulcrum of jaw movement. If the child is fed from the front, the adult's thumb is placed on the child with the middle finger under the chin to control opening and closing of the jaw. The index finger then rests on the side of the child's face to provide stability.

An OT is advising the parent of a 5 y/o child with athetoid CP about the type of toy they might buy to facilitate their childs construction play. what is the best type of toy to recommend for this child? a. large magnetized shapes and blocks b. blocks that are small and have a firm surface c. soft, non-hardening modeling clay in a variety of colors d. colorful books with heavy paper or laminated pages

a. large magnetized shapes and blocks children with athetosis have fluctuating muscle tone and difficulty grading or timing movements and inadequate isolation of movements that interfere with hand function. Efforts should be made to stabilize toys and equipment. Magnets will provide some stability. Answers B and C are more challenging to manipulate and therefore incorrect answers. Answer D may be enjoyed but has not direct influence on construction ability.

A college student with a history of substance abuse has been admitted to the hospital following an accidental overdose at a party. He states his goal is to return to school as soon as possible so that his GPA does not drop below 3.0. What is the MOST important area for the OT evaluation to focus on. a. leisure skills b. ADLs. c. academic/study skills d. family education

a. leisure skills in this situation, the college student appears to be having more difficulty with healthy use of leisure time than with academic skills. In this case, it is particularly evident that leisure is affected, with the individual's primary leisure activity being substance use. Skills required to accomplish ADLs or IADLs are rarely severely impaired. Family education is an important element, however, it would not be the focus of an evaluation.

A child with autism present as clumsy and has difficulty with moving through space and executing efficient gross motor control. The OT can best support this child through which approach? a. look at pictures of children playing on an obstacle course and talk about what they are doing with this child just prior to beginning the activity b. include ample opportunities for practice and repetition in parts of an obstacle course before trying multiple segments together c. prior to beginning, remind the child about what an obstacle course is and demonstrate various ways of moving through its segments d. limit tactile sensory experiences in the obstacle course segments, preferring primarily vestibular experiences

a. look at pictures of children playing on an obstacle course and talk about what they are doing with this child just prior to beginning the activity Children with autism have difficulty planning new movements and visual cues provide helpful support. B is incorrect because motor skills executed in isolation are less likely to be meaningful for children with autism. C is incorrect because the directions do not help with the ideation to move through the obstacle course. D is incorrect because sensory environments are used in a way that enhance praxis.

An OT practitioner is addressing concerns about sexual activity with a person who has left sided hemiplegia with spasticity. The BEST recommendation for positioning during sexual intercourse for this person would be a. lying on the left side, propped up with pillows b. lying on the right side, propped up with pillows c. laying in supine position d. laying in prone position

a. lying on the left side propped up with pillows According to the AHA, lying on the left side is the correct answer because it allows the unaffected right extremities to remain free and provides weight bearing to the affected side to assist with tone reduction. The pillows behind the individual allow support and pressure relief. Lying on the right side would not reduce tone. Lying in supine or prone would be uncomfortable. Also, an individual lying in prone has less mobility.

An individual has sustained a large, full thickness burn to both UEs and is in the acute phase of treatment. Which of the following BEST represents a short-term rehab goal in the acute stage of recovery? a. maintain and or prevent loss of joint and skin mobility b. issue AE c. provide compression and vascular support garments d. prevent scar hypertrophy through scar management techniques

a. maintain and or prevent loss of joint and skin mobility. During the acute stage, when burn wounds are partial or full thickness in nature, maintenance of joint ROM and skin mobility is the primary goal of intervention. In the acute phase, splinting and positioning are used in combination with exercise and activity. Answer B, C, and D are goals most commonly implemented during the rehab phase.

The OT has determined that progressive muscle relaxation training would benefit an individual who has anxiety and a limited attention span. What is the FIRST step in the training program? a. make a fist, then gradually relax it b. focus on a rhythmic, repetitive word c. walk until an increased heart rate is achieved d. deeply inhale and slowly exhale

a. make a fist and then gradually relax. PMR is one of several methods OTs use for relaxation training. Making a fist is an example of this technique. The remaining answers represent different types of relaxation training.

An OT is evaluating assistive technology needs for an adult with severe motor limitations. The OT will MOST likely: a. make recommendations for ways of operating the technology b. assist with vocational goals c. seek job placement for the client d. solve mechanical or software problems

a. make recommendations for ways of operating the tech Answers B and C are most often the job of a vocational rehab agency. Answer D is usually the job of a rehab engineer.

A client presents with chronic, poststroke UE edema of the right arm and hand. Which of the following would the OT most likely suggest to manage the edema? a. manual edema mobilization b. hot pack applications c. paraffin d. sensory reeducation

a. manual edema mobilization This is a method of edema reduction based on methods to activate the lymphatic system. Answers B and C are often contraindicated in cases in which edema is present because the direct heat increases bloodflow and could increase edema. D would address limitations with sensation, not edema.

What should an OT do to promote playfulness and self-expression in a young child with mild ID? a. model imagination, and use playful facial expressions and voice b. ask child to demonstrate his favorite things to do during playtime c. provide child with toys that are familiar and played with frequently d. provide activities with a means of release such as leather tooling.

a. model imagination, and use playful facial expressions and voice asking the child to demonstrate play or use familiar toys do not help expand the childs current play behavior to become more playful. Answer D is a technique for structure and focus, not playfulness.

an individual in stage 1 of ALS who is currently ambulatory, is independent in ADLs and complains of overall weakness has been refereed for OT in an outpatient setting. Which of the following interventions is most appropriate for this individual? a. moderate exercise in uninvolved muscles and AROM b. progressive resistive exercises of involved muscles and AROM c. use of orthosis and identification of AE needs d. PROM and use of orthosis

a. moderate exercise in uninvolved muscles and AROM early interventions should target the individuals symptoms as they affect occupations. Stage 1 activities should maintain motor function by moderate exercise in unaffected muscles and AROM. B may cause cramping and fatigue. C and D will be required later to prevent contracture.

An OT is preparing the discharge summary for a preadolescent child with limited UE strength and endurance. Which home adaptation is MOST important to recommend? a. mount lever handles on doors and faucets b. remove all throw rugs c. install nonskid pads on steps d. use a tabletop easel for written homework

a. mount lever handles on doors and faucets These require less energy than knob handles and are recommended for weak grasp. For children with reduced UE strength and endurance, using less complex movements results in energy conservation. Answers B and C are environmental adaptations for minimizing slipping and falling for children with incoordination or postural instability. Answer D is contraindicated because it requires more energy.

The lead OT investigator gave a group of patients who were diagnosed with C5 to C6 tetraplegia a test prior to scar management intervention and another test after the intervention. What kind of test design was the investigator MOST likely using? a. pretest-posttest b. survey c. nonexperimental d. ex post facto

a. pretest-posttest this design is significant in describing what occurs after the introduction of the independent variable. The design can answer questions about change following exposure to the independent variable.

after several months of working with an OT, a client with severe and persistent mental illness has obtained a job cleaning empty offices for four hours a day. While taking public transportation to the job site, the client experiences ridicule from school-age children taking the same bus to school in the morning. Using an environmental adaptation approach, what is the best action for the OT to take so that this client can keep this job? a. negotiate work hours that would allow the client to travel at times that children are less likely to be on the same bus b. have the client take a taxi rather than a city bus c. work with the client on coping skills so that the school childrens taunting is more manageable d. volunteer to offer a mental health awareness day for the school district

a. negotiate work hours that would allow the client to travel at times that children are less likely to be on the same bus in this case, the OT would want the most cost-effective and expedient environmental intervention to enable the client to continue working the job. Negotiating a change in work hours could occur quickly and would not cost anything. A taxi would be more expensive so it is not the first choice,. Coping skills are necessary, but it is not an environmental adaptation and neither is a mental health awareness day.

an orthotic for an individual with carpal tunnel syndrome should position the wrist in: a. no more than 20 degrees of extension b. 10 degrees of flexion c. 5 to 10 degrees of extension d. 10 to 15 degrees of extension

a. no more than 20 degrees of extension This is a condition resulting from compression of the median nerve. The orthotic should position the wrist in neutral.

An OT is teaching her colleagues about survey development and various forms of survey design. A survey is considered to be which of the following? a. mom-experimental b. quasi-experimental c. pre-experimental d. time series tool

a. non-experimental these designs primarily rely on statistical manipulation of data rather than mechanical manipulation of data. The other answers are all forms of experimental design.

an OT has been hired to design and implement a lifestyle redesign program for recently retired older adults. With which activity should the OT begin each module? a. occupational self-analysis b. transportation c. safety d. social relationships

a. occupational self-analysis lifestyle redesign programs emphasize the power of occupation by educating participants about the health related consequences of their occupation. Each module begins by guiding small groups though an occupational analysis.

while developing an OT program for an adolescent with significant learning disabilities, the OT most likely includes which strategies to promote the developmental changes associated with adolescent development? a. opportunities to explore new areas of interest, helping to identify strengths and needs. b. encouragement to join clubs in high school to meet new friends c. recommendations that the family pursue OT outside of school to prevent negative peer pressure d. intervention in private area of the school to protect privacy

a. opportunities to explore new areas of interest, helping to identify strengths and needs The OT provides the student with exploration and experimentation that provide teens with success an failure, both of which are required if they are to develop a sense of boundaries in their competence. The other answers remove the teen from his peer group and reduce the opportunities he has to self-advocate and develop positive interactions with his peers.

An OT is preparing to do a parachute activity as part of a sensory integration program. Because several of the patients in the group are taking antipsychotic medications, the OT should be alert for which possible side effect that could occur as a result of this activity? a. OH b. photosensitivity c. excessive thirst d. blurred vision

a. orthostatic hypotension The parachute activity involves significant up and down body movements and therefore warrants the therapists full attention with this population. Photosensitivity is a common side effect of antipsychotic drugs and dry mouth resulting in excessive thirst is a common side effect of antidepressants. Blurred vision is a common side effect of anticonvulsant drugs commonly used to treat bipolar disorder.

an OT practitioner is developing a factory onsite work-hardening/work conditioning program protocol for workers returning to light duties following musculoskeletal work related injuries. Which of the following would be MOST relevant to include in the design of the program? a. pain management techniques b. achieving a balance between work and leisure c. EC techniques d. vocational counseling

a. pain management techniques work hardening programs focus on returning individuals to work in physically appropriate settings as quickly as feasible. Proper body mechanics would be addressed as opposed to EC techniques. Although answer B is important, it is not the emphasis of this type of program. Vocational counseling helps individuals enhance their potential and addresses skills necessary for employment.

An individual with serious and persistent mental illness demos the ability to participate in an inpatient task-oriented group. She Is able to stay within her own space and use her own materials, with minimal verbal exchange. She prefers to focus on the activity rather than on the other group members. Which level of group should the OT provide to enable this individual to participate as fully as possible? a. parallel b. associative c. basic cooperative d. mature

a. parallel Most of the time, individuals functioning at the parallel level of social participation are able to participate without interfering with others. This question wanted the answer that most defined a parallel group.

An OT has been working with an individual with depression who is an inpatient. Now the individual is ready to return to the job held before taking a leave of absence. Which of the following actions should the OT take next? a. perform a job analysis b. request reasonable accommodation c. emphasize activities that promote a sense of self-efficacy d. encourage the individual to participate in a weekly support group

a. perform a job analysis this provides an objective basis for evaluating, training, accommodating, and supervising people with disabilities and identifies essential functions of particular jobs. Based on the results, the OT can work with the individual to maximize performance or request accommodations. Answer C would have been used earlier in the process. Answer D may be an effective way for the individual to obtain support but the job analysis is more relevant to immediate needs.

An individual's family wants to build a ramp to the primary entrance of the home. What is the maximum slope that the OT should recommend to the family? a. 1 inch of ramp for every foot of rise in height b. 1 foot of ramp for every inch of rise in height c. 10 inches of ramp for every 2 inches in height d. 1 foot of ramp for every foot of rise in height

b. 1 foot of ramp for every inch of rise in height According to ADA the maximum slope should be 1:12.

as part of an initial eval in an outpatient setting, the OT is developing an occupational profile on an 18 month old with Erbs palsy. Which approach best represents the OTs use of narrative clinical reasoning skills? a. perform interviews with the parents regarding their daily routines, social life and child rearing practices b. rely on info gathered form the collaboration between the OT and family c. determine the childs performance level through an activity analysis task and occupational performance checklist d. focus energy on the available social and financial resources of the family to target goals for participation in community based activities

a. perform interviews with the parents regarding their daily routines, social life and child rearing practices This enables the therapist to understand the meaning of this experience from the clients perspective and helps the therapist focus on what is important to the family. B is most representative of the interactive facet of clinical reasoning. C is used to define specific client diagnosis-related problems. D is a task most related to pragmatic or the practical side of clinical reasoning

An individual diagnosed with COPD is participating in a pulmonary rehab program. Which of the following will the OT MOST likely recommend? a. perform pursed lip breathing when doing activities b. use a long handled sponge while in the shower c. take hot showers to reduce congestion d. avoid air conditioned rooms during warm months

a. perform pursed lip breathing while doing activities. This technique prevents tightness in the airway by providing resistance to expiration. It has been shown to increase the use of the diaphragm and decrease assessor muscle recruitment. Taking hot showers and avoiding air conditioning are incorrect activities that are contraindicated for individuals with COPD. Using a long handled bath sponge may be helpful but not the most likely tip to be included in a home program.

The OT investigator is exploring the lived experiences of individuals diagnosed with terminal cancer in order to prioritize end of life OT interventions. Which of the following forms of naturalistic or qualitative injury would be most appropriate? a. phenomenological b. ethnographic c. participatory action research d. grounded theory

a. phenomenological this aims to discover meaning of the lived experience. These researchers believe a phenomenon can only be understood by those who lived it. Ethnography is a primary research approach in anthropology concerned with description and interpretation of cultural groups. Answer C is an approach that directly involves study participants. D is a method of naturalistic inquiry used to generate theory.

an OT is working with the family of a 3 y/o child who lacks sitting balance in the bathtub. The familys insurance does not cover DME costs. What should the OT recommend for this child who loves bath time? a. place a foam lined plastic laundry basket in the tub for use with direct adult supervision at all times b. wrap a horseshoe shaped inflatable bath collar around the childs neck while bathing c. utilize a bath hammock in the tub while bathing d. suggest that the child shower in a standing position instead of bathing in the tub.

a. place a foam lined plastic laundry basket in the tub for use with direct supervision at all times. This piece of equipment addresses both cost and positioning concerns. It will also permit the child to continue to submerse herself in the tub. Answer B would be indicated for a child with severe motor limitation who is lying supine in shallow water. Answer C might be considered as an alternative because bath hammocks fully hold the body and enable the parent to wash the child thoroughly, but this option is not as cost effective as the laundry basket. D does not address the needs of bathing in the bathtub and the child who lacks sitting balance will likely lack standing balance.

a child who has had a TBI demos tongue thrust. What should the OT do first, prior to feeding? a. position the childs trunk, head, and neck and shoulders into proper alignment b. recline the childs seat backward and position the neck in slight extension c. place the OTs digits directly under the childs chin, facilitating tongue retraction d. provide upward pressure under the childs lower jaw prior to chewing

a. position the childs trunk, head, neck, and shoulders into proper alignment Proper positioning is the first thing the OT should address prior to feeding. B is contraindicated. Answers C and D are commonly used to facilitate proper feeding techniques, but they should not be initiated until after the child is appropriately positioned.

An individual with LUE flaccidity is observed sitting at a table in his wheelchair at lunchtime with his left arm dangling over the side. The FIRST positioning strategy the OT should introduce to the individual is: a. positioning the UE on the tabletop surface with dycem b. repositioning the arm back on the wheelchair armrest. c. using an arm sling d. attaching a lap tray

a. positioning the UE on the tabletop with dycem The arm should be passively positioned as normally as possible during ADL tasks to provide sensory and proprioceptive feedback. When the client is eating have him/her place the arm on the table. This technique keeps the arm safe and in appropriate position. A lap tray would provide support but is more restrictive than answer A. C would provide support but immobilize the UE in adduction and IR.

An OT is ordering a mobility device for a child with CP. The child has some arm control and hand grasp function and frequently expresses the desire to be independent in mobility, especially within the school setting. What type of device will the OT MOST likely recommend that the family order? a. power wheelchair b. standard wheelchair c. caster cart d. powered scooter

a. power wheelchair This answer would be the most appropriate for the deficits mentioned. Answers C and D are not appropriate for a child with severe CP because adequate UE function is required for use. Answer B would be preferred if the individual had greater use of the UEs.

an individual is receiving OT to promote independence in meal prep and cleanup activities. Which method of structured activity practice would best promote retention of learning and generalization of meal prep skills? a. preparing a variety of foods using different cooking methods and recipes b. cooking one meal from beginning to end in the same kitchen setting several times c. mastering one part of a meal at a time d. performing each step of the food prep process, such as cutting veggies

a. preparing a variety of foods using different cooking methods and recipes generalization is the ability to apply a skill to an altogether new task or environment. The other answers focus on a more systematic or constant practice.

the parent, child, and OT worked collaboratively to develop a STG: "demonstrate increased manipulation shills by opening a 3 inch wide thermos jar independently". the most important tool for determining the childs progress is: a. goniometer b. 3 inch screw top jar c. dynamometer d. developmental fine motor assessment

b. 3 inch top screw jar the goal was written as a functional behavioral objective.e This requires that the item be used to directly measure the functional goal.

Individuals with schizophrenia living in a supported housing apartment have expressed an interest in having more control over what they do on weekends. In response, the OT began offering a leisure skills group; however, attendance has been poor. What actions should the OT take FIRST to improve group attendance? a. provide a desirable snack for those who attend b. ask the program manager to make attendance mandatory for all residents c. break down the activities into smaller steps d. provide positive feedback and avoid criticism.

a. provide a desirable snack for those who attend Avolition, or decreased motivation, is one of the primary negative symptoms of schizophrenia. Individuals with schizophrenia have identified several reasons for this including hallucinations and delusions that interferer with concentration and secondary depression that leads to withdrawal. When individuals with this diagnosis demo avoidance, rewards can serve as motivational strategies. Making attendance mandatory will improve attendance but does not address the underlying issue. Activity breakdown would be beneficial if the group members were having difficulty completing the tasks resulting in a feeling of failure. Providing positive feedback and avoiding criticism are strategies that can be used to promote self efficacy.

The OT has been asked to identify strategies to reduce agitation in residents of a SNF with mild to late stage Alzheimers disease. What are the MOST appropriate environmental modifications? a. provide rocking chairs, music, dimmed lights and reduced clutter b. train caregivers in use of validation and reinforcement c. install monitoring devices such as door alarms, pressure gates, and video monitors. d. institute mealtime positioning and adaptive feeding strategies

a. provide rocking chairs, music, dimmed lights and reduced clutter the environment should be reassuring, reminiscent, and calming, and include sensory stim. Answer B has been found to be helpful, however it is a communication method, not an environmental adaptation. C enhances the safety of the environment but are not directed toward the agitated behavior. D is not indicated with this scenario.

An OT is working with an individual who has a brain injury and is unable to remember what day it is and frequently tries to put on pajamas after the midday meal. what is the MOST appropriate compensatory intervention to facilitate awareness of person, place, and time? a. provide verbal cues, external aides, and opportunity to practice using the aids b. reduce the number of distractions by moving the individual to a quiet room when reviewing info c. present info about the environment in short units with time between each segment d. connect new orientation info to previously learned knowledge

a. provide verbal cues, external aids, and opportunity to practice using the aids Answers B and C would be adaptive strategies to promote attention and information processing. Answer D is a technique to aid retrieval of info and improve memory. It is a remedial strategy, not a compensatory strategy.

while measuring the active range of motion of a patient's MCP joints, It is most important for the OT to provide stabilization: a. proximal to the MCP joints b. distal to the MCP joints c. at the rest d. on top of the MCP joints

a. proximal to MCP joints. stabilization is applied proximal to the MCP joints to isolate the joint movement being measured into eliminating any combined movements. The other answers would allow for combined movements of the joint and would invalidate the individual joint measurements. answer D would block joint movement and make any individual joint measurements and correct.

a 3-year-old child with down syndrome is dependent in all areas of dressing. when using a developmental approach with this child, which skill should be first addressed? a. pulling shoes off feet b. putting on a t-shirt c. removing sweatpants d. putting on a hat

a. pulling shoes off feet children first learn to remove garments. C is still a skill that follows removing shoes and socks.

an OT is selecting foods for a treatment session with a client who has decreased oral motor control and difficulty chewing. In general, which type of foods would be most appropriate to introduce? a. pureed foods such as pudding or applesauce b. mechanical soft textured foods such as ground tuna c. soft foods such as banana or mac n cheese d. thin flavored liquids such as juice

a. pureed foods such as pudding or applesauce to minimize the risk of aspiration, pureed foods are chosen for clients with decreased oral motor control and chewing difficulties. B and C may increase aspiration and would be introduced if pureed foods do not present a problem. D is the most difficult food to handle.

A preteen with ID enjoys independent use of his computer assisted technology, but engages for brief periods then complains of general fatigue. How can the OT best assist the student? a. reassess the physical environment and location of AT b. increase the size of the computer screen to reduce visual exertion c. encourage the student to request assistance when feeling fatigued d. take turns with the student while typing homework assignements

a. reassess the physical environment and location of the AT ergonomic work arrangements allow for relaxed positions and EC. B addresses other relevant factors in computer use, but none that would directly affect fatigue. C and D do not coincide with the students independent use of the computer

An OT received the following info 6-27-15: ADL evaluation and treatment for diagnosis of RCVA: Three times a week for 1 month. Signed, L. Martell, MD. What did the OT just receive? a. referral b. screening c. goal setting d. treatment plan

a. referral A referral is a request for OT services. Screening is the process of observing and collecting info about the individual to determine the need for OT services. Goal setting and treatment planning occur after an evaluation has been performed.

An elderly individual who was hospitalized for a right CVA with LUE flaccidity and decreased sensation is beginning to experience sensory return in the LUE. What intervention strategies should now be included in the treatment plan? a. remedial treatment ,such as rubbing or stroking the involved extremity. b. remedial treatment, such as the use of hot mitts to avoid burns c. compensatory treatment, such as testing bathwater with the uninvolved extremity. d. compensatory treatment, such as using a one-handed cutting board to avoid cutting the insensate hand.

a. remedial treatment, such as rubbing or stroking the involved extremity. When sensation begins to return, it is appropriate to initiate remedial activities for sensory retraining. The other options are interventions to protect the client from further injury but are compensatory approaches.

An OT is consulting in a long term care setting to recommend programs for residents who seem isolated and disengaged from other residents. Which type of group would be BEST for enhancing self-esteem, providing opportunities for socialization, and assisting residents in integrating past experiences with present life? a. reminiscence b. meditation c. grooming d. movement

a. reminiscence The focus of this group is on providing social opportunities for sharing life stories and feelings. The other answers would address some goals, but not as comprehensively as answer A.

A child with ataxic CP exhibits tremors in the UEs. When she feeds herself, tremors cause most of the food to fall off the swivel spoon before it reaches her mouth. Which adaptations should the OT recommend? a. replace the spoon with a blunt ended fork b. built up handle of the swivel spoon c. provide HOH assistance d. bend the spoon handle at a 45 degree angle

a. replace the spoon with a blunt ended fork Stabbing food is more often effective for feeding. The food will not fall of the fork. Answer B is more appropriate for a child with a weak grasp. C and D are more appropriate for a child with limited forearm and wrist motion.

A student is learning to activate a switch for a communication device. Although the switch is mounted on the wheelchair tray, the student continues to have difficulty operating it because of excessive muscle tone. Despite practicing for extended periods of time, the student is not making any progress. What should the OT do NEXT? a. reposition the switch to facilitate easy access and adjust further as needed b. passively stretch the students UE to increase ROM. c use a brightly colored switch to increase visibility d. use systematic behavioral reinforcement through shaping

a. reposition the switch to facilitate easy access and adjust further as needed. Answer B addresses a limitation in ROM, C addresses visual impairment, and D pertains to behavioral and cognitive issues. None of these were concerns mentioned in the scenario.

An OT and OTA share and coordinate therapy for a caseload. Which of the following tasks would be MOST appropriate for the OTA to perform? a. reviewing charts for pertinent info b. completing the nonstandardized portions of the eval c. interpreting the results of the nonstandardized portions of the eval d. independently designing a treatment plan for the individual

a. reviewing charts for pertinent info an OTA contributes to the screening, eval, and reeval process by implementing delegated assessments and providing reports. Answers B and C are not within the scope of practice for an OTA.

an OT is working with an adult who is 10 years post brain injury, has severe attention and memory deficits, and has poor self awareness. The clients goal is to develop basic job skills for working in a packaging plant. What is the best approach for teaching how to package three pieces into a plastic bag? a. rote repetition of packaging task substeps with gradually fading cues b. preparatory activities to develop sequencing skills c. activities designed to improve memory and attention d. use of instructional cards to use as a reminder of how to perform the packaging task

a. rote repetition of packaging task substeps with gradually fading cues The use of task-specific training or rote repetition within natural contexts is recommended for individuals with chronic or severe cognitive deficits. The other answers are all remedial activities. For an individual s/p 10 years, the task-specific approach is more appropriate.

An OT is performing a home management eval of an ambulatory individual with CP who is cognitively intact but exhibits ataxic gait patterns. The PRIMARY focus of the eval should be: a. safety and stability b. the individuals ability to reach and bend c. whether the individual has adequate strength to perform homemaking tasks d. fatigue and endurance levels.

a. safety and stability Incoordination, ataxia, and athetoid movements can be symptoms of CP and other CNS diseases. These deficits can cause step length to be uneven and create a tendency to fall. Strength was not identified as an area of concern for this individual. Answer B is of primary concern for those with limited ROM. and D is a primary concern for individuals with MS, guillian barre, ALS, and other neurologic conditions.

The OT is working with an individual who experiences panic attacks when near dogs, and it has begun to interfere with socialization. Using a CBT approach, what is the BEST way for the OT to help this individual handle this challenge? a. select a picture of a calm, gentle dog, then practice mentally shifting from the image of a scary dog to the image of the gentle dog b. help to identify strategies that will allow avoidance of dogs in everyday life c. provide instruction of how to use mindfulness when in the presence of dogs d. start by touching a therapy dog just for one second, then gradually transition to petting the dog for increasingly longer periods of time.

a. select a picture of a calm, gentle dog, then practice mentally shifting from the image of a scary dog to the image of the gentle dog. CBT is effective with this disorder. In this method, the individual confronts the fear in a safe situation and challenges negative thinking. B would be an example of an environmental adaptation. C is a widely used intervention for anxiety. D attempts to diminish anxiety.

A child with ASD craves tactile stim, rubbing objects on his arms and legs; however, he avoids being touched by others. This behavior MOST likely indicates a sensory integration problem related to what area? a. sensory modulation b. sensory registration c. sensory seeking d. over responsiveness

a. sensory modulation Studies indicate that many children frequently demo behaviors of under responsiveness and over responsiveness, often within the same sensory system. Answer B is incorrect as sensory registration refers to the CNS knowing when to pay attention to stimulus and when to ignore it. Answer D only describes part of this childs behavior.

An individual with schizophrenia has worked as a janitor for about a year. The case manager requested an OT consult when the individual was transferred to an area with carpets and began having difficulty with job performance, specifically vacuuming, reporting that the vacuum triggered auditory hallucinations. Which is the MOST appropriate area for the OT to assess? a. sensory processing b. cognitive level c. social participation d. balance

a. sensory processing individuals with schizophrenia often find sensory stimuli unpleasant and engage in behaviors to avoid the sensation. A cognitive eval would not be warranted because only the environment has changed for this individual. Nor is there anything to suggest that the individuals social skills are an area of concern. Vacuuming may challenge balance but if other activities such as mopping are not problematic then there is nothing to suggest this issue.

A flight attendant with low back injury is participating in a work-hardening program. The individual can successfully simulate distributing magazines to all passengers in a plane using proper body mechanics. To upgrade the program gradually, what should the OT NEXT request that the individual simulate? a. serving from the beverage cart b. issuing blankets and pillows c. distributing magazines to half the passengers d. putting luggage in the overhead compartments

a. serving from a beverage cart the demands of jobs vary considerably. When distributing magazines, the flight attendant uses reaching, bending, pushing, and pulling movements. Serving from the beverage cart also involves these movements and more resistive reaching that required to distribute magazines. Blankets and pillows are lightweight like magazines and would not be considered an upgrade. Putting luggage into overhead compartments would be the final step in the work hardening process because it is the heaviest and riskiest task. C would be downgrading the activity.

What are the PRIMARY functions of an OT leading a therapeutic group in the beginning stages of group development? a. set the atmosphere, model appropriate behavior, and develop the rules. b. facilitate expression of feelings and conflict resolution. c. provide balance between support and confrontation, and encourage new skills. d. reinforce change and how it can apply to everyday life.

a. set the atmosphere, model appropriate behavior, and develop the rules. The other answers reflect the transition, working, and final stages of group development respectively.

When providing caregiver training to the spouse of an individual diagnosed with early stage dementia, the OT will MOST likely need to instruct the caregiver in strategies to compensate for what deficits? a. short-term memory b. fine and gross motor skills c. social skills d. dressing skills

a. short term memory the earliest symptom of Alzheimer's is short term memory loss. Sensorimotor abilities used in dressing tend to follow. Motor skills become involved in the later stages. Superficial social abilities are often preserved until the last cognitive changes.

An OT places a 6 month old infant in the supine position and arranges attractive toys overhead to provide an opportunity to work against gravity. This position will MOST likely encourage antigravity movement of: a. shoulder flexion and protraction b. shoulder extension and retraction c. head control d. trunk control

a. shoulder flexion and protraction Answer B is incorrect because shoulder extension and retraction would not be encouraged during supine activities when the toys are placed over head. C and D are incorrect because the head and trunk are not working against gravity when the infant lays in supine.

a 6th grader with poor UE control due to fluctuating muscle tone needs an adapted computer for communication. What is the best adaptation to allow for computer use? a. single pressure switch, firmly mounted within easy reach b. lightweight keyboard placed at midline c. low-resistance mouse and pad d. mercury switch headband set to respond to minimal movement

a. single pressure switch, firmly mounted within easy reach A child with fluctuating muscle tone lacks stability and demos extraneous movement. The other answers involve devices that would respond to slight touch and would not be effective for a person with extraneous movement or grading motor action.

A young child with DD has just learned to sit independently on the floor. To facilitate the NEXT step toward refining postural reactions in sitting, what should the OT encourage the child to do? a. sit straddling a bolster with both feet on the floor b. maintain sitting balance on a scooter while being pulled c. bounce on a ball with a handle without falling off d. maintain floor sitting position with the therapist providing pelvic support

a. sit straddling a bolster with both feet on the floor Once independent postural reactions are developed on a stable surface, the child can further refine sitting skills by learning to maintain posture when placed on unstable surfaces. At this point, external stabilization is no longer necessary. At first, the child should be in control of the movement and have feet on the floor for maximal stability, then the child can progress to more difficult tasks such as answers B and C.

an OT practitioner is starting an inpatient groupfor individuals with schizophrenia whose psychosis has been controlled through medications, but who continue to display negative symptoms, including difficulty sustaining attention, limited ability to express feelings and ideas, depressed affect, and low interest and energy levels. which type of activities would the OT be most likely to plan for the members of the group at this beginning stage? a. skill building activities with concrete goals, such as time management training b. self management activities, such as relaxation training c. expressive/projective activities such as journal writing d. verbal activities such as discussing movies or current events

a. skill building activities with concrete goals, such as time management training individuals with negative symptoms often need highly structured activities with concreate goals. The other answers are treatment formats that could be used with clients who have schizophrenia but may present too high a level of challenge at this stage.

A 60 y/o auto mechanic with diabetes and impaired sensation in the residual lower limb has been referred to OT for prosthetic training following an above the knee amputation. The FIRST thing the OT should address is: a. skin inspection b. grooming techniques c. retirement planning d. returning to work

a. skin inspection visual inspection of an insensate area is essential for preventing pressure sores. Nail trimming is an important issue to address in individuals with diabetes, but it is a secondary concern. Retirement planning and returning to work are issues that may be addressed when discussing discharge plans.

during lunch, the OT observes one individual who grabs the ketchup away from her neighbor, chews with her mouth open, and does not make eye contact with those around her while others seem to be trying to avoid her. These behaviors exhibited by this individual most likely indicate a deficit in which of the following? a. social interaction skills b. physicality in communication c. self-control d. coping skills

a. social interaction skills C demos poor self control. By developing interpersonal sills and coping skills, the individual may demo improved social conduct. But social interaction skills are most relevant.

when preping an eval summary for a child with juvenile RA, what is the most important info to share with the childs school teacher? a. summary of the childs cognitive and visual perceptual skills b. AE and ADL needs in the classroom c. info on the childs ROM status d. recommended HEP

b. AE and ADL needs in the classroom A and C are of lesser importance and may be unimportant for the teacher to support this students learning. D is most useful to the childs family, not the teacher.

an individual is recovering from digit amputations of the right hand and complains of pain when any sensation is felt on the affected hand. When implementing a program of desensitization training for the patient, the MOST appropriate sequence for grading the sensory stimuli that will be applied to the patients hand is from: a. soft to hard to rough b. tap to rub to touch c. light to medium to heavy D. rough to hard to soft

a. soft to hard to rough. Answer B is incorrect because the force of the stimuli begins at level of touch, progresses to rub, and moves to tapping. This sequence is rather out of order. Answer C does not specify the texture of force or stimuli. Answer D is backwards.

An individual with chronic mental illness is moving into a group home and would like to learn to take a bus to a friends home three stops away. In the first training session, the OT accompanies the client to the bus stop and onto the bus, demos how to pay, points out when they have arrived at their destination, and walks with the client from the bust stop to the friends home. Using the scaffolding approach, which one of the following should the OT do differently next session? a. stand back while the client pays for the ride b. encourage the client to walk from the point of disembarkation to his friends home independently. c. teach the client to recognize the first, last, and intervening bus stops on the map inside the bus. d. instruct the friend to watch from the window until the client is able to get to the house independently.

a. stand back while the client pays for the ride scaffolding is a method of grading an activity by providing assistance to the client at times that he or she might struggle or be unable to complete this step. Eventually, some of the support and assistance can be taken away. Using this approach, the OT would allow for more independence during the next session. B is more complex and would be more pertinent to the backward chaining approach. C is a skill development approach. D may enhance safety but is not a scaffolding approach.

Which is the BEST position to promote isolated head control in a child with very limited postural control and significant UE and LE weakness while participating in a fine motor activity? a. standing in a standing frame with knee and hip support b. quadruped with chest supported in a sling c. prone over a wedge with toys in front d. side-lying with suspended toys nearby

a. standing frame with knee and hip support This is the best choice as demands on head righting are less in an upright position than a horizontal position. B and C are incorrect because although the chest is supported by a sling, the childs shoulders, arms, and hips must be able to control the position. Sidelying only allows for lateral head rights, a component of neck stability that contributes to head control.

Which activity is MOST appropriate to recommend for vocational skills training of high school students with severe learning disabilities who are developing skills for community-based employment. a. stocking the shelves at a local grocery store b. packaging items in a local sheltered workshop c. cleaning the school cafeteria after lunch d. role playing interviews with a mentor

a. stocking the shelves at a local grocery store. A is correct as it is a real life functional activity in an actual work setting. B and C do not provide real life settings for job training. D is not considered vocational training.

an individual who is several days post MI experiences nausea during a bathing eval. Which of the following is the first action the OT should take? a. stop the activity b. document symptoms c. instruct the individual to sit and then continue the activity d. ask the individual whether he can continue the activity

a. stop the activity activity should stop immediately when symptoms occur during a treatment of an acute cardiac patient. Symptoms include confusion, SOB, profuse sweating, light-headedness, clammy skin, and nausea should be reported to the physician. After the session is over, the symptoms should be documented. C and D is dangerous and could result in further cardiac damage .

during a kitchen activity, an individual with decreased shoulder ROM demos difficulty retrieving items from the higher shelves. Which of the following recommendations will BEST facilitate home management for this individual? a. store the most frequently used items on shelves just above or below the counter b. use the largest joint available to move or lift items c. perform shoulder ROM exercises 10 times each, twice per day d. continue reaching for items on high shelves because it will help improve ROM

a. store the most frequently used items on shelves just above or below the counter whenever an individual experiences difficulty reaching items due to limited ROM, convenient placement of the commonly used items will facilitate home management. Answer B is an important principle of joint protection, and although it may be an appropriate suggestion for an individual with arthritis, it would not address the specific problem of reaching. D and C may eventually improve function, but neither answer provides a home management solution.

Which leisure activity BEST suits a 6th grade student with juvenile RA to help maintain ROM? a. swimming b. basketball c. soccer d. aerobics

a. swimming Swimming provides active movement through wide ranges of motion with minimal impact on the joints. The other answers involve additional stress on the joints and would be contraindicated.

A child with limited sitting balance is unable to put on and remove LE clothing. Which BEST addresses this functional issue? a. teach the child to dress in a side lying position b. add loops to the waistbands of pants and skirts c. use hook and loop fasteners in place of zippers d. teach the child to dress in a standing position

a. teach the child to dress in a side-lying position this position lessens gravity and aids independence for LE dressing. B is incorrect because the primary purpose of putting loops on waist bands is to help a child with limited grasp strength. C is incorrect because using hook and loop fasteners in place of zippers is also an adaptation to help children with limited ability to grasp and pull. D is considered more difficult than dressing in sitting.

An OT is using leather stamping as part of a group activity for individuals with neurological deficits and needs to increase the degree of problem solving demand within the group. What is the BEST approach for encouraging problem solving in a craft media group? a. teach them to break the activity down into smaller, more manageable steps b. begin with gross motor activities and progress to fine motor activities c. structure the number and kinds of choices available d. gradually increase the time used in the activity by 15 minutes

a. teach them to break the activity down into smaller, more manageable steps. training in problem solving strategies involves teaching a person to break down complex activities into more manageable steps. Fine and gross motor activities can heighten awareness of self and develop coordination. Answer D helps development of attention span. C is a method for developing decision making skills.

When planning services for a preschool child using MOHO to guide the intervention, what is the OT most likely interested in learning about the child? a. teachers report of how the child spends classroom free play time and choices made during outdoor recess b. how the childs posture and seating positions impact visual motor control and use of hands during learning activities c. instructions, assistance, and cues child is given in preparation for transition between activities, and response following attempts to change activities d. childs preferences among activities that require movement, such as playground activity, move and sing activities, and sand and water play

a. teachers report of how the child spends classroom free play time and choices made during outdoor recess. MOHO emphasizes the occupational adaption through motivation, routines, patterns, and skill performance. Answer B includes areas related to postural control as a foundation for skilled distal motor performance, suggesting a biomechanical approach. D considers activities rich in sensory input, suggesting a SI approach. C suggest a behavioral practice model, as it suggest re-enforcers.

To avoid overstimulation when handling a stable, 12 week premature infant in the NICU, the OT FIRST: a. teaches parents to use firm, steady touch without rocking to help the infant during feeding and diapering b. helps the infant establish a calm state by playing a musical mobile c. encourages auditory stim through singing music around the infant d. establishes a bond through visual orientation to the therapists face

a. teaches parents to use firm, steady touch without rocking to help the infant during feeding and diapering. The tactile system develops early and is responsive in the young NICU patient. Early stimulations for these younger preterm infants may be safest as it replicates normal parenting activities. young infants tolerate unimodal stimulation best. Answer B and C can be overstimulating because it provides multiple sensory stimuli. D is optimal if the bond is encouraged between baby and parent.

An individual with ALS is no longer able to ambulate for kitchen or home management activities. Which of the following interventions BEST addresses the goals of independence in meal preparation for this individual? a. teaching wheelchair level techniques b. training in the use of adapted equipment c. standing at the counter for gradually increasing amounts of time d. beginning with cold meals and progressing to hot meals.

a. teaching wheelchair level techniques as the disease progresses, individuals with ALS lose the strength required for ambulation. In phase II of ALS, the patient is often prescribed a power wheelchair if the patient wants to be independent with mobility. Answer A encompasses answer B. C is not appropriate because motor function will continue to deteriorate. D is more appropriate for individuals with cognitive or perceptual deficits.

an OT is working with an individual who complains of hypersensitive fingers following a crush injury to the hand. Which of the following methods would be most appropriate for achieving desensitization? a. textured material, rubbing, tapping, and prolonged contact b. massage, functional estim, and a progressive desensitization program c. pressure, percussion, vibration, icing, and edema massage d. visual compensation and functional use of the UE

a. textured material, rubbing, tapping, and prolonged contact a program of desensitization typically includes these aspects. treatment is most successful when carried out and controlled by the individual. Answers B and C provide an ungraded or unspecific level of touch which would not be tolerated well by a person with hypersensitivity. D are techniques for a person with impaired sensation.

Results of an OT eval show that a young child has many tactile defensive behaviors. What is the MOST appropriate beginning activity for intervention for this child with sensory processing difficulties? a. the child pretends to be part of a sandwich between heavy mats b. the therapist applies lotion to the childs arms and legs c. the child is blindfolded and guesses where he or she is touched on the body d. the child and therapist play duck duck good circle game

a. the child pretends to be part of a sandwich between heavy mats This provides heavy touch pressure over the body, which most individuals with tactile defensiveness can feel comfortable with. Answer B is incorrect because tactile stimuli that are self-applied are better tolerated than therapist-applied sensations. C is incorrect because that may be threatening to the child. D is incorrect as it includes light touch that is provided by others.

which best represents a typically developing 12-15 month old childs functional mobility? a. the child uses a wide base of support and takes small, inconsistent steps b. the child uses a wide base of support and can navigate uneven ground c. the child uses a narrow base of support and low arm guard position d. the child uses a narrow base of support, can walk well, and can run for short distances

a. the child uses a wide base of support and takes small, inconsistent steps most infants are early in their walking development at this age. The other answers are typical of an older toddler.

A preteen with spastic CP wants to use a computer. Prior to evaluating computer needs, wat should the OT learn about FIRST? a. the student and family's goals for the device use b. the family's ability to afford a computer and its upgrades c. computer learning programs to facilitate the students participation in the classroom d. the students physical and cognitive capacities to determine appropriate keyboard and screen options.

a. the student and family's goals for the device use. Prior to the AT assessment, the team needs to understand the consumer's goals related to roles and the performance of activities to support roles. Although the other answers do address other relevant factors, they need to be considered in relation to the overall goals the AT is attended to achieve.

An OT manager in a large department is trying to determine which of the four staff therapists will be able to supervise a level II fieldwork student in the upcoming months. Which therapist CANNOT be assigned a level II fieldwork student? a. the therapist who graduated 9 months ago, has taken the fieldwork educator certificate program course, and has supervised two level I students b. the therapist with 2 years of experience, has taken the FEC program course, but has no experience supervising level I students c. the therapist who graduated 3 years ago, has supervised two level I OTA students, but has not taken the FEC program d. the OT with 5 years of experience who trained at a foreign university

a. the therapist who graduated 9 months ago, has taken the fieldwork educator certificate program course, and has supervised two level I students. An OT must be licensed for 1 year prior to accepting fieldwork level II students.

an individual has been referred to OT for prevocational assessment upon admission to a community mental health day program. When considering likelihood of success in the workplace, what are the MOST important areas to assess? a. time management, ability to accept feedback, following instructions b. attention, memory, and visual motor performance c. dressing, grooming, and hygiene skills d. the anticipated work environment and level of supervision

a. time management, ability to accept feedback, following instructions evidence suggests that the best clinical predictors of future work performance are ratings of a persons work adjustment skills such as appearance, punctuality, ability to accept feedback and follow instructions, and productivity. B and C should also be assessed to guide intervention, but these are broad areas and not necessarily predictive of success. Likewise, D should be assessed to see if modifications or supports are warranted.

A 13 y/o boy with paraplegia wants to take a bath without assistance from his mother. What would the OT most likely recommend to increase independence with bathing? a. a transfer bench with a handheld shower b. a hydraulic lift with a sling seat c. an inflatable tub and bath mitt d. a wheeled shower chair

a. transfer bench with a handheld shower a child with paraplegia would most likely be able to transfer himself onto the bench and a handheld shower would allow him to wash without having to adjust the faucet head. B would be indicated for an individual who is obese or has limited function of UEs and LEs. D would be for washing in a shower, not a tub.

An OT is working with a client with an ID in an alternative work environment. When a new member joins the work line the OT observes a sudden change in behavior- the clients eyes get wide, his voice gets high, and he repeatedly asks if he Is doing the job correctly. How should the OT respond to the client in this situation? a. use a directive approach, increase structure and repetition, and provide guidance to the client. b. reassure the client that you can see his worry about the new worker c. remove the client immediately from the work environment d. change the background music to something more soothing.

a. use a directive approach, increase structure and repetition, and provide guidance to the client. This individual demos symptoms of anxiety. Encouraging the client by providing reassurance, increasing the clients sense of control, and slowing the pace of therapy is a great method. It is important to validate the clients feelings of anxiety, however, prolonged validation of the clients anxiety should be avoided (answer B). C would be the last resort, as the clients goal is to be able to work. D may be beneficial to this client, but may not be beneficial for other workers who need a more stimulating environment.

The OT is working with an injured bus driver who is planning on returning to work soon. The bus driver works evenings and needs to sleep during the day. The client reports not sleeping well since the accident and asks the OT for suggestions. Which environmental adaptations are most important for the OT to recommend? a. use room darkening curtains and lower the temp b. use a fan to drown out noise and raise the temp c. have a glass of wine before bed and participate in a quiet activity d. recommend eval for a CPAP machine

a. use room darkening curtains and lower the temp environmental adaptations that can facilitate sleep include darkeneing the room, cooler temps, earplugs or a fan to drown out noise, and comfortable furnishings. A CPAP is used to control breathing for individuals with sleep apnea and is not an environmental modification. C may produce drowsiness originally, but after the sugar levels drop causing awakening.

following the use of a HOH approach with accompanying verbal cues to help a child practice bringing the spoon with food to her mouth, the OT is ready to reduce cueing. Which is the best method that represents the next step in cue reduction? a. using an index finger on childs wrist, OT guides hand with spoon to childs mouth b. OT cuts childs food into pieces for easier spearing c. as child brings spoon to her mouth, OT reinforces by saying "spoon in" d. OT implements a sticker chart to reinforce successful eating

a. using an index finger on childs wrist, OT guides hand with spoon to childs mouth researchers described a hierarchial approach to presenting artificial cues from least intrusive (verbal cues) to most intrusive (verbal AND physical cues). using a finger to guide the childs hand represents less physical guidance and also eliminates the verbal cue. Answer B is incorrect because although it may help to spear the food once it is cut, the focus of the intervention is bringing the utensil to the mouth. C is incorrect because all cues have been eliminated until the endpoint of the movement when the spoon nears the childs mouth. While providing a sticker chart may motivate her to improve results, it is not focused on changing the cueing.

what principles are most important to include when training an individual and joint protection? a. using the strongest joint and avoiding positions of deformity b. preparing muscles and joints with massage before exercise c. practice vivid imagery and relaxation exercises during difficult functional activities d. applying heat before treatment and cold after ROM treatment

a. using the strongest joint and avoiding positions of deformity These may help restore function and prevent further impairment. The other answers involve common muscle relaxation and stress management techniques not always associated with joint protection techniques.

An OT observes that in individual who had been doing well on a pureed diet has demonstrated a gurgle, or wet voice after swallowing a second time. What is the most appropriate recommendation for the OT to make? a. videofluoroscopy b. diet change to include thin liquids c. tracheostomy tube d. advancement to a regular diet

a. videofluoroscopy this is the most commonly used instrumental assessment to examine swallowing disorders. Answers B and D would be inappropriate for an individual who does not have a normal swallow or is showing signs of aspiration. C is usually in place prior to the initiation of a feeding pogram due to difficulty breathing- not swallowing.

an individual who uses a wheelchair and has limited financial support is moving into a new apartment and wants to select a floor surface for easy maneuverability. Which of the following surfaces is most appropriate for this situation? a. vinyl b. short pile carpeting c. deep pile carpeting d. several area rugs

a. vinyl these are the easiest and least expensive surface for wheelchair maneuverability. Although it is possible to find inexpensive short-pile carpeting, a smooth uncarpeted surface is still easier to maneuver over.

At a team meeting, a teacher reports that a child is having difficulty copying letters, completing paper mazes, and using scissors. This behavior MOST likely indicates a problem in which area? a. visual motor integration b. visual acuity c. visual tracking d. visual discrimination

a. visual motor integration This is the discrete motor skill that enables the coordination of the visual stimulus with the corresponding motor action. Visual acuity refers to ability to discriminate fine objects or details in the visual field. Discrimination refers to the ability to distinguish whether the item is different from others. C is the continued fixation of a moving object.

an OT is planning treatment activities to use for a child with postural instability. What would be the best preparatory activities to recommend? a. weight shifting and weight bearing activities b. activities that provide tactile and proprioceptive input c. activities while the child is prone over a wedge d. activities that increase the time child is upright against gravity

a. weight shifting and weight bearing activities A is correct as these are used to activate motor control. All postural movements, whether gross or subtle, occur with a shift of weight. The other answers are incorrect because the weight shift and movement are not clear.

an OT with expertise in hand rehab is assessing an OTAs services competency in hand function assessment. At what point is service competency established? a. when the OTA consistently obtains the same results at the OT b. after the OTA passes the NBCOT exam c. when the OTA has obtained a specified number of continuing education credits in in-hand rehab d. after the OTA has practiced for a minimum number of years in in-hand rehab as specified by state liscensure.

a. when the OTA consistently obtains the same results at the OT service competency relates to the specific knowledge, skills, and attitudes to perform at an expected level resulting in the same results as another trained professional.

a 3rd graders readiness for direct OT as a related service has been determined on the basis of: a. whether OT services support participation in the education program b. the degree of functional skills possessed by the child c. the level of independence in ADLs d. the degree of accessibility of the learning environment

a. whether OT services support participation in the education program related services are defined as services needed to help the students benefit from their education program. answers B and C are addressed in school programs only when they impact the students participation and ability to benefit from their education program. D is an important concern, but it would be covered by the consultation with the school or teacher, not through direct services provision.

An OT is assessing the ROM of an individual who actively demos IR of the shoulder to 70 degrees. How would the OT MOST likely document this measurement? a. WNL b. WFL c. hypermobility that requires further treatment d. limited mobility that requires further treatment

a. with a normal limits normal range of motion for internal rotation is 70°. within functional limits is noted when range of motion is adequate for functional activities but not necessarily complete range of motion. hypermobility is when range of motion is passed what it is supposed to be.

an agency placing an individual with mental health conditions in municipal jobs program has hired an OT to make recommendations for reasonable accommodations. Which of the following areas would be most essential to assess in order to make these recommendations? a. work environment, structure of work tasks, rules, and supervision b. architectural barriers within the worksite c. work aptitudes and interests of the persons to be placed in the work program d. capacity of the individuals to perform specified work tasks.

a. work environment, structure of work tasks, rules, and supervision this would be key to making recommendations. B would be irrelevant unless the population had physical disabilities as well. C and D would be relevant for placement recommendations for the worksite regarding this population.

During a self-care eval of an individual who recently sustained a brain injury, the OT instructs the individual to comb his hair immediately after he washes his face. The individual washes his face quickly, but then the therapist must give him several reminders to comb his hair. The OT is MOST likely to identify this as a deficit in what area? a. working memory b. judgement c. hearing d. abstraction

a. working memory working memory is the temporary storage of information while one is working with it or attending to it. It includes the ability to recall information immediately after exposure. It also allows one to focus conscious attention and keeps track of information as one is performing an activity. Judgment is the ability to make realistic and safe decisions and would not be needed for this task. Because the person performed the first request, hearing would seem intact. Abstraction is the ability to extrapolate information from an idea to generalize to another situation and would not be needed to follow this direction.

an OT PR campaign is being instituted in your state. It was suggested by the OT state rep that a committee be formed in order to educate potential OT clients at the local level. Which of the following would be the best strategy for targeting this market? a. write articles about OT for local newspapers b. hold an open house in your department and invite members of the hospital community c. provide an in-service workshop for nurses working at a local home health company d. conduct a workshop in your area of expertise at your local or state OT conference

a. write articles about OT for local newspapers In this situation, the target market is potential consumers of OT ie. patients and clients. The best answer of targeting these individuals would be for local newspapers that reach the general public. B and C are good examples of how referrals may be targeted. D is a good example of professional development but the impact would be limited to other OTs.

An OT who works in an outpatient facility frequently recommends that the individuals with Parkinsons disease consider taking part in a community-based therapeutic group. What is the PRIMARY reason for recommending group treatment? a. is more effective in preventing motor problems associated with the disease b. provides social interaction and support as well as activity c. is an effective way to present therapeutic exercise activities d. requires less therapist time because the therapist can leave once the group has started.

b provides social interaction and support as well as activity Parkinsons frequently causes social isolation because of the decreased mobility and communication. .Answer A is incorrect because group treatment is not necessarily better for addressing motor problems. Answer C can be correct, but is not the primary answer. D is incorrect because the therapist is responsible for leading the group and would not leave til the group is over.

An OT is planning intervention with a young teenager following 2nd degree skin burns on her hands and face. What other tool would add important information to what the therapist has already learned from the medical record, eval of the child's physical, and sensory function, and ADL performance? a. school record, including past grades and extracurricular activities b. COPM c. Jacobs prevocational assessment d. Ranchos los amigos scale

b. COPM It allows the OT to gather info concerning the teen's perception about the importance of various daily occupations, together with her rating of satisfaction with current levels of performance. It also establishes a baseline that can be referenced over time to contribute to the plan to document intervention outcomes. Answer A is incorrect, as this information is not a priority to help the OT plan intervention early in the course of burn recovery. Answers C and D suggest other potential assessments, yet these are incorrect as neither provides important info for planning at this stage in the rehab process.

An OT is initiating an eval of a preschool child diagnosed with autism spectrum disorder. What is the OT MOST likely to include in the evaluation process? a. one-to-one interview with the child b. observation of the child in a social, gross motor, and self-feeding task. c. the Peabody Developmental Scales-2 d. Assessment of the child's performance skills while outside on the playground.

b. Observation of the child in a social, gross motor, and self-feeding task. Recommended practices suggest that OTs conduct observations of clients with ASD before selecting and administering formal assessments because of the wealth of knowledge that is gained through observation. Answer A would not be the most valuable choice as communication difficulties are a core feature of ASD. It would be more helpful to combine an interview of the child in conjunction with parent-teacher interview. The Peabody may be something the therapist administers based on the recommendations following review of the observation data. Answer D is something the OT might consider, but a more comprehensive picture can be drawn from observing multiple areas of occupation.

An individual with bipolar disorder demos frequent verbal outbursts and has been referred to OT to work on developing social skills. When using a Dialectal Behavioral Therapy approach, what is the MOST appropriate format to use? a. individual treatment sessions in a quiet place such as the individual's room b. a group format following a specified protocol c. individual treatments that allow for flexibility within each session d. A group format that alternates leadership opportunities between group members

b. a group format following a specified protocol DBT uses a protocol based cognitive behavioral approach to work with individuals with complex psychiatric conditions in a group format. The skills taught in these sessions include mindfulness, interpersonal effectiveness, emotional regulation, and distress tolerance. Consumers benefit from the interpersonal interaction, support, and modeling of group members. Therefore, an individual format would be incorrect. A structure that alternates leadership would be inappropriate.

an individual uses a mouth stick when working with a computer. which of the following devices will prevent the mouth stick from accidentally striking other keys? a. moisture guard b. key guard c. an autorepeat defeat d. one finger access softwear

b. a key guard this is a device that covers computer keys and provides a guide for a finger or stick without punching extra keys. A is a flexible plastic cover that protects the keys from drool or moisture. C stops repetition of letters caused by overlong or involuntary depression of keys. D allows the user to lock out keys such as shift of enter.

a young mother is s/p TBI and demos deficits in sequencing and problem solving. She will need to resume cooking for her family. In her most recent OT session, she successfully made a peanut butter sandwich . What is the next meal prep activity that should be planned? a. a cold cheese sandwich b. a lettuce, tomato, and cucumber salad c. a casserole d. a spaghetti dinner with salad and garlic bread

b. a lettuce, tomato, and cucumber salad The following sequence is for grading up a meal preparation activity: cold meal, hot one dish meal, hot multi dish meal. Making a cheese sandwich is not an upgrade in this case. Answer B requires cutting and sequencing various ingredients.

An OT is implementing a self-feeding session with an individual with C5 SCI. Which piece of feeding equipment would be MOST appropriate for the OT to introduce? a. a wrist driven flexor hinge orthotic b. a mobile arm support c. an electric self feeder d. built up utensils

b. a mobile arm support. An individual with this diagnosis will be able to operate a mobile arm support for self feeding and hygine activities. This is a mechanical device attached to the wheelchair that supports the weight of the arm and reduces friction in motion. A wrist driven flexor hinge orthosis would be used for a lower-level injury such as C6-C8 in which the individual had functional use of the shoulder and arm muscles and fair plus or better wrist extension strength. This orthosis is indicated for individuals who lack prehension power. Built up utensils may be indicated for individuals with C8 or T1 SCI because they may lack strength to grasp regular utensils.

When evaluating an individual for phantom limb pain, the OT would expect the individual to report: a. feelings that the entire limb is still intact post-amputation b. a perception that the amputated extremity is painful, crampy, hot, or achy. c. feelings of sharp pain at the residual limb d. perception that the distal amputated limb is actually intact.

b. a perception that the amputated extremity is painful, crampy, hot, or achy Phantom sensations can be constant or intermittent. Answers A and D are associated with phantom limb, but not with phantom sensations. C may be due to a neuroma.

An individual with a complete high-level tetraplegia SCI is returning home. Which adaptive tech would the individual MOST likely require to ensure safety in the home? a. simple electronic aid to daily living (EADL) b. second generation EADL device with speakerphone c. remote control power door opener d. electric page turner

b. a second generation of EADl device with speakerphone The simplest EADL does not allow independence in operating appliances, lights, and so on through the voice control. A remote control power door opener would be useless if the individual is unable to call for assistance.

In preparing a patient with unilateral BKAs for discharge from a rehab facility, the most important adaptive equipment for the OT to recommend is: a. lightweight cooking utensils b. a tub bench and toilet rails c. long handled dressing devices d. reacher

b. a tub bench and toilet rails This makes bathroom transfers easier and safer and allows a person with LE amputations to transfer independently. Answer A are recommended for individuals with weakness or joint involvement of the UEs. C and D are incorrect because they are more likely to be recommended when compensating for hip or trunk flexion.

An OT is concerned about a child's inability to control flexion and extension of the arm when reaching for toys. He flexes or extends the arm too much, making accurate placement of the hand very difficult. When developing goals, what area is MOST likely in need of development? a. ability to isolate movement b. ability to grade movement c. ability to control speed of movement d. bilateral integration of arm movements

b. ability to grade movement. Children with poorly graded movements lack the ability to use the middle ranges of movement effectively. Answer A is incorrect because the goal would be appropriate for a child who cannot break up a flexion or extension pattern during a movement. C is incorrect because this goal is appropriate for a child who has difficulty with arm or hand movement being too fast or slow. Answer D is incorrect because this goal is appropriate for a child who has difficulty bringing both arms together or using them effectively in every day activities.

An OT has been hired to develop social skills training programs for persons with serious, persistent mental illness in a community mental health setting and needs to select a behavior to assess as an outcome measure. Improvement in which area would BEST indicate that the program was successful in achieving goals for this population? a. attain a balance of rest, work, play, and leisure b. achieve verbal and nonverbal communication skills c. identify areas for vocational exploration d. perform daily self-care and home management activities

b. achieve verbal and nonverbal communication skills this would be the most relevant behavioral outcome indicating program effectiveness in the area of social skills development. The other answers may all indirectly benefit as a result of improved social and communication skills, but these would not directly reflect positive outcomes for measuring effectiveness of social skill training programs.

After a radial nerve injury, an individual initially had trace muscle strength in the elbow extension. One week later, strength is noted to have increased to poor minus. The individual is ready for which activity? a. passively self ranging the injured arm through a full arc of motion b. actively extending the elbow in midrange 30-40 degrees with the forearm resting on the table c. pushing a cup filled with pennies with the back of the hand, with the arm resting on the table to full extension d. lifting a book placed on the back of the hand up off the table

b. actively extending the elbow in midrange 30-40 degrees with the forearm resting on the table. This is correct because muscles with poor minus strength would only be able to move through partial ROM in a gravity eliminated position. A is incorrect because PROM would not utilize the increased strength available because the muscle does not contract. Answers C and D are incorrect because both involve resistive activities and muscle with poor minus strength could not complete this against gravity or resistance.

An OT is planning a vocational intervention program to assist an individual in a community mental health day program develop skills needed for obtaining employment. Which of the following would be the MOST relevant intervention to include? a. self-assessment of work habits and personality characteristics b. activities focused on time and stress management skills as well as practice of job seeking strategies. c. educating the work supervisors about the individuals needs and offering environmental modifications to maximize performance d. expressive activities such as making a collage with pictures of different types of jobs

b. activities focused on time and stress management skills as well as practice of job seeking strategies. There are several aspects and phases to vocational programming. Answer A is a prevocational evaluation method used to help determine the individuals potential for work readiness rather than skill development. Answer C might occur after employment is obtained. Answer D provides opportunities for exploration of ideas about job possibilities, but would not directly develop job seeking skills.

An OT has provided a cup with a cutout area at the rim to a 6 y/o child with dysphagia. What is the BEST way to explain the purpose of the cutout in the cup to the family? a. slow the drinking process b. allow the chin to remain tucked when drinking c. allow the caregiver to control the flow of liquid d. minimize the biting reflexes when the cup is placed in the mouth

b. allow the chin to remain tucked when drinking tucking the chin toward the chest is recommended when a child has delayed swallow initiation. Methods the caregiver can use to control or slow the rate of liquid include using a drinking spout with a small opening, pinching a straw, or using a vacuum feeding cup. Plastic cups and plastic-coated utensils are best for individuals with a bite reflex.

The OT is working with a client who sustained an explosive related extremity injury. which of the following issues with the OT most likely be evaluating when receiving a referral for this type of injury? a. tympanic membrane rupture, dizziness, and sensitivity to noise b. amputations, fractures, crash injuries, and/or burns c. hypertension, renal contusion, and failure d. hemothorax, pneumothorax, and/or sepsis

b. amputations, fractures, crash injuries, and/or burns. answer at typically results from an auditory or vestibular explosive related injury, where answer c results from a renal injury, and answer D from my respiratory related explosive injury.

after administering an interest checklist, the OT documents that an individual has identified a few vague, solitary leisure interests. Based on this info, what is the best activity to use in the next session to focus on healthy use of leisure time? a. a leisure inventory assessment b. an activity exploring leisure opportunities and problems c. an activity that encourages the individual to sign up for social activities d. a calendar of community leisure activities for the next few weeks.

b. an activity exploring leisure opportunities and problems the OT can assist in developing self-knowledge of leisure and play preferences. Once the therapist has collected data about past and present interests the info can be applied to create interventions. A would be a duplication of services. C may not be effective because the individual has indicated no interest in these activities so far. D is premature at this point because the individual has not identified any goals

Which one of the following individuals would MOST likely need a transfer board? a. an individual who is unable to follow commands b. an individual who cannot bear weight on the LEs c. an individual who has good LE strength but is fearful of fatigue d. an individual who is able to perform a standpivot transfer

b. an individual who cannot bear weight on the LEs When an individual requires assistance because of decreased strength or balance, the sliding board is an appropriate device to include during transfer training. Answers A and C are issues related to the use of the sliding board. A slideboard would not be indicated if the individual had the strength to perform a standpivot transfer.

following a brain injury, an individual complains she is unable to concentrate on balancing her checkbook for more than a few minutes. what is the most appropriate intervention using a strategy training approach? a. Make sure the desk is free of clutter before balancing the checkbook b. ask questions such as what am I currently doing and what am I supposed to do next c. turn off television, music, and phone alerts d. first calculate all additions to the account, then calculate all subtractions from the account

b. ask questions such as what am I currently doing and what am I supposed to do next Strategy training for attention involves helping a person learn to control, monitor, or prevent the emergence of attentional symptoms using techniques such as self protection and self questioning. The other answers are also appropriate strategies but are environmental or task adaptations and not training strategies.

and OT is working with a family and their 1-month-old infant who have just enrolled in early intervention. which is most likely a first step in her initial home visit? a. update the developmental assessment completed two weeks ago by the evaluation team to establish a current baseline b.ask the parents to describe the infants daily schedule and identify routines that are going well and those that are challenging c. confirm the parents schedule and review the dates of planned visits for months before the next quarterly review d. handle the infant to assess muscle tone and provide visual, touch, and movement stimuli to observe sensory responses

b. ask the parents to describe the infants daily schedule and identify routines that are going well and those that are challenging. This is the essential first step when meeting new families. The other answers are incorrect as they conflict with the family-centered approach.

An OT is evaling an individual with a cognitive disability. The ability to complete a tile trivet following a sample is demonstrated, and an inability to utilize written instruction is observed. Which one of the following jobs would be MOST appropriate assignment in an employment training program? a. sorting plastic utensils into separate containers b. assembling packets that include a knife, fork, and spoon, and napkin based on a sample c. selecting matching shoelaces from a mixed pile and lacing them into a display card d. gluing labels on cans and placing them in an appropriate container according to color.

b. assembling packets that include a knife, fork, spoon, and napkin based on a sample. This suggests an ACL level 4. At this level, an individual is able to copy demonstrated directions presented one step at a time. Answer A reflects an individual functioning at ACL 3. C is more appropriate for ACL 5, and D can be performed by an individual who is at ACL 6.

When evaluating an individual with coronary artery disease for controllable risk factors, what is MOST important for the OT to include as part of the assessment? a. determine the individual's age and gender b. assess the individual's lifestyle and dietary habits. c. observe the individual for obesity and cholesterol levels. d. determine whether the individual has a family history of heart disease.

b. assess the individual's lifestyle and dietary habits. controllable risk factors include smoking, cholesterol, hypertension, sedentary lifestyle, obesity, diabetes, and psychological stress. OTs have expertise in working with individuals to address goals associated with lifestyle performance and dietary habits that can help prevent heart disease. Age, gender, and family history are all uncontrollable risk factors. A doctor would address the other three answer choices.

The OT has completed an eval on an individual with severe and persistent mental illness while observing the individual in a horticulture group. The client was able to share gardening tools for brief periods of time and participate in the planting activity for a few minutes. Occasional cuing was required from the group leader to allow her to interact appropriately with others. Which group level would best enable social participation for this individual? a. parrallel b. associative c. cooperative d. mature

b. associative at this level, individuals are able to begin to share and collaborate briefly with others. Individuals working with others would not be sharing with others as often. Individuals in a cooperative group are more interested in engaging with others and maintaining participation for longer periods of time. Individuals at the mature group level are able to participate independently.

an individual who had a THA (posterior lateral approach) is working on independence in LE dressing. which of the following instructions is most important to convey to this individual regarding safety? a. sit during dressing activities b. avoid internal rotation and adduction of the involved hip c. use a long handled shoe horn and dressing stick d. wear shoes with elastic laces.

b. avoid internal rotation and adduction of the involved hip precautions include avoiding flexion past 90, adduction, and IR for this type of approach. These precautions are most important for safety following this procedure. C would be appropriate, however, other forms of AE could be more beneficial based on the clients preference and function. The two items listed are not necessarily useful for all clients. A is recommended, but is not more important than following precautions.

An individual with a job installing carpeting experienced extrapyrimidal syndrome side effects after being placed on anti psychotic meds in the hospital. If this individual is to continue taking the medication after discharge from the hospital, which instruction is the MOST important? a. limit sun exposure as much as possible b. avoid use of power tools and sharp instruments c. get up slowly from standing, sitting, or sidelying position d. be aware of the dehydrating effects of caffeinated drinks and alcohol

b. avoid use of powertools and sharp objects extrapyrimidal symptoms are a set of possible neurological side effects most often associated with psychotropic meds. Common symptoms include extreme restlessness and involuntary muscle movements (rigidity, tremors, spasms). Photo sensitivity is another side affect but is less of a concern because the individual works indoors. Dry mouth is a common side effect of many drugs that is worsened by caffeine and alcohol. Of all the possible side effects, answer B is most important because it is the only side effect the client has experienced.

a toddler with feeding difficulties due to limited oral motor control and oral defensiveness now demonstrates the ability to eat dry cereals with milk. how would the OT progress the child's diet? a. applesauce and mashed bananas b. cut up meat and sandwiches c. strained fruits and vegetables d. scrambled eggs and pureed bacon

b. cut up meat and sandwiches To increase oral tolerance and control of food, textures are gradually modified from smooth and consistent to smooth and slightly varied, to increasingly resistive foods with a combination of contrasts.

an OT practitioner is instructing an individual with a total hip replacement (posterior- lateral approach) how to perform a passenger side car transfer. which of the following is the best method for entering the car? a. stand the body parallel to the car, hold on to a stable section of the car, lift and place the involved leg into the car, and slowly sit and follow with the opposite leg b. back up the body to the passenger seat, hold onto a stable section of the car, extend the involved leg, and slowly sit into the car c. back up the body to the passenger seat, hold on to a stable section of the car, flex both legs simultaneously, and slowly sit in the car d. back up the body to the passenger seat, hold onto a stable section of the car, flex the involved leg, and slowly sit into the car

b. back up the body to the passenger seat, hold onto a stable section of the car, extend the involved leg, and slowly sit into the car This transfer will prevent the hip adduction, IR and flexion. The other answers are contraindicated due to hip precautions.

At the end of an outpatient treatment session, a patient states that his medicare cap has been reached and he does not want to continue OT because he cannot afford it. Which of the following actions would be most appropriate for the OT to take? a. treat the individual per the physicians order and notify the office manager about the patients refusal b. based on his refusal, do not treat the individual and document the interaction in the chart c. treat the individual, but do not charge or document the service d. do not treat the individual and only charge for the time spent completing the chart review

b. based on his refusal, do not treat the individual and document the interaction in the chart the code of ethics states that an OT will respect the recipients right to refuse services permanently or temporarily. Answers A and C go against his will.

an OT is working with a school-age child to improve her power grasp technique. Which activity would the OT most likely encourage the child to perform to practice this grasp? a. pegboard activities over increasingly longer time periods b. brushing her own hair in the morning and evening c. carrying a lightweight book bag in her hands rather than on her back d. throwing a weighted ball at a target on a wall

b. brushing her own hair in the morning and evening a power grasp is used to control tools or other objects. Oblique object placement in the hand, flexion of the ulnar fingers, less flexion with the radial fingers and thumb in extension and adduction facilitate precision handling. A would most likely facilitate pincer grasp or tip pinch, C would be a hook grasp. C would encourage a spherical grasp.

An individual who sustained a mild brain injury continues to have difficulty with remembering appointments and activities. Which compensatory strategies would be appropriate for the OT to recommend to assist this person with memory following discharge home? a. visual imagery b. calendars c. mnemonics d. repetition

b. calendars interventions to improve memory include restorative approaches, memory aids, and strategy training. The most promising interventions include calendars, memory notebooks, and alarm clocks. Visual imagery techniques, mnemonics, and repetition or rehearsal, are all examples of restorative strategies.

A preschooler is having difficulty performing tasks requiring eye-hand coordination as a result of poor visual tracking skills. What activity should the OT use FIRST to promote visual tracking skills.? a. tossing and catching a water balloon b. catching and bursting soap bubbles c. throwing and catching a beach ball d. playing soft ball

b. catching and bursting soap bubbles this activity involves visually tacking a slow-moving target and requires minimal fine motor precision to accomplish a successful hit. The other answers also require visual tracking and hand-eye coordination but they involve faster-moving targets that require immediate, more precise movement. These activities can be used to promote advanced skills as the child's tracking ability improves.

A school-age child with fine motor skill difficulties is ready for discharge from outpatient OT services. What is the MOST important info to include in the discharge summary that will go to his school? a. child's interests and hobbies b. childs writing, dressing, and self-feeding performance c. childs academic achievement d. recommendations for the child to return to the clinic for follow-up services

b. childs writing, dressing, and self-feeding performance The info should focus on important occupations that are relevant for roles and performance this student will use in the school setting. Education personnel rely on information that helps them include the student in curricular and extracurricular activities. Answers A and D describe information that is relevant in the overall discharge plan but not specifically relevant to the information the school needs.

A long-term goal for an individual with back pain is to be able to return to work as an illustrator, which requires long periods of sitting. which of the following is the best example of a short-term goal for this individual? a. client will draw sitting at a work table. b. client will draw for 1 hour, taking stretch breaks every 20 minutes c. instruct client and stretching techniques to be performed every 20 minutes d. and struck client in the use of proper body mechanics that apply to prolonged sitting

b. client will draw for 1 hour taking stretch breaks every 20 minutes Answer A is not measurable and answers C and D describe what would be included in the plan section of the note.

An OT is conducting an interview with a patient upon admission to an inpatient psychiatric unit. The individual answers the question in an unfocused, tangential manner. In order to get as much information as possible before the session is over, what type of question should the OT incorporate? a. open-ended b. close-ended c. directed d. double

b. close-ended These can be appropriate when seeking specific info and tend to discourage communication. open ended questions tend to be more longer and detailed. Leading questions suggest the desired response, and a double question asks two questions at once to force a choice.

when treating individuals in the acute phase of cardiac rehab post-MI, it is important for the OT to first select activities that: a. prompt dyspnea b. decrease the effects of prolonged inactivity c. promote strength and ROM d. assist with independence in relearning daily activities

b. decrease the effects of prolonged inactivity some of the main objectives of inpatient cardiac rehab include decreasing the effects of prolonged inactivity. Phase 1 includes monitored low-level activity, including self care. Prompting dyspnea is contraindicated. Activities that promote strength are beneficial but ROM is not usually an area of concern. Most individuals do not need to relearn activities, other than applying EC techniques to activities they have already learned.

An individual preparing for discharge following a brief inpatient hospitalization for depression describes to the OT the type of services she would like to be involved in postdischarge. She is interested in structuring her day around work but does not feel she is ready for paid employment. She enjoys being in the company of others and does not feel she will need the support of a professional mental health provider. Which setting best meets her criteria? a. transitional employment b. clubhouse model c. partial hospitalization d. group home

b. clubhouse model The clubhouse model operates with a blend of staff and members to assume leadership for all clubhouse operations with an egalitarian and strength-based context in which members pursue personal goals related to their recovery. It is centered on the concept of the work ordered day and conveys the expectation of members and staff to run the clubhouse side-by-side at least 5 days per week. Members are not paid. Transitional employment may be provided by the clubhouse, these positions however, are paid. Partial hospitalization programs provide more structure and professional support for members who still need a higher level of care. Group homes are a type of residency in which the members can opt to participate in a clubhouse during the day.

during an interview following a THR, it is critical that the OT determines the individuals: a. marital status b. cognitive status c. leisure interests d. work responsibilities

b. cognitive status the individuals ability to safely perform ADLs and IADLs while maintaining safety precautions is related to cognition. The other answers may be relevant to the occupational profile and interventions, however, it is not as critical as safety.

A client with chronic pain is working with an OT through a workers comp program. Despite collaborative development of goals and strategies to begin volunteer activities, the individual has not yet identified a location at which to volunteer. Concerns about loosing workers comp benefits because of the volunteering has been raised. Using a motivational interviewing approach, what is the best action for the OT to take? a. reply that the client is ready and needs to make a decision b. collaborate to find a location and phone number c. reassure the client that the workers comp benefits will remain intact d. identify a volunteer opportunity for the client and schedule an appointment

b. collaborate to find a location and phone number motivational interviewing is designed for people who are not ready to change or are ambivalent about changing. The fundamental approach of MI are collaboration, evocation, and autonomy. A is a confrontational approach that can lead to defensiveness. C would be inappropriate as the OT cannot determine how long the individuals benefits will continue. D is not a collaborative or client-centered approach

When evaluating self-care performance with an individual with functional limitations in shoulder abduction and ER, which of the following is MOST essential for the OT practitioner to assess? a. buttoning a shirt b. combing the hair c. tucking in a shirt to the back d. tying a shoe

b. combing the hair An individual normally abducts and ER to comb the hair. Shoulder abduction is not required for buttoning a shirt or tying a shoe. Tucking in a shirt requires shoulder abduction and IR.

on which areas of function should an evaluation of a three-year-old child with moderate intellectual disability be focused? a. positioning and communication skills b. communication, self care, and social skills c. reading skills and independent self-help d. developmental writing skills

b. communication, self care, and social skills. early programs for children with intellectual impairment usually focus on the attainment of developmental milestones. answer A is a more appropriate focus for a child with significant intellectual disabilities who relies on a caregiver for assistance with all daily activities and care. answer c is incorrect as individuals with moderate intellectual disabilities can develop independence and ADLs and can achieve academic performance at a third to seventh grade level. however, these areas are not a focus of early preschool programs. answer D as in correct because it focused on skills expected in kindergarten programming.

a member of a discussion group frequently monopolizes the discussion and interrupts others. The OT has tried to give the client various indirect cues to decrease the disruptive behavior, but the client continues to monopolize the discussion. Which direct intervention should the OT implement next to modify the individuals behavior? a. sit beside the individual who is monopolizing the discussion and touch his or her hand or arm as a reminder not to interrupt others who are talking b. confront the individuals behavior and ask are you aware that your frequent interruptions prevent others from having a chance to contribute? c. redirect the individual and say now lets hear what others have to say about this d. restructure the task by selecting a group activity that requires sequential turn taking

b. confront the individuals behavior and ask are you aware that your frequent interruptions prevent others from having a chance to contribute? The individual has not responded to the more subtle and indirect cues. The OT must be able to provide clients with structure, direction , and feedback in a way they can understand. In this case, the OT must take a more directive approach. The other answers are examples of indirect feedback which have not worked in the past.

An OT is preparing an individual who recently sustained a partial thickness burn injury to perform a home program of positioning and use of orthosis. The MOST appropriate recommendation to prevent deformity would be to: a. discontinue the positioning and use of orthosis program upon returning home b. continue the same positioning and use of orthosis program that was indicated on discharge c. continue with the positioning and use of orthosis program only during the day d. continuing with the positioning and use of orthosis program only at night.

b. continue the same positioning and use of the orthosis program that was indicated at discharge It is necessary to continue this because active scar development continues for many weeks. Changes will be made as needed at follow-up visits. A would only occur if the individual had been hospitalized until the injury completely healed which is unlikely

An OT practitioner wants to provide functional activities as part of an individuals hand rehabilitation program. Which of the following activities are appropriate? a. active and self-care ROM techniques b. crafts, games, and self-care tasks. c. cone stacking, pegs, and pulleys d. mild, moderate, and resistive TheraBand exercises.

b. crafts, games, and self-care tasks The other answers are considered to be adjunctive activities that may be precursor to functional activities.

an individual with an anxiety disorder has been placed on a new anti-anxiety medication. While monitoring the individual over the next few days, the OT practitioner should be particularly observant for which of the following effects? a. akathisia b. decreased arousal and drowsiness c. extrapyrimidal syndrome d. tardive dyskinesia

b. decreased arousal and drowsiness medications for anxiety reduce anxiety but also may have side effects of drowsiness, loss of coordination, or mental slowing. The other answers are adverse side effects typically linked to antipsychotic meds.

The OT is instructing the client in tendon gliding exercises. Which of the following is most representative of a potential immediate outcome of performing a twice-daily tendon gliding exercise program? a. increased independent use of AE b. decreased edema and prevention of tendon adhesions c. increased ability to engage in fine motor coordination tasks d. increased ability to perform resistive tasks

b. decreased edema and prevention of tendon adhesions gentle muscle contraction, as seen with ROM encourages return of fluid and decreases discomfort in the acute stage of healing. The other answers are all potential outcomes as a result of implementing this program after the edema and potential adhesions are prevented.

the OT is assessing sensory awareness in an individual who had a right CVA. From observation, the OT notices the client using visual compensation to complete ADL tasks. Which of the following techniques is most appropriate? a. establish rapport, test the affected side followed by the unaffected side b. demo the procedure on the unaffected extremity, then occlude the individuals vision and test the affected side. c. occlude the individuals vision, demo the procedure on the affected extremity, then unocclude vision and test both sides d. interview the individual and assess only the areas that the client reported to be impaired

b. demo the procedure on the unaffected extremity, then occlude the individuals vision and test the affected side. the presentation of stimuli in sensory eval is extremely important. It is necessary to occlude vision to prevent compensation. Also, the unaffected side should always be tested first. Stimuli should be presented in a proximal to distal pattern and the whole extremity should be tested. Rapport should be established before beginning any eval.

To practice transfers using a transfer board, the OT must have the individual use a wheelchair that has: a. detachable foot rests b. detachable arm rests c. anti-tip bars d. brake-handle extensions

b. detachable arm rests. The arm rests must be removed for the slideboard to fit safely. Footrests may be swung away and do not necessarily have to be removed. Anti-tippers prevent the chair from tipping when going up or down a step. D allows for breaks to lock more easily but would be in the way of a slide board transfer.

an OT is treating an individual who has been refusing to wear his splint, and now reports it is lost. Prior to fabricating a new splint, the therapist should first: a. give the individual 48 hrs to locate it, then fabricate a new one if the first cannot be found b. determine whether there are any motivational or cultural issues interfering with the splint wear compliance c. ask the individual to find the splint and demand that he begin wearing it or you will call his physician d. discharge the individual because he has no interest in regaining function

b. determine whether there are any motivational or cultural issues interfering with the splint wear compliance Some individuals refuse to wear orthoses owing to cultural norms and others are simply embarassed to wear it. In this case, it is not necessary to fabricate a new orthosis unless the individual agrees to adhere to a wearing schedule and cannot find the other orthosis. C would not be the first step. D is not appropriate until the therapist discusses the case with the individual and the physician.

An OT has been hired by a community residence for women with mental health issues as a consultant to address the problem of low motivation and low activity levels in the areas of ADLs and IADLs. Which approach is MOST appropriate? a. provide OT to increase occupational performance in the areas identified b. develop a plan with staff to change the social environment to one that will enhance motivation an activity level c. design a range of living skills groups so that every resident will be included d. help the residents to achieve personal goals, make decisions, or change behaviors

b. develop a plan with staff to change the social environment to one that will enhance motivation and activity level Consultation is providing best advice or plan to assist the organization to meet the needs of its service population . The consultant is able to do this without becoming enmeshed in the day to day operations. Answer B is correct because it best represents this. A represents direct service. C represents a program development approach and D would be more of an occupational therapy intervention.

and OT is leading a group for individuals who have difficulty coping with chronic pain. based on a cognitive behavioral frame of reference, with which of the following steps would The therapist begin? a. providing the individuals with information on activity pacing to minimize pain b. developing awareness about how thoughts and behaviors affect ones perceptions and ability to cope with pain c. practicing relaxation techniques to be used when the individuals experiencing stress d. setting up a method to monitor and report on how successful the individual's use of recommended techniques was at home

b. developing awareness about how thoughts and behaviors affect ones perception of and ability to cope with pain. All of the answers are aspects of the cognitive-behavioral process. However, treatment begins with an educational approach. Therapy then moves to learning alternative ways of responding to pain, followed by rehearsal of techniques, and then implementation techniques and coping strategies to decrease pain and stress.

A 9 y/o with ID has received OT to become independent in dressing and feeding. He is now dressing himself and is ready for discharge. What is the BEST approach for the OT to take when developing home program recommendations? a. schedule a session with his parents to demonstrate specific therapeutic strategies for use b. discuss daily routines with his parents and explain how they can reinforce independence within those routines c. recommend his parents purchase oversized clothing with hook and loop closures and large snaps. d. advise his parents to give child verbal cues when he is dressing and help with closures only.

b. discuss daily routines with his parents and explain how they can reinforce independence within those routines The other options are not appropriate and providing additional prompts or different clothing may cause him to lose independence.

a male patient with an indwelling catheter asks the OT for advice concerning sexual activity. Which of the following responses is most appropriate? a. refer the patient to his physician b. discuss precautions necessary for sex c. teach the individual how to remove and replace the device d. explain that it is dangerous to avoid having sex

b. discuss precautions necessary for sex Sexual functions are within the scope of an OT. Sex is possible with an indwelling catheter, but some precautions should be taken. If it becomes kinked during intercourse, pressure should not be placed on the bladder. Urine flow cannot be restricted for more than 30 minutes. An OT may refer the patient to the physician if uncomfortable or unfamiliar with the specific education to provide the patient.

During perceptual evaluation, an OT determines that an individual exhibits constructional apraxia, body scheme disturbances, and unilateral neglect. During the functional part of the eval, these deficits are MOST likely to be exhibited as self-care deficits related to: a. spatial relations b. dressing apraxia c. anosognosia d. figure-ground discriminations

b. dressing apraxia difficulty with spatial relations is a problem wit awareness of the relationship of ones self to another object. A person with anosognosia is unaware of any deficits. Figure-ground discrimination is the ability to distinguish an object from its background..

When the OT suspects tactile defensiveness as a rational for a child's challenges, in what area of participation should the OT focus on FIRST? a. play behavior b. dressing habits c. social skills d. leisure interests

b. dressing habits Children with tactile defensiveness are bothered by tactile aspects of ADLs including specific types of clothing. The child may avoid wearing certain clothes. Conversely, some children may never take their shoes off to avoid tactile stimulation. Play behavior and social skills could be affected secondarily as a result of intolerance to certain textures or human touch. Knowledge of the child's dressing habits will give the OT key information at the start of the evaluation process.

during a coloring activity and OT observes a preschooler stabilizing a cramp between the thumb and first two fingers. how would the practitioner most accurately document this grass? a. pincer b. dynamic tripod c. palmar d. lateral pinch

b. dynamic tripod A pencil grass is characterized by opposition of the thumb and index fingertips to allow the child to make a circle with the fingers. The palm or grasp is a power grasp in which the individual flexes the fingers around an object while stabilizing it against the palm. In a lateral pinch, the individual places the pad of the thumb against the radio side of the index finger near the dip joint.

Prevention of cumulative trauma disorders in the workplace is the primary focus of an OT working as an industry consultant. What suggestions should the OT make that would have the greatest impact to reduce the risk of CTD in an industry in which there is heavy keyboard use? a. teach employees to identify early symptoms b. educate employees about ergonomic adaptations c. provide inexpensive resting orthoses to employees d. instruct employees in exercise routines to increase strength in weak UEs.

b. educate employees about ergonomic adaptations this would be an effective way to introduce this population to a change in task methods related to keyboarding that may prevent CTD. The other answers are incorrect because they represent interventions that might occur at some point following the onset of CTD.

Following eval of an elderly individual, an OT determines that the individual no longer leaves her bed primarily because of a fear of falling when moving from bed to the wheelchair. What is the most important concept for the OT to remember when working with this individual? a. teach the client to increase LE strength b. educate the client to increase confidence in the area of functional transfers c. modify the environment to reduce safety risks d. reduce the use of medications that may be contributing to falls

b. educate the client to increase confidence in the area of functional transfers the most important concept is that each client will be unique in his or her own assistance needs. The other answers are components that will help increase confidence in transfer performance.

an individual who alternately laughs and cries without a parent provocation throughout an evaluation would be identified as a exhibiting what type of behavior? a. mania b. emotional lability c. paranoia d. denial

b. emotional lability lability is the state of unstable emotions. And it may be one of the symptoms observed in individuals experiencing mania. Paranoia describes enduring beliefs about being harmed. denial is not acknowledging the presence of information

As the OT plans intervention for a parent with mild ID who has recently returned home from the NICU with her daughter who was born 6 weeks prematurely, the OT considers ways to help this mother develop positive parenting skills during feeding and dressing activities. Which is MOST likely going to help this parent? a. provide handouts that picture the sequence of steps required to prepare the baby's formula and launder clothes in the washing machines. b. encourage problem-solving about how she will respond when the baby is fussing during feeding and diaper changes c. consult with the service coordinator to ensure that caretaking supports are available for the mother d. practice preparing formula and washing clothes and include multiple opportunities for repetition.

b. encourage problem solving about how she will respond when the baby is fussy during feeding and diaper change. OTs should focus on helping parents with intellectual or mental health problems build problem solving skills since every day care for children requires constant problem solving. Answers A and D do not include problem solving. Answer is incorrect as there is no evidence in this question that the parent needs additional support.

An individual with cognitive limitations has been participating in a social skills training group to improve the ability to make conversation. Residential home staff report to the OT that they are seeing minimal progress. IN order to promote development of conversation skills, what should the OT do NEXT? a. have the client watch movies that include simple conversations b. encourage staff to practice conversation skills with the client in a local market with local people c. have staff provide only positive feedback d. instruct staff to discourage the individual from conversing with others until the skill has been mastered

b. encourage staff to practice conversation skills with the client in a local market with local people. The individual is not generalizing the skills gained in the social skills training group to every day life. Watching movies is not a focused enough activity to enable this individual to benefit from it. answer C and D would both be counterproductive.

an individual diagnosed with substance abuse experiences difficulty gluing two pieces of a birdhouse together, becomes increasingly agitated, and finally storms out of the room. Which approach would be best for the OT to use to facilitate occupational performance? a. provide a project with larger pieces b. encourage the individual to take a relaxation break c. make sure each piece of the birdhouse is clearly labeled d. move the individual to an area with fewer distractions

b. encourage the individual to take a relaxation break Many addicts use drugs and alcohol as a way to fill leisure time and manage stress or fill boredom. When this individual experienced stress, he was unable to cope with it. Relaxation techniques can help the individual decrease his or her stress response. A is an adaptation that would benefit individuals with coordination deficits. C would benefit individuals with poor visual perception. D would benefit individuals with attention deficits.

a goal for a child with neuromuscular disorder is to develop postural reactions for increased participation in play. Which activity best addresses this goal? a. introduce the pulley exercise to increase UE strength b. engage the child in reaching activities while seated on the floor c. encourage the child to engage in play dough activities while in the prone position d. engage the chlid to practice finger painting while in a supported seated position

b. engage the child in reaching activities while seated on the floor All postural movements occur with a shift in weight and occur in degrees of amplitude and various planes. OT can grade the weight shift demands through the location of objects as the child reaches The other answers are not necessarily related to postural reaction development, but instead strengthening, decreasing tactile defensiveness, and visual perception.

a multidisciplinary team is conducting a needs assessment to develop a falls prevention program for the frail hospital population. The most likely section of the needs assessment for the OT to perform would be: a. analysis of statistical data on the number of falls occurring each year b. environmental hazard analysis and falls risk during performance of ADLs c. eval of gait and mobility device use of those at high risk for falls d. assessment of medication impact on fall incidence

b. environmental hazard analysis and falls risk during performance of ADLs A is performed by risk management or human resources personnel. C is done by a PT. D would most likely be performed by a pharmacist.

in order to screen an individual referred for a work hardening program, the OT needs to locate background info about the individuals work history. The best method for obtaining detailed info about the individuals job requirements is to: a. interview the individual b. examine the results of a job-demand analysis c. look up the individuals job in the dictionary of occupational titles d. request info from the referring physician

b. examine the results of a job-demand analysis assessing physical demands of a job by job demand analysis is often beneficial. This is a detailed description of the physical, sensory, and psychological demands of a job. A is useful to obtain info about the patients perception of the injury, motivation to return to work, and sense of responsibility for rehab. However, the worker may not be able to give an objective, detailed, consise analysis of the job.

An OT has developed a work conditioning program for men and women who were previously homeless and exhibit generally decreased endurance. What is the FIRST part of a work conditioning program? a. work activities adapted to the level of their ability. b. exercise and limited work task simulation c. work tasks specific to the jobs they will be getting d. a full day of on the job training

b. exercise and limited work task simulation work conditioning programs emphasize physical conditioning, which addresses issues of strength, endurance, flexibility, motor control, and cardiopulmonary function. They are usually half-day programs. Nonspecific job simulated tasks such as carrying, pushing, pulling, are appropriate examples of work conditioning that will improve skills needed for a variety of physical labor jobs. Job specific work tasks are tasks aimed at specific jobs. Performing adapted work activities is an example of work hardening that is implemented after the clients achieve work-conditioning goals. Answer D would also occur after a work conditioning program has been completed.

During a home visit with a parent and her 11 month old who was born 4 weeks prematurely, the OT observes the child assume a quadruped position and rock back and forth momentarily. what does this behavior MOST likely indicate? a. perseverative behavior b. expected development c. low muscle tone d. delayed development

b. expected development Rocking while supported on hands and knees is expected between 7-12 months. This is referred to as mobility superimposed on stability, and this stage is essential in the development of coordinated antigravity movement.

an individual covered by Medicare who has been receiving OT and PT in the home is now able to transfer in and out of the car with supervision of a caregiver and visit friends 30 minutes away. OT services are still required to improve mobility, UE function, and home management skills. Which of the following actions should the OT practitioner take first? a. provide a home program and discharge the individual b. explain to the individual and caregiver that one must be home bound in order to be eligible for home care services c. refer the individual for outpatient therapy and provide a comprehensive discharge summary to the outpatient setting d. inform the PT of the individuals status

b. explain to the individual and caregiver that one must be home bound in order to be eligible for home care services. Absences from the home are permitted, but must be infrequent in nature, short in duration, or for the purpose of receiving medical treatment. Given that this individual is able to leave the home for a social visit, he is not considered homebound. The OT would first inform and explain this criteria to the individual and caregiver. After this has been explained, the OT would communicate with the PT and refer the individual to outpatient therapy. Simply providing an HEP and discharging would not meet the needs because he still requires therapy.

The administrator of an assisted living facility has asked an OT to help implement programming that will decrease the number of residents needing to move from the assisted living facility to nursing homes. The MOST important area for the practitioner to address is: a. adaptive equipment needs b. falls prevention c. meaningful use of leisure time d. balancing work, leisure, and rest

b. falls prevention falls present a major problem in the elderly. half of individuals older than 80 fall every year.

The OT is instructing the parents of a child with strong LE extensor tone in dressing techniques. What is the best way to position the child to make putting on shoes and socks less difficult? a. extend the childs hips and knees b. flex the childs hips and knees c. adduct the childs LEs d. flex the childs shoulders

b. flex the childs hips and knees Flexing hips and knees inhibits ankle plantarfleion through the key point of control (hip). A is incorrect because it would contribute to more extensor tone. shoulder extension would likely affect the hip and knee flexion more than ankle plantarflexion. D is incorrect because the abnormal pattern at the ankle is usually influenced by the hip.

An OT is working with a group of children in a preschool program. All of the children in the group are able to sit independently on the floor except for one child with CP. The child has told the OT that he wants to sit on the floor with his peers and not in his wheelchair during morning circle time. What would the OT MOST likely recommend for the child to use? a. hammock seat b. floor sitter c. wheelchair insert d. prone stander

b. floor sitter this may enable the child to sit with his or her peers. Answer A would not allow the child to feel as if he fits in. Answers C and D ignore the childs desire to be on the floor.

An individual who previously worked as a cashier in a clothing store has been referred to a work-hardening program following knee surgery. Limitations are present in standing tolerance and balance. The simulated activity that best prepares this individual to return to work is: a. moving piles of clothing from one end of the clinic to the other b. folding clothing and putting it in a basket. c. washing dishes while standing d. putting price tags on clothing while seated

b. folding clothing and putting it into a basket Work hardening programs prepare individuals to return to work by combining work simulation, strengthening, and behavioral components. A cashier must stand during to job, remove clothes from a hanger, folds it, scans it, and puts it in the bag. Answer D does not include these critical job components. C incorporates standing but is not related to the job. A involves walking, not standing statically.

What is the method MOST important to apply when conducting a SEMI-STRUCTURED interview? a. utilize questions to influence a clients thinking and facilitate self-reflection b. follow a predetermined protocol, adjusting the process as needed c. ask questions exactly as they are stated on the eval tool d. use good timing and attention to the clients boundaries.

b. follow a predetermined protocol, adjusting the process as needed generally occupational therapists administer two basic types of interview. The first type includes a semi-structured interview in which a therapist follows a predetermined protocol and asks a set of questions designed to probe for specific information. skillful interviewing also involves good timing and attention to boundaries and utilizing questions to influence a client's thinking to facilitate self-reflection, regardless of the type of interview. standardized assessments require the therapist to follow protocal exactly.

which practice should the OT follow when administering standardized tests to a 4-year-old preschool child? a. test in a simulating environment that keeps a child alert and engaged in test items b. follow the test manual directions and attend to any deviations that are made c. report testing was not possible once the child fatigue during initial session and did not complete items d. using courage and conversation and give breaks as needed to keep the young child at ease

b. follow the task manual directions and attend to any deviations that are made when administering a standardized test, directions from the test manual should be followed closely to ensure reliability of test results. The test environment should be free of visual or auditory distractions or the child may have difficulty concentrating. therefore answer a testing in a stimulating environment is incorrect. answer c is incorrect because there are oftentimes when a child's fatigue, behavior, or a time constraints make it impossible to complete the test in one session, and most tests provide guidelines on how to perform the test in two sessions. answer D as in correct because although the overall success of an evaluation can depend on the OT's ability to establish rapport with the child and family can cause too much conversation that may be distracting.

A child is lacing a series of geometric beads by copying from a stimulus card and is unable to identify a mood-shaped bead when it is turned sideways on the table. What area of visual perception is MOST difficult for the child? a. figure ground b. form constancy c. position in space d. visual sequencing

b. form constancy Form constancy is the recognition that forms and objects remain the same in various environments, positions, and sizes. Answer A is incorrect as figure ground is the ability to perceive a form visually, and find its form when hidden in a busy area. position in space is the determination of the spatial relationship from figures and objects to ones self or other objects. Visual sequencing is an activity that requires the ability to copy the same sequence of beads.

To develop a preschool child's letter recognition skills, what would the OT MOST likely encourage the child to do? a. use flashcards with the bright colors b. form letters out of clay c. match cut-out letters to a sample d. color large letter outlines

b. form letters out of clay Preschoolers learn best from a multisensory approach and they should be encouraged to feel shapes, letters, and words through their hands. Furthermore, recognition of letters occurs before actual letter writing. The other answers rely primarily on visual processing, and these approaches are better suited for strengthening existing skills instead of building new skills.

an OT is selecting treatment activities to use with a young adult diagnosed with schizophrenia that would help to increase the ability to receive, process, and respond to sensory info. What are the most suitable types of activities to address this area? a. social skills training b. gross motor exercises c. life skills d. expressive projects

b. gross motor exercises the SI approach often uses gross motor exercises to integrate tactile, vestibular, and proprioceptive senses to address postural instability, gravitational insecurity, oculomotor dysfunction, and oral motor difficulties. Answer A could also be appropriate for the purpose of developing interpersonal skills. C would be used for addressing self-care skills and independent living goals, and D might be used when the individual needs opportunities for nonverbal communication and outlets for emotion and creativity. Although these interventions may be a part of the overall treatment program, this individual would benefit from being able to receive and process sensory info before embarking on more complex treatment formats.

The OT is working on lower body dressing with an individual who is s/p CVA and has some residual cognitive deficits. When using a forward chaining approach to dressing with pajama bottoms, how should the OT begin? a. using HOH, allow the client to complete as much of the activity as possible. b. guide the client to position the pajama bottoms in front and insert one leg c. assist the client with all but the very last step of the activity d. review all the steps of the activity prior to beginning.

b. guide the client to position the pajama bottoms in front and insert one leg When using the forward chaining approach, the therapist prompts or demonstrates the first step on the first trial, the second step on the second trial, and so on and so forth. Answer B would be the first step. In backwards chaining, the training starts with the last step (answer C). HOH guidance and having the client state the steps are other methods that may work but are not examples of forward chaining.

An OT is simulating cylindrical grasping activities with an individual who desires to work on the skills necessary to be a carpenter. Which of the following activities would MOST likely address these needs? a. positioning a nail on a piece of wood b. hammering a nail into a piece of wood c. carrying a pail of bolts d. unscrewing a lunchbox thermous

b. hammering a nail into a piece of wood This is the most common static grasp pattern and is used to stabilize objects against the palm and the fingers with the thumb acting as an opposing force. This pattern is used for gripping a hammer, pot handle, drinking glass, or handhold on a walker or crutch. A is a tip prehension, C is a hook grasp, and D is considered a spherical grasp.

An individual in a partial hospitalization program who is interested in obtaining employment is observed grabbing tools from others, acting out of turn, and rejecting feedback. Which of the following occupation-based interventions is MOST appropriate for addressing this individuals skills and deficits and preparing him for a work environment? a. operating the photocopy machine in a clerical group. b. handing out trays and utensils to people in a food service group c. placing books back on the shelves in a library group d. reconciling income and expenses at the end of the day in a thrift store group

b. handing out trays and utensils to people in a food service group This individual demos limitations in the area of interpersonal skills which are necessary for successful employment. This job requires minimal interaction but provides opportunity for practice with others at a limited level in a natural environment.

an OT working in a psychosocial rehab program has been asked to design a leisure-skills group for individuals functioning at the ego-cooperartive group level. It is mid december and the OT wants to utilize a holiday theme. Which would the most appropriate activity be for this group? a. provide a list of holiday songs for sing along b. have the group create a list of their favorite holiday songs and set a date for a performance c. organize a holiday cookie swap in which each group member brings a dozen cookies to contribute d. set up a reminiscence group in which participants express their memories of past holidays

b. have the group create a list of their favorite holiday songs and set a date for a performance egocnetric-cooperative groups are characterized by group members selecting, implementing, and executing relatively long-term tasks through joint interaction. The task remains essential, but satisfaction of some social-emotional needs of fellow group members is encouraged. A, C, and D are one time occurrences and lack long-term cooperation.

An individual with complete C4 tetraplegia is able to independently use a mouth stick to strike keys on a computer keyboard for 3 minutes. To upgrade this activity, the OT practitioner should a. provide a heavier mouth stick b. have the individual work at the keyboard for 5 minutes c. progress the individual to a typing device that inserts into a wrist support d. teach the individual how to correctly instruct a caregiver in use of the keyboard

b. have the individual work at the keyboard for 5 minutes. increasing duration of tolerance is the most appropriate way to progress the client. A heavier mouth stick would make the task more difficult and yield no benefit. An individual with a C4 tetraplegia would not have the potential to use a typing device that inserts into a wrist support. D would be downgrading the activity.

A client who presents with little to no AROM in the Left shoulder after a CVA requires the OT to perform PROM. How would the OT proceed? a. teaching the client to utilize a wall-mounted shoulder wheel b. having the OT move the shoulder joint through its full ROM c. Using an overhead pulley system d. training with an arm ergometer.

b. having the OT move the shoulder through its full ROM. a client with little or no active ROM would require PROM. the other answers are all representative of strategies to elicit AROM or AAROM.

an individual with moderate cognitive limitations lives in a group home and shares a bathroom with two other clients. He is able to open the toothpaste, apply it to the toothbrush and brush his teeth independently. But he has difficulty identifying which toothbrush is his and occasionally uses someone elses toothbrush by mistake. What modification would best meet this individuals needs? a. using an electric toothbrush b. having the only red toothbrush c. putting a built up handle on his toothbrush d. requesting a caregiver brush his teeth twice a day for him

b. having the only red toothbrush this individual appears to be at ACL 4.4-4.6. These individuals can display a heavy reliance on simple visual cues. The environmental adaptation strategy offers a strong visual cue. An electric toothbrush will require new learning. Built-up handles are for individuals with difficulty grasping. D is not desirable if the individual is able to do it himself.

An OT is working with a client with a C4 SCI to determine the best way for him to operate a power wheelchair. What is the MOST effective recommendation for this OT to make? a. no assistive device would be suitable for this level of injury b. head, chin, or breath control device c. generator/backup battery device d. mouth stick

b. head, chin, or breath control device no device and a backup battery/generator would likely be used for individuals with C1-C3 injury dependent on a ventilator. Answer D is usually for someone to operate an environmental control unit.

The OT observes that an individuals brain injury causes difficulty using a bus schedule to get to work on time. What would be the FIRST step toward developing this skill, using a problem solving training approach? a. generate as many solutions as possible for the problem b. identify using a bus schedule as a problem area c. weight the pros and cons of various solutions d. recognize the faulty solutions, self-correct the errors, and develop alternate hypotheses.

b. identifying using a bus schedule as a problem area This is a systematic approach involving 5 steps: problem orientation/identifying and analyzing the problem. problem definition and formulation generating alternatives and solutions decision making and solution verification and evaluation

A high-functioning 10 y/o with a behavior disorder has innately difficult temperament. Which treatment approach is MOST appropriate for the OT to initiate? a. utilize the time-out consistently in response to maladaptive behavior in the classroom. b. help the child develop cognitive strategies for anxiety-producing activities c. help care providers develop an unpredictable routine to promote problem-solving strategies d. provide a play environment in which the parent and child acts out conflicts.

b. help the child develop cognitive strategies for anxiety producing activities B helps the client engage in challenging activities and encourages him to use intellect to support positive behavior and participation. A is not effective for this childs capacities, nor does it help him practice positive behavior. C is incorrect because unpredictable routines are not helpful. D is incorrect because the parent and child need to learn mutual play in an environment that promotes positive engagement.

An individual who had a stroke is copying a picture of a clock. The drawing appears as a lopsided circle with a flat side on the left. The numbers 1-8 are written in numerical order around the right side of the clock. The hands are correctly drawn on the clock to represent 3oclock. The individuals performance appears to demonstrate: a. right hemianopsia b. hemi-inattention c. cataracts in the left eye d. bitemporal hemianopia

b. hemi-inattention this is the inability to respond to perceptions on one side of the body. Answer C would cause blurry vision. Answer D causes tunnel vision or the loss of peripheral vision.

When fabricating an orthotic device for an individual with swan-neck deformities, the orthotic should be designed to prevent what positions? a. hyperextension of the PIP and DIP joints b. hyperextension of the PIP joint and flexion of the DIP joint c. flexion of the PIP joint and hyperextension of the DIP joint d. hyperextension of the MP joint and flexion of the PIP joint

b. hyperextension of the PIP joint and flexion of the DIP joint answer A may be seen with lower neuromotor palsies. C is descriptive of the boutonniere deformity. Answer D would result from an individual who has overstretched the volar plates.

An OT is helping the parent of a 4 y/o boy with autism identify strategies to support grooming and hygiene. They are focused on ways to overcome difficulties during toothbrushing routines. What should the OT recommend? a. advise that many children with autism outgrow difficulties with toothbrushing b. identify and trial several different strategies, such as various toothpaste flavors, use of electric toothbrush, and pictures to show steps. c. give the child a choice between brushing his teeth and another, less-preferred activity. d. vary the schedule so the child doesn't begin to associate negative activities with bedtime.

b. identify and trial several different strategies, such as various toothpaste flavors, use of electric toothbrush, pictures to show steps. Children with ASD can learn to be independent with assistance of visual schedules, skills acquisition, task modifications, and learning techniques. Answer A is inappropriate as it does not provide the parent with support for the current concerns. Answers C and D are incorrect as they do not establish clear expectations for desired behavior nor do they represent the consistency that benefits many students with ASD.

an OT has been asked to design a series of leisure skill development sessions for individuals in recovery from substance abuse. What activities should be included in the first session? a. introduce healthy alternatives for leisure activities b. identify leisure skill strengths and weaknesses c. encourage problem solving d. provide strategies for healthy social interaction

b. identify leisure skill strengths and weaknesses all answers include strategies that are important for individuals in recovery from substance abuse. However, B and willingness to do something about it are the first steps. Additional sessions could focus on the other answers.

a school based OT has been asked to develop a suicide prevention program for teens. What would be the best program goal for this age group? a. create a volunteer program in a readily accessible environment b. identify peer leaders who can model and train their peers to identify adults they can trust c. collaborate with the school faculty to develop a filmmaking course for school credit d. create an after-school tutoring program to develop self esteem

b. identify peer leaders who can model and train their peers to identify adults they can trust this is most appropriate because they often do not know where to turn and increasing their comfort in asking for help or identifying individuals they can trust could potentially save lives. A would be appropriate for older adults. C and D may both be age-appropriate but they may not be meaningful to many adolescents and they do not target suicide prevention.

An OT is performing an assistive technology intervention with an individual who demos severely impaired motor performance. What should the OT do first? a. research the most appropriate commercially available forms of AT b. identify the individuals abilities, needs, and goals c. select the appropriate method of accessing the technology d. modify the AT to meet the needs of the client

b. identify the individuals abilities, needs, and goals this should occur before any other step in the process in order to make a match between the individuals abilities, environmental demands, and the appropriate technology. All other answers would come later in the process.

An OT practitioner is designing a stress management series using CBT approach for individuals who have chronic fatigue syndrome. What should the FIRST module in the series include? a. teaching time management techniques b. identifying thoughts and beliefs that contribute to negative feelings c. provide aerobic exercise d. teaching how to perform progressive resistive exercise.

b. identifying thoughts and beliefs that contribute to negative feelings Guided self assessment of each individuals stressors and stress reactions is the first step I a stress management program. The goal of CBT for this population is on helping a client become aware of his/her thought process to identify and change negative behaviors and emotions. Time management techniques ma comprise one of the following sessions. Aerobic exercise is an inappropriate method for individuals with CFS Progressive relaxation exercises involve systematic tensing of muscles which are not appropriate for individuals with HTN, cardiac disease, upper motor neuron lesions, or spasticity.

the OT is working on feeding skills with an individual with ALS who is in the late stages of the disease process. Which of the following is the MOST appropriate intervention for this individual? a. provide a rocker knife, plate guard, and nonskid mat b. implement a pureed diet and allow adequate time for eating c. emphasizing UE strengthening d. minimize use of AE

b. implement a pureed diet and allow adequate time for eating As ALS progresses, speaking and swallowing become more difficult and a pureed diet becomes necessary. AE is provided much earlier in the disease process. C would be contraindicated at this stage in the disease.

members of an OT group at a psychosocial rehab program who have physical disabilities in addition to mental illness began expressing anger about the lack of accessibility in the building where the day program is housed. Which of the following offers the action that will most empower the group members? a. employ an OT practitioner to assess the facility for ADA compliance b. incorporate advocacy skill training into the group format c. bring a lawsuit against the facility for violating the ADA d. involve the clients in a stress management group

b. incorporate advocacy skill training into the group format it is important for OTs to prepare our clients to advocate for themselves. Stress management techniques would not be as powerful in this situation. A is a possible option but because the OT leading the group can perform the assessment, it would not be necessary to hire an OT. C would be the last resort after many other attempts have been made.

An OTA and OT are planning to discharge a child from an early intervention program. What advice to the patents will MOST likely promote parents' ability to support their young child's continued development? a. set aside certain time daily to focus on developmental stimulation activities b. incorporate developmental stimulation activities into family routines c. provide therapeutic activities on an as needed basis d. do therapeutic activities daily, but vary the time of day.

b. incorporate developmental stimulation activities into the family routines These types of interactions are most successful. Separating therapeutic activities can take up extra time and energy and may interfere with family life. Activities provided on an as needed basis will never become habits.

An individual has been instructed to place towels, one at a time, on a high shelf to improve shoulder function. The individual is able to easily place 10 towels. Which of the following modifications would MOST effectively improve endurance in the shoulder flexors? a. place the towels on a higher shelf b. increase the number of towels from 10 to 20 c. place the towels on a lower shelf. d. add a 1 pound weight to each arm.

b. increase the number of towels from 10 to 20. Endurance is improved by increasing the number of reps so the muscles have to work over a longer period of time. Placing towels on a higher shelf would help increase ROM. Placing them on a lower shelf would downgrade the activity. Weights help increase strength but do not increase endurance.

Which of the following reflects a primary goal area appropriate for wellness programs with the older adult population? a. augmenting strength following strokes b. increasing overall physical activity levels and fitness c. enhancing social interactions for depressed elders d. increasing independent performance of ADL and mobility

b. increasing overall physical activity levels and fitness Wellness programs focus on developing personal control of behaviors through educational approaches and active participation in activities that promote health. The other answers improve performance in specific deficit areas rather than promoting general good health.

An individual was unable to achieve the following goal: "the client will initiate two requests to other group members for sharing materials within a 1-week period." What would be the BEST revised goal for this client? a. initiate two requests to other group members for sharing materials within a 2 week period b. initiate one request to one other member for sharing group materials within a 1 week period. c. greet the other group members at the start of each group session d. greet the group leader at the start of each group session

b. initiate one request to one other member for sharing group materials within a 1 week period Reducing the number of requests and the variety or number of individuals with whom the client is expected to interact with is the best way to simplify the initial goal. A does not make the goal easier to achieve. Increasing the number of individuals and requests also makes the goal more difficult to achieve. Changing the interactions to those with the group leader moves the goal away from the original problem are.

when ordering a wheelchair for an individual who has medicare, part B, the OT must be sure to document that the wheelchair: a. increases functional independence b. is medically necessary c. maintains patient function d. reduces deformity

b. is medically necessary Medicare payments are based on the type of service provided. Type B also covers some DME. The other answers are all broader statements of medical necessity.

in using an assessment that is norm referenced for children, what assumption can the OT make about the test? a. it measures typical behavior of children b. it compares performance with a normative sample c. it is valid and reliable d. it was standardized on groups of typically developing children

b. it compares performance with a normative sample. These tests have been given to a large number of people in a specific population that is called the normative sample. Norms or average scores are then derived from this same. A and D are incorrect because although norm-referenced tests are often developed with samples of typical individuals, some tests include smaller subsamples of clinical populations.

an OT is working with a client experiencing anxiety after being in a physically and mentally abusive relationship. The treatment facility utilizes a psychodynamic object relations approach. Which of the following interventions would be most consistent with this type of approach? a. meditation and yoga poses b. journal and diary writing c. personal hygiene and grooming classes d. aerobics and fitness program

b. journal and diary writing this FOR provides a framework by which therapists can use expressive techniques in collaboration with the client to develop insight and understanding and facilitate adaptive behaviors to meet goals. The other answers would benefit self-confidence and stress management needs but they are not expressive activities.

An OT is working with an individual with Alzheimer's disease who demos mild to moderate decline. She lives with her husband who works during the day. The OT should initially focus intervention on which of the following areas? a. ability to chew and swallow b. kitchen safety c. anger management d. recognition of family members

b. kitchen safety short term memory loss is one of the earliest symptoms of Alzheimers. The individual would demo moderate memory loss, decreased concentration, and difficulty with problem solving. Intervention would focus on analyzing and adapting meaningful leisure and home management activities to allow the client to safely participate. Because the husband is still working, the individual may want to continue preparing meals. There may be a need for anger management strategies but this is a secondary concern. A and D may be necessary in later stages of the disease.

An OT is working with an older adult who has diabetes and subsequent issues related to vision, specifically contrast sensitivity. Owing to this, he is having difficulties discriminating between medications each morning. The BEST adaptation for the OT to provide is: a. braille labels b. labels with white print on a black background c. a pill organizer box d. brightly colored pills with each type of medication a different color.

b. lables with white print on a black background contrast sensitivity is the ability to detect detail when gradation between an object and the background Is subtle. With poor contrast sensitivity, it would be impossible to discriminate between the background and the object. Answer A is not appropriate for individuals with peripheral neuropathy because they have decreased sensation on the finger tips. Answer C is useful for taking pulls on a schedule and is mostly for people with cognitive deficits. Brightly colored pills would make it easier, however the therapist has no control over this.

which of the following best meets the criteria for an individual to be a candidate for a mobile arm support? a. diagnosis of quadriplegia b. lateral trunk stability c. fair plus elbow flexion d. at least 90 degrees of passive shoulder flexion and abduction

b. lateral trunk stability criteria for the MAS include adequate power from neck, trunk, shoulder girdle, or elbow muscles, adequate motor control, 0-90 degrees of PROM in shoulder flexion and abduction, and PROM in IR/ER, trunk stability and supportive environment. A person with this level would be able to stabilize the elbow on the table to bring the hand to the mouth and would have enough strength to move the arm without the mobile support.

An OT is working with an individual in a work program setting. What is the FIRST step to achieving the program objective of preventing reinjury? a. performing a pre-work screening b. learning proper body mechanics c. participating in work hardening d. engaging in vocational counseling

b. learning proper body mechanics This is one of the first steps to reducing the risk of reinjury in a work program. Answer c, work hardening, is appropriate to implement after the physical demands of the jobs specific tasks are achieved. Answer D is appropriate after it is determined that a client cannot return to the same job or employer. Answer A is typically completed by the practitioner before the employer offers the new employee a job.

An OT qualitative researcher has more than 80 hours of narrative interviews related to a sequence of life events that were shaped by sociopolitical events in a client diagnosed with polio. This type of research is commonly referred to as: a. ethnographic b. life history c. phenomenological d. participatory action

b. life history This is an important design in naturalistic inquiry that uses a narrative strategy. Answer A is typically used to understand a culturally related topic. Answer C examines the lived experience of an individual or group. D is founded in the principle that those who experience a phenomenon are most qualified to investigate it.

an OT is running a sensorimotor group in an adult day program for individuals with chronic mental health conditions. Using Ross's Five-stage group approach, which would be the most appropriate activity to follow up the introduction and orientation? a. task activity such as making a fruit salad b. line dancing involving movements in 3 planes c. group planning for the next session d. group members taking turns introducing themselves

b. line dancing involving movements in 3 planes The approach begins with introduction and orientation. then is followed by maximum exertion in movement incorporating all 3 planes. Phase 3 addresses perceptual motor skills, 4 is the time for cognitive stimulation and 5 provides resolution and termination. Answer A would fit best under stage 3 as it is not as large of a motor task. C belongs in stage 5

an OT is developing a retirement planning protocol for a group of older adults at a community senior center, based on the transtheoretical model. Which activity would be most appropriate for individuals in the precontemplation stage? a. identify one life role you wish to continue after retirement b. list five potential changes you will face following retirement c. list the steps you need to take to accomplish one of your goals d. choose one goal to address for this session

b. list 5 potential changes you will face following retirement individuals in the pre contemplation stage have only a vague awareness of the need to change. Identifying potential changes helps develop awareness. Answer A is appropriate for the contemplation stage, D is representative of the preparation phase and C represents the action phase.

an OT consulting with a drop-in shelter for homeless women is developing a leisure activity group. The OT has observed high levels of anxiety and distrust in the residents. One of the goals of the group activity is to foster a sense of competence and mastery over the environment. Which of the following activities best address this goal? a. participating in a small group collage activity b. making leather lacing change purses from kits c. going to see a movie at a local theater d. presenting a residents talent show

b. making leather lacing change purses from kits All other choices are good leisure group activities. However, activities that can be completed in one session, are structured for success, and yield a tangible end product are likely to turn out well regardless of the individuals skill level. They are best for promoting a sense of mastery over the environment. D can promote self-esteem and self-expression but it would take more than one session to prepare

a client presents with post cerebral vascular accident edema in her dominant hand. The OT has introduced manual edema mobilization method and attempts to activate the lymphatic system. which of the following is the most representative of what the OT would perform via the MEM program? a. retrograde massage, use of orthosis, and positioning b. massage, compression bandaging, exercise, and external compression c. ultrasound, fluidotherapy, and AROM d. elevation, use of orthosis, PROM, and desensitization

b. massage, compression bandaging, exercise, and external compression The goals are to stimulate the initial lymphatics to absorb excessive fluid. The other answers are not representative of MEM.

When an individual with neurological deficits sits down to read a magazine in the living room of her group home, she becomes distracted by the conversations of her housemates. Which of the following activities BEST addresses the underlying cognitive problems? a. playing a simple, repetitive card game in a quiet environment b. measuring ingredients for a recipe while there is music playing c. referring to a catalog and filling out a catalog form d walking and bouncing a ball simultaneously

b. measuring ingredients for a recipe while there is music playing. The client demos difficulty in attending to the activity because of the presence of environmental stimuli. This suggests a deficit in selective attention. Answer A provides practice in sustained attention. Answer C provides practice in alternating attention. And C practices dividing attention.

an OT is treating an individual who demos progressive weakness and atrophy of the thenar muscles and numbness and tingling in the thumb and index and middle fingers. The client also complains of difficulty while grasping a coffee cup. The individual is not experiencing proximal UE limitations, therefore, the OT will most likely predict problems with which of the following? a. ulnar nerve b. median nerve c. radial nerve d. brachial plexus

b. median nerve the median nerve passess through the carpal tunnel at the wrist. Impingement in this region causes sensory changes in the thumb, index finger, long finger, and half of the ring finger. C causes sensory damage only. A causes decreased grip strength and complete or partial loss of sensation over half the fourth digit and all of the fifth digit. A brachial plexus injury may result in damage to all or any of the UE peripheral nerves.

An OT wishes to identify how a patient spends his leisure time, which leisure activities he especially enjoys, and which others he has participated in that he would be interested in renewing. The MOST appropriate tool for this purpose is: a. KELS b. modified interest checklist c. OT-Quest d. Leisure satisfaction scale

b. modified interest checklist this is frequently used to initiate discussion of how a patient usually spends his time to identify areas of specific interest. although answer a addresses the area of leisure is primarily used as an assessment of personal care safety and health and money management. answer c was designed specifically for the OT to assess spirituality. answer D measures the client's level of satisfaction and leisure activities in six categories.

When preparing a home program with the goal of independent toileting for a young child with postural balance difficulties, what is the MOST important adaptation the OT can recommend? a. replacing buttons on cloths with loop closures b. mounting a safety rail next to the toilet c. introducing toilet paper tongs to facilitate independence d. placing a colorful target in the toilet bowl

b. mounting a safety rail next to the toilet Safety rails next to the toilet, low toilets, and reducer rings to decrease the size of a toilet all help provide maximal stability. The other answers describe adaptations for other needs. A is used for children with reduced strength or fine motor coordination. C helps children with limited ROM. and D helps boys aim into a bowl.

What are the MOST important items to assess when evaluating motor control after a traumatic injury? a. developmental factors and primitive reflexes b. muscle tone, postural tone, reflexes and coordination. c. BP, HR, endurance, and confusion d. self-concept and self-awareness

b. muscle tone, postural tone, reflexes, and coordination. After observing functional performance, the OT usually will find it necessary to assess the performance components that underlie motor control. Answer C, evaluation of vital signs is associated with the evaluation of cardiopulmonary function. Answer D, assessment of self-concept and self-awareness is related to the psychological impact of trauma. Answer A is related to an individual's life experiences and how those experiences relate to coping with the musculoskeletal disorder.

The teacher reports to the OT that a 5 y/o child with Down syndrome and low muscle tone sits only in a "W" sitting position for any floor activity. what does this observation MOST likely indicate? a. increased muscle tone in the LEs b. need for this position to achieve stability c. lack of control over LE movement d. a posture that is typically seen in early motor development

b. need for this position to achieve stability This posture is used to maintain stable sitting but is not recommended because it may dislocate the hip and limit trunk strengthening. The child is most likely compensating for postural control limitations that affect dynamic balance. A is incorrect because the child has already been identified as having low muscle tone. C is incorrect as this type of sitting is not a functional strategy to control lower body movement. And it is not an expected pattern in typically developing children.

An OTA working in outpatient rehab teaches herself how to use paraffin by reading books on PAMs, carefully reading the instructions that came with the paraffin bath unit, and practicing on herself for several weeks. Is it now acceptable for this OTA to provide paraffin treatments? a. no, OTAs may not administer PAMs b. no, it violates the AOTAs position paper on modalities c. yes, she demonstrated service competency d. she must have an OT on duty when she uses the PAM

b. no it violates the OTAs position paper on modalities. Despite the level of training she has achieved, according to the position paper, it is still necessary for the OTA to demo service competence to administer PAMS. Having an OT in the facility does not make it acceptable for the OTA to administer the modality if he or she has not demonstrated competence.

what is the BEST method for assessing ADL performance of an individual with severe cognitive limitations and poor communication skills? a. self report b. observation c. ACL screen d. allen diagnostic model

b. observation observation assessments are particularly useful with people with communication difficulties, limited mental capacity, or younger clients. It allows the degree of flexibility that enables the therapist to use judgments and improvisation when moving from theory and hypothesis. answer A would not be appropriate for a person with poor communication skills. answers c and D evaluate cognitive level and assist with treatment planning and intervention, but they do not assess ADL performance.

In planning the eval for an adolescent with psychosocial problems, which is the school-based OT MOST likely to include? a. prevocational and career skills b. observation in the cafeteria and during classroom free play period c. standardized measures of visual motor performance related to school tasks d. community mobility checklist

b. observation in the cafeteria and during classroom free play period. One of the most valuable assessment methods is observation. data form this observation helps identify weaknesses. No evidence to support specific focus on vocation and career skills. D is appropriate for a person at risk for movement limitations.

a toddler diagnosed with DD does not finger feed and turns away when presented with food in the clinic. The BEST next step for the OT is to: a. assume fine motor delays limit finger feeding ability b. observe him in his home during feeding time c. review his chart for food allergies d. repeat the observation in a quiet area to minimize distraction

b. observe him in his home during feeding time considering the context that surrounds a child's performance is critical in occupational therapy assessments. although answers ANC provide useful information for treatment planning, they do not address feeding skills. answer Dee does not put the skill to be assessed into an environmental context.

the OT plans to use a top down approach for evaling a childs self-feeding performance skills. How does the assessment most likely begin? a. assessing seating and positioning needs b. observing the child eating during a meal c. identifying assistive devices d. isolating performance component skills

b. observing the child eating during a meal This is a top down approach that considers the contexts in which the child performs valued occupations. The other answers focus on the clients limitations, which are more consistent with a bottom up approach.

which of the following is the MOST important adaptation to recommend to an individual returning home following a THR? a. move items from high cabinets to lower locations b. obtain a raised toilet seat c. place high contrast tape at the edge of each step d. install a handheld shower head

b. obtain a raised toilet seat the individual will most likely require this to avoid flexion past the designated range. A handheld shower is not always necessary. answer C may make stairs safer for individuals with vision deficits. An individual could not reach into lower cabinets to retrieve items to due hip flexion.

An OT is scheduled to interview an individual with a head injury about the home environment and family and children care responsibilities. Knowing the individual has an attention span of 10-15 minutes, which of the following strategies should the therapist utilize first to obtain the most information? a. schedule a 30 minute treatment session b. obtain as much info as possible from the chart prior to the interview c. interview the individual using appropriate verbal and nonverbal communication skills d. perform the interview in an environment where distractions can be minimized

b. obtain as much information as possible from the chart prior to the interview this individual will most likely have difficulty with the length of the interview. by reviewing the individual's medical record before the interview, the OT can determine what information has already been obtained and Taylor questions accordingly. with such limited attention span, it would be more efficient to schedule two 15-minute sessions rather than one 30-minute session. Good communication skills and providing an environment where distractions are minimized are both important to successful interview. these techniques would be optimally applied after the therapist reviews the chart.

a young adult is hospitalized with anorexia nervosa. The OT wants to collect data on the individuals life history, patterns of daily living, interests, values, and needs. By what means would the OT best obtain this info? a. overal physical assessment b. occupational profile c. activity configuration d. psychoeducational session

b. occupational profile the OT profile is designed to gather information about a clients history, experiences, patterns, interests, and needs. A would be performed to determine strength, endurance, and balance. C would be used to contribute info to the OT profile, and D would be used to provide info about the disease or strategies for managing it.

An OT is leading a discussion with a group of individuals who are diagnosed with major depressive disorder. What is the MOST helpful therapeutic communication approach for the OT to take? a. be upbeat, positive, and cheerful when encouraging the individuals to discuss their feelings b. offer even-tempered acceptance, reflection, and assurance that they will get better c. remain silent and still while the individuals are describing their feelings d. allow the individuals to structure and lead the group discussion

b. offer even-tempered acceptance, reflection, and assurance that they will get better Empathy and validation are important when working with an individual with depression. B is a good example of this type of response. A is perceived as denying the significance of the individuals feelings. C may be perceived as unaccepting. D is incorrect because initiating and maintaining discussion is often difficult for depressed individuals.

An OT working with a group of children who have behavioral challenges is planning activities to develop their play patterns and interaction skills. Which activity will MOST effectively address this aim? a. have the children paint pictures during "free time" b. organize a game of soccer during recess with the group of children c. recommend the teacher include additional game software in the classroom library d. suggest the teacher include age appropriate jigsaw puzzles in the classrooms game center

b. organize a game of soccer during recess with the group of children The soccer fame is a naturally occurring activity during child's recess in which the OT can embed intervention strategies to promote play skills. Answers A and C do not promote interaction among the group of children. Answer D does not mean the children will use them or does not promote social skills if the puzzles are completed independently.

an individual is being discharged from inpatient rehab following a CVA. The individual requires min A in ADLS, is independent in transfers, demos continuing improvement in RUE function, and plans on eventually returning to work. What would the recommendations for continued OT services most likely be for this individual? a. home health b. outpatient c. work hardening program d. discontinuation of services.

b. outpatient this is appropriate for individuals who are medically stable and able to tolerate a few hours of therapy at a clinic. A patient with such high functional levels is not appropriate for home health referral. C are for individuals who are severely deconditioned as a result of injury . Services should not be discontinued if the patient has potential for further recovery.

the hospital based OT is working with a 6 y/o child who has UE weakness and in coordination sustained after a recent head injury. The child wishes to put on and take off her pants independently. What would the OT most likely recommend? a. pants with velcro inserts placed in the zipper area b. pants with an elastic waist band c. cotton pants with large buttons inserted where the zipper is located d. pants with an enlarged zipper pull attachment

b. pants with an elastic waist band These are easier for dressing. The other options typically make undressing easier, but because of this childs incoordination, elastic waistbands would be the preferred method to introduce first.

an OT is using a sensory integration approach with a group of regressed residents on an inpatient unit who display a very low energy level, hyposensitivity to stimuli, and poor visual and tactile perception. What is the BEST activity for beginning a session focusing on identifying pleasant memories? a. go around the circle and ask each patient to introduce themselves b. pass around a scent box and ask each patient to smell the contents c. ask each patient to select a favorite poem and read it d. discuss the lunch menu and healthy eating habits

b. pass around a scent box and ask each patient to smell the contents sensory integration theory believes that individuals can learn by receiving, processing, and responding to sensory stim. Answer A can be confusing and time consuming. Answers C and D are activities more suited to patients functioning at higher levels.

A person with a long history of Parkinson's is experiencing considerable fatigue during the day. The BEST way to enable the individual to maintain his level of function is to teach him how to: a. work through the fatigue b. perform desired activities in a simplified manner to conserve energy c. employ pursed-lips breathing d. eliminate activities or reduce activity level as much as possible

b. perform desired activities in a simplified manner to conserve energy Introduce task simplification to conserve energy. Encouragement to work through fatigue would further deplete energy. Using pursed lip breathing is recommended for individuals with pulmonary diagnoses. Recommendations to decrease activity would be detrimental to maintaining occupational performance.

an individual with severe depression is withdrawn and exhibiting a low energy level. Which of the following interventions would be most effective in supporting participation at this time? a. selecting a leisure activity of interest and identifying materials needed b. performing a simple task c. practicing meditation d. writing suggestions for copying with daily life stresses

b. performing a simple task individuals with severe depression benefit from activities that are concrete and tangible, short term, simple, and success enhancing. The other answers would eventually be relevant but not initially.

an individual recovering from peripheral nerve injury demos weakness in the thumb opposition. Which of the following instruments most effectively evals strength in the affected area? a. aesthesiometer b. pinch gauge c. dynamometer d. volumeter

b. pinch gauge This measures pinch patterns that require strength from thumb opposition. A measures two point discrimination, C measures grip strength but not necessarily thumb strength, and D measures edema.

Which of the following is the BEST instrument to use when assessing 3 point pinch strength? a. aesthesiometer b. pinch gauge c. dynamometer d. volumeter

b. pinch gauge a pinch gauge is used to measure the strength of pinch. answer a measures two point discrimination, answer c measures grip strength, and answer D measures edema in the hand.

an individual who is functioning at ACL 4 has difficulty remembering to take his meds twice daily. Which of the following is the MOST appropriate recommendation? a. instruct the client to take the medication at 9 am and 9 pm b. place a card near the refrigerator showing the blue pill with a picture of breakfast and white pill with a picture of lunch c. teach the caregiver to remind the client to take medication twice daily d. teach the caregiver to place the pills into the clients hands at the dedicated times.

b. place a card near the fridge showing the blue pill with a picture of breakfast and white pill with a picture of lunch individuals at this level can carry out familiar routines and are able to attend to visual cues. Linking medications with meals helps the goal become routine. Answer A is consistent with ACL 5. C is consistent with ACL 3 and D is ACL 2

What type of interactive play would an OT MOST likely recommend to a mother of a 5 y/o child? a. coloring on paper b. playing a simple board game c. looking at picture books d. participating in reading club

b. playing a simple board game A 5 y/o engages in cooperative play and is able to play games with rules and plays well with others and enjoys social interaction. Answers A and C are appropriate for younger children. D is appropriate for an older child who has developed independent reading skills.

an individual with ALS has asked how to maintain strength in the weak (fair +) wrist extensors. Which is the most appropriate intervention for the OT to recommend? a. wearing a cock up wrist support b. playing hook and loop checkers to tolerance c. performing AROM of the wrist daily without resistance d. completing wrist extension exercises several times a day against max resistance.

b. playing hook and loop checkers to tolerance gentle, repetitive, and resistive exercise such as velcro checkers help to maintain strength and endurance in weakened muscles. A compensates for loss of muscle strength but does not help maintain it. C will not help either. D is contraindicated with individuals with ALS.

An OT working with a young child with autism learns from the parent that falling asleep at night is difficult for the child. Which is the FIRST strategy the OT would try to assist the child to fall asleep? a. a mini trampoline to tire the child out before going to bed b. positive bedtime routines that are predictable c. let the child pull al prebedtime activity from a bag containing favorite items d. play with multisensory tactile media before bedtime

b. positive bed time routines that are predictable Children with ASD often respond well to routines in general. A, C, and D are unpredictable sensory input that may increased arousal .

the goal for a patient who has had a TBI is to be able to put on a shirt independently. The most effective way for the OT too structure dressing training for maximum learning retention and generalization of the skill is: a. teaching and practicing each segment of the dressing procedure during consecutive treatment sessions b. practicing the whole task of putting on a shirt in a setting similar to the real environment c. providing dressing simulation activities d. allowing the client to view a videotape of how to put on a shirt and providing written directions for completing the steps of dressing.

b. practicing the whole task of putting on a shirt in a setting similar to the real environment retention of a skill will be enhanced if the task is practiced in its entirety during each performance trial. Whole task performance is easier to recall than separate steps. A is useful for performance but not generalization and retention.

An OT working with an individual following a UE amputation is determining whether a hook terminal device or a functional prosthetic hand would be most appropriate. The individuals primary concern is his ability to return to work and function as a carpenter. What is the MOST important factor in the therapists recommendation? a. cosmetic appearance b. prehensile function c. weight of the terminal device d. ease of cleaning

b. prehensile function A hook TD is lighter and provides better visibility when grasping objects. It is more durable and functional than prosthetic hands. Answers A, C, and D are incorrect because although they can be important considerations, they are not relevant to this persons specific goals.

a manager of an OT department is attempting to increase the visibility of an OT practice within a community setting that provides services for recipients of Medicare. In order to market the program, the OT manager would most likely market via: a. community newsletter b. presentation to local physicians with flyers and business cards c. local community workshop d. presentation to staff at a local hospital

b. presentation to local physicians with flyers and business cards The other answers are all examples of internal and external marketing strategies that may not directly target Medicare recipients.

An OT is fabricating an orthotic for an individual who presents with a low ulnar nerve injury lesion. The orthosis MOST appropriate for this individual is one that includes which components? a. prevents hyperextension of the PIP joints and allows PIP flexion b. prevents hyperextension of the MCP joints and allows MCP flexion. c. allows hyperextension of the MCP joints and prevents MCP flexion d. allows flexion and hyperextension of the MCP joints.

b. prevents hyperextension of the MCP joints and allows MCP flexion. This injury results in loss of most hand intrinsic and the ring and small fingers present as claw deformity which is a position of MP hyperextension and PIP flexion associated with muscle imbalance in ulnar innervated structures. An ulnar nerve orthosis's primary purpose is to support the hand secondary to ulnar intrinsic muscle paralysis or weakness. This orthosis allows for MCP flexion. The other answers are all inappropriate techniques for fabricating an ulnar nerve orthosis.

A resident of a long-term care facility is in a weakened condition and has midstage dementia. OT is requested because the resident requires assistance with dressing. What is the best way to reestablish dressing routines? a. have the resident select the clothing she prefers and then have the OT dress the client b. preview each step of the process as the resident dons and doffs loose-fitting clothing in bed c. have the resident walk to retrieve clothing to encourage independence d. instruct the resident in the use of reacher, sock donner, and button hook.

b. preview each step of the process as the resident dons and doffs loose-fitting clothing in bed. Because of her cognitive deficits, a compensatory strategy would be most appropriate. No new learning will be able to occur at this stage of alzheimers. Dressing in bed requires less energy, and lose fitting clothes are easier to take off. Answer A does not provide enough participation to be therapeutic. C and D are too demanding and do not provide the structure necessary to provide safety and ensure successful performance.

The OT is observing a worker with an ID perform a packaging task requiring assembly of a game box by placing first a pad of paper, then a pencil, then a plastic game piece into a box. The OT realizes the client is having difficulty utilizing the correct assembly sequence. The OT decides that backward chaining would be the most effective technique for training this worker. How can the OT BEST introduce this technique? a. instructing the worker to reverse the packaging sequence, placing the plastic game piece in first. b. prompting the worker to use the correct sequence with each item, then gradually eliminating prompts beginning with elimination of the prompt for the plastic piece. c. having the worker master the first step, putting the pad into the game package, then passing the package on to another worker to insert the pencil and plastic piece. d. demonstrating and repeating the correct sequence before each of the worker's attempts to package all 3 items.

b. prompting the worker to use the correct sequence with each item, then gradually eliminating prompts beginning with elimination of the prompt for the plastic piece. Backwards chaining is used to teach multistep tasks. In this approach, the therapist shows or prompts all steps of the task and on the next trail, all of these steps except for the last one are demonstrated or prompted. This occurs until the whole sequence has been mastered.

Individuals in a mental health clubhouse program are participating in a social skills training group focused on learning skills for meeting someone for the first time. The participants have completed the step in which they performed the role-play as it most recently occurred, without trying to use the most effective social skills. What is the NEXT action the OT should take when using a responsive skills training protocol? a. give a homework assignment focusing on meeting someone for the first time b. provide positive and constructive feedback about performance in the role-play c. discuss how to carry out strategies when meeting someone for the first time d. run the role-play again, this time using effective social skills for the same situation.

b. provide positive and constructive feedback about performance in the role play. After providing feedback , the roleplay would occur again but this time demonstrating more effective social skills. After more feedback, the OT would provide real life homework and closes the session by summarizing the skills covered in the session. Discussion (answer C) would have occurred earlier during the problem-solving sequence.

A 6 y/o received OT for dressing skill development and is now independent. At discharge, what is the BEST advice for the OT to give the child's parents to maintain the child's independence in dressing at home? a. give assistance when the child asks for it to provide a successful experience. b. provide praise for completed dressing; do not help the child get dressed. c. supply oversized clothing with hook and loop closures and large snaps d. use verbal prompts when needed and help with closures only.

b. provide praise for completed dressing and do not help the child get dressed. The social environment, family, and other caregivers provide encouragement and support ADL independence. Since the child has achieved independence, he does not need assistance, clothing adaptions, or verbal prompts to complete the task.

An individual is having difficulty getting around her home as a result of low vision. What is the MOST appropriate strategy the OT can recommend to improve accessibility? a. Instruct the individual to sit while performing ADLs. b. provide strong color contrast at key areas to identify steps, pathways, etc. c. arrange for another person to provide assistance when moving within the home. d. recommend training in white cane use for identifying obstacles in the home

b. provide strong color contrast at key areas to identify steps, pathways, etc. Using contrast is a key environmental adaptation for people with visual impairments. Answers A and C would not directly address accessibility. Answer D, is a mobility technique for someone who is blind.

The OT is working individually with a client who has demoed anger and aggression on several occasions. When the TO observes the client becoming increasingly restless, then yelling, what is the first action the OT should take? a. remove him or herself from physical proximity of the client altogether b. provide the client with the choice to talk about it or end the session c. call for help and escort the client to the ER d. leave the door to the treatment room open

b. provide the client with the choice to talk about it or end the session Restoring control is an effective means of compensating for feelings of loss and intervening in the client anger. Providing the client with a choice is one way of restoring a sense of control to the individual. Answer D is something the therapist should have done prior to the session beginning. A and C might be necessary if attempts to deescalate the clients behavior are not effective

The MOST effective method of compensation for both unilateral neglect and absence of sensation in an UE with good motor control is to: a. avoid the use of sharp tools or scissors and avoid extreme water temps b. provide a warning tone such as a noisy bracelet on the wrist as a reminder to visually scan toward the affected side c. use an electric shaver d. wear elbow pads on the affected side

b. providing a warning tone such as a noisy bracelet on the wrist as a reminder to visually scan toward the affected side. using a cue is a compensatory technique. It is the best to teach visual scanning and is a technique that can be used anywhere. D is an intervention that is only feasible in a persons home. Answers A and C does not teach him or her to attend to the affected side visually because it mainly addresses sensation and safety.

During the clean-up portion of a cooking activity, an elderly woman with a diagnosis of depression and dementia begins to dry the plates and utensils she has already dried. What should the OT do NEXT? a. tell the client that the same dishes and utensils are being redried. b. put the dried dishes away and begin to hand her wet dishes c. suggest the client stop the activity because it seems too difficult d. ask the client to describe what she is doing

b. put the dried dishes away and begin to hand her wet dishes. The OT must focus on upholding the dignity of the person during each stage when working with this diagnosis. This answer maximizes the individuals engagement in the process. Compensating for mistakes helps to increase sense of self-worth. D can undermine the clients self confidence. All other answers draw attention to the individuals errors.

a child with CP tends to flex forward while riding her adapted tricycle, even though her LEs are correctly positioned. Which adaptations should the OT recommend to BEST enable the child to maintain an upright position while riding? a. raise the seat height b. raise the handlebars c. lower the seat height d. lower the handlebars

b. raise the handlebars Raising the handlebars demands that the arms are raised, thus bringing the child into an upright position. A and C are incorrect because the hips and LEs are already positioned correctly. D is incorrect because the arms would be lowered increasing trunk flexion

An OT is performing a home eval for an individual with paraplegia who uses a standard manual wheelchair and notes that the entrance to the bathroom is 32 inches wide and the toilet is 15 inches high. Which of the following recommendations will best facilitate the use of the bathroom for this individual? a. widen the doorway b. raise the toilet c. widen the doorway and raise the toilet d. widen the doorway and lower the toilet

b. raise the toilet The minimum doorway that allows the width of a standard wheelchair is 32 inches. thus, the door does not need to be widened. A standard toilet is 15 inches, but a standard wheelchair is 18 inches. Therefore, raising the toilet would be more beneficial for completion of safe transfers.

An individual diagnosed with borderline personality disorder tells an OT that she is the only one she can trust. The next day, she accuses the therapist of lying to her. What is the BEST way for the OT to respond? a. tell the individual her feelings have been hurt b. remain matter of fact and consistent in approach c. ask the individual how she felt when lied to in the past d. apologize and try to determine how the misunderstanding occurred.

b. remain matter of fact and consistent in approach Individuals with this diagnosis demo inconsistent behavior and have difficulty managing consistent relationships. It is important that the OT does not take this behavior personally. In addition delving into exploration of the individuals past is not recommended.

A college student with neurological deficits is planning to return to independent living following discharge from therapy. The student has been working on shoelace tying, but is unable to carryover skills learned from one day to the next day. What is the most appropriate strategy to try next. a. practice the same method more frequently b. replace regular shoelaces with elastic laces c. move therapy sessions to a distraction free environment d. instruct caregivers to provide assistance for shoelace tying

b. replace the regular shoelaces with elastic shoelaces a patient who exhibits no capacity for new learning will be unable to benefit from therapy that requires the transfer of learning. Therefore the other answers would not be appropriate. A distraction free environment would be useful for individuals with attention deficits.

An OT is fabricating a static orthotic device that will assist with the maintenance of a functional hand and finger position while keeping the soft tissues of the hand in a midrange position. Which orthotic device would the OT most likely select to address these needs? a. bivalve b. resting c. dynamic extension d. wrist cock-up

b. resting a resting orthosis is the most appropriate to fabricate for the maintenance of a functional hand position. A bivalve cast is usually used when circumferential pressure of a body part is required to maintain a desired position. Answer C is not considered a static orthosis device. Answer D does not impact the position of the entire hand because it ends at the MCP crease.

A 7 y/o with CP demos fair sitting stability and good head control with fluctuating LE extensor tone. What position device would the OT MOST likely use during feeding? a. prone stander with lateral trunk supports b. rifton child's chair with footrest and padded abductor post c. caregivers lap with one arm stabilizing the trunk d. beanbag chair for full sitting support.

b. rifton child's chair with footrest and padded abductor post stability in the trunk and support of midline orientation with the head and neck aligned in neutral or slight flexion is the proper guidelines for optimal oral motor control. Answer A would place the child in standing or extended position which would reinforce the undesirable extensor tone. Answer C is not appropriate for eating if other positions are available. The child would not benefit from a beanbag chair since it does not successfully inhibit extensor posture.

the OT is working with a young woman with motor control deficits following a TBI and feels the client would benefit from deep proprioceptive input. Which craft activity would best provide the needed type of input? a. mixing paper mache dough by hand b. rolling clay into a 1/4 inch slab using a rolling pin c. gluing mosaic tiles onto a premade form d. stringing glass and silver beads onto a leather cord

b. rolling clay into a 1/4 inch slab using a rolling pin treatment using motor behavior models attempts to make an impact on the neurological system through positioning, normal movement patterns, and tactile and proprioceptive cues. Crafts that involve weight bearing are often considered most likely to achieve normal neural reactions. A is good for sensory processing. C and D do not provide much proprioceptive input

an individual with a below elbow amputation lacks sensation in the residual limb. what is the most appropriate intervention for the OT to teach the patient? a. techniques of rubbing, tapping, and application of textures to the residual limb. b. routinely inspect the skin closely for any signs of skin breakdown c. deeply massage the residual limb d. perform proper skin hygiene

b. routinely impact the skin closely for any signs of skin breakdown teaching patients with a residual limb to compensate for lack of sensation is essential to prevent injury. A is incorrect because those techniques are used when a limb is hypersensitive. Answer C, deep massage, is used to loosen and prevent scar adhesions. D is important but not, by itself, prevent skin breakdown

an OT is assisting an individual with mild hemiparesis in transfering from the wheelchair to a mat table using a stand pivot transfer technique. After locking the brakes, what is the first verbal cue the OT gives the individual? a. stand up b. scoot forward to the edge of the wheelchair seat c. unlock the wheelchair breaks d. position the wheelchair so that it directly faces the mat table

b. scoot forward to the edge of the wheelchair seat after locking the brakes, the OT encourages the client to scoot forward to put his or her feet flat on the floor. At this point in the scenario, the wheelchair would already be properly positioned and it would be unsafe to stand if the wheelchair was not locked.

A child demos aggressive and disruptive behavior in school as a result of a low sensory threshold. Which suggestions would be MOST useful to discuss with the teacher regarding an upcoming class bus trip to the zoo? a. review the bus rules with the child and apply consequences consistently b. seat the child at the front of the bus and use earmuffs to dampen noise. c. have the child monitor classmates as "bus patrol" and report behavior d. let the child set the criteria for a successful trip and provide a reward if criteria is met.

b. seat the child at the front of the bus and use earmuffs to dampen noise This answer reduces sensory stimulation present of the bus ride. A child with low sensory threshold over responds and is overwhelmed by ordinary sensory input. When seated at the front of the bus, the child will experience less jostling resulting in less tactile and visual stimulation. The other answers are behavioral management techniques.

an individual has been referred to OT following open heart surgery and a period of prolonged bedrest. After the individual is able to tolerate sitting unsupported at EOB, and according to MET levels, the next activity the OT should introduce is: a. knitting while seated b. seated sponge bathing c. engaging in sexual intercourse d. taking a hot shower

b. seated sponge bathing MET value for sitting EOB is 1-2. Seated sponge bathing is 2-3. MET values for knitting is similar to what the patient is already doing, so to upgrade the METS the patient must do a more energy consuming task. C and D range between 4-6 METS.

an individual with parkinsons is at risk for aspiration. When instructing the primary caregiver in proper positioning during feeding, the OT practitioner should recommend: a. feeding the individual in bed in a supine position b. seating the individual upright on a firm surface with the chin slightly tucked c. positioning the individual in a semireclined position in a recliner chair d. providing food to the individual in bed in a side-lying position

b. seating the individual upright on a firm surface with the chin slightly tucked the best position for feeding an individual with a swallowing disorder is upright and symmetrical with the chin slightly tucked. The other answers increase the risk of choking.

The OT has been working with an individual with deficits in the area of executive functioning postsurgery. When the OT asks about scheduling a therapy appointment after discharge and transportation to the outpatient clinic, the client is unable to identify how to obtain the clinic's phone number or where to park. When the OT points out these missing areas, the client appears perplexed, then laughs it off, stating that someone else would be calling to schedule appointments and providing transportation. Attempting to subtly hint about cognitive deficits is no longer working, and the OT is concerned because despite repeated efforts, the individual is not retaining safety precautions. This patient's behavior indicates a problem in what area? a. denial b. self-awareness c. emotional regulation d. sequencing

b. self-awareness Problems that contribute to deficits and self awareness include recognizing errors, inability to use feedback, and false beliefs about abilities. Whereas an individual who lacks self-awareness may be surprised or confused when given feedback regarding limitations an individual who is in denial may demonstrate anger or resistance when given feedback. Deficits in emotional regulation may be characterized by uncontrolled anger, laughter, or crying. Deficits in sequencing may be characterized by difficulty planning or enacting the steps of an activity.

An OT is working with an individual with schizophrenia who is in the process of preparing to move from a state hospital to a group home. During a baking group, the client becomes agitated and leaves the room when another client uses the electric hand mixer to mix the cake batter, and again when two clients begin to argue loudly about which type of icing to use. How would the OT BEST describe this behavior? a. low registration b. sensory avoiding c. sensory seeking d. a hearing impairment

b. sensory avoiding The individual's actions are indicative of sensory avoiding behavior, characterized by low threshold to stimuli perceived as noxious, followed by an active response such as leaving the room. Individuals with sensory avoiding behavior may become distressed in situations in which they cannot control the environment. An individual with low registration, sensory seeking behavior, or a hearing impairment would not have difficulty with the auditory stimulation.

An individual with severe and persistent mental illness and no work experience has indicated a strong interest in paid employment. Following eval, the OT determines the client is not ready for mainstream employment. What is the NEXT step in progressing the client toward the goal of mainstream employment? a. transitional employment b. sheltered employment c. income requirements determination d. job coach identification

b. sheltered employment These settings offer work experience in a segregated environment for individuals with developmental and psychiatric disabilities. This individual would benefit from a sheltered employment setting first, in which intensive and continuing supervision is available, in order to gain work experience before moving to an in house job (AKA transitional employment). Income requirements would have little bearing until the individual is qualified for mainstream employment. D may benefit the client in various work environments, but identifying a job coach would occur after placement in a vocational environment.

during eval, the OT responds using paraphrasing to an individual who recently lost a spouse in a car accident. Why is the OT most likely implementing this technique? a. refocus or redirect the individuals comments b. show acceptance and understanding regarding the individuals situation c. persuade the individual to make a choice d. encourage the individual to provide additional info

b. show acceptance and understanding regarding the individuals situation active listening is a technique that helps to establish the therapuetic relationship. It involves paraphrasing the speakers words rather than reacting to them. Redirection, answer A, is used to promote healthier thoughts and behaviors. No need for a choice is indicated in this scenario. D is encouraged when an individual asks open ended questions.

the OT is instructing an individual with COPD in EC techniques. Which of the following should the OT recommend in order to limit the amount of energy expended during bathing? a. using a reacher to retrieve and place items b. showering on a tub bench while incorporating rest breaks c. using a bath mit while bathing d. encouraging use of proper body mechanics.

b. showering on a tub bench while incorporating rest breaks When patients are taught EC techniques, rest breaks and seated activities should be incorporated. A and C would be most likely used by individuals with limited reach and grip strength. D is important for individuals with back injuries.

the OT is working to develop play behavior in a child with low muscle tone who has difficulty engaging in activities against gravity. What is most likely position for the OT to use with this child? a. long sitting along a wall b. side lying on a mat c. supine on a large wedge d. prone over a bolster

b. side lying on a mat in this position the child has an excellent opportunity to try out skilled arm and hand movements on the upside arm. This position demands less trunk control than sitting. There are no weight bearing demands on the shoulder of the arm that is not reaching, nor does the infant have to work against gravity. A and D require the child to work against gravity and would likely be too difficult.

A client recently underwent bilateral BKAs. The client presents with good UE strength and sitting balance. In which of the following transfer techniques would the OT most likely instruct the client FIRST? a. stand pivot b. slide or transfer board c. dependent d. sit to stand

b. slide or transfer board this technique is appropriate for clients with paraplegia of the Les, bilateral amputations, severe LD weakness, or inability to bear weight through LEs. Answer A and D would be appropriate once pylon training begins where as C would not be indicated for a client with good UE strength and balance.

An OT is working with a 6 y/o child who occasionally drops his utensils when eating because of a slight decrease in hand ROM/grasp limitation. Which piece of equipment would the OT most likely recommend FIRST? a. swivel utensil b. pediatric universal holder c. foam tubing around the utensil d. weighted utensil

b. slightly flexed A chin tuck position may be recommended when the child has delayed swallow initiation. Positioning with head in extension can increase risk of choking. Rotating the head does not facilitate swallowing.

A roofing contractor was diagnosed with extrapyrimidal disorder Parkinsons. When the OT is educating the client about hypokinesia, what symptoms would be described? a. hyperextension or hyperflexion of the wrist and digits b. slow movements c. arrhythmic or wormlike movements of the distal extremities d. purposeless jerky movements

b. slow movements individuals experiencing extrapyrimidal disorders are characterized by hypo or hyperkinesia. PD is characterized by hypokinesia, cogwheel and lead pipe rigidity, pill rolling tremor and decrease in postural mechanisms. A is associated with dystonia. C describes athetoid movements that affect distal extremities. D is chorea more typically associated with huntingtons disease

an OT observes another OT exit a patients room laughing loudly, clearly intoxicated, with alcohol on her breath. After having discussed his concerns privately with the OT and witnessing this for the third time, the OT believes the individual is a danger to patients in the facility and should be prevented from practicing. In order to achieve that outcome, to whom should the OT report the colleague? a. NBCOT b. state licensure board c. AOTA ethics commission d. facility administrator

b. state licensure board This institution safeguards and promotes public welfare by ensuring that qualifications and standards for professional practice are properly evaluated, applied, and enforced. They are the only group that has the power to prevent an individual from practice such as by revoking the license.

the OT is working with a client who takes cardiac meds because of abnormal cardiac rhythms and chest pain. Which of the following meds is typically utilized to treat these symptoms. a. diuretics b. statins c. beta blockers d. anti platelet agents.

b. statins C are common cardiac medications with the purpose and use to decrease HR and cardiac output, lower BP and make the heart beat more slowly. A are used to lower blood pressure and reduce edema. B is used to lower LDL and HDL. D is to prevent stroke or heart attack from build up of plaque.

An OT working in an outpatient setting has completed ROM measurements on an individual who had hand surgery. After bandaging the open wounds, what should the OT do with the stainless steel goniometer? a. place it in a plastic bag and label it with the individuals name b. sterilize it before using it again c. store it with the used equipment and sterilize it at end of day d. wash it with hot soappy water before using it again

b. sterilize it before using it again Work areas should always be sterilized immediately after use to prevent the spread of infection.

An individual with CRPS has been referred to OT in order to return to work as an administrative assistant in an educational environment. The client has expressed that returning to work is the primary goal, but has concerns about being able to perform all of the required tasks due to pain. using a MOHO approach, on what should OT intervention focus? a. job retraining for a more appropriate position b. strategies to promote resumption of the worker role c. voice activated software and environmental adaptations d. communication and assertiveness training.

b. strategies to promote the resumption of the worker role MOHO recognizes how the relationship between an individuals performance abilities, habits, motivations, and interests impact his or her occupational performance. A and D are both strategies that could be used to support resumption of role performance. A could be necessary but is not indicated based on the given information.

a young child has been wearing a LUE prosthesis for 3 weeks. What is the most important activity recommendation for the OT to give the childs preschool teacher? a. offer toys that they child can manipulate with the one hand so play is successful b. stress bilateral arm/hand use incorporating both arms to hold and stabilize objects c. help the child play by stabilizing and holding toys fo him as he plays d. involve the child in activities that do not require manipulation

b. stress bilateral arm/hand use incorporating both arms to hold and stabilize objects A person with a unilateral amputation can be expected to use the prosthesis primarily for sustained holding or stabilization and use it more slowly compared to the unaffected extremity. Two handed activities for play incorporate the prosthesis into the childs body image and helps develop bilateral skills. The other answers do not help the child integrate the prosthesis into normal patterns of use.

during a meal prep eval, an individual with a brain injury repeatedly cuts up the veggies before washing them and turns on the stove before filling up the tea kettle with water. Which of the following activities MOST appropriately addresses the deficit area demonstrated? a. using rubber stamps in a random design to create notecards b. stringing beads for a necklace, following a pattern c. putting together a 100 piece puzzle and then framing it. d. setting a table with dishes and utensils for four people

b. stringing beads for a necklace following a pattern The individual should have washed the veggies prior to cutting them and filled the kettle before turning on the stove indicating deficits with sequencing. This activity requires sequencing. Answer A utilizes a random approach. Answer C is a good activity for problem solving but does not target sequencing. Answer S is good for developing organizational skills.

To develop the MOST relevant goals for a student's school-based OT program, what should the OT focus on? a. teachers perspective about why the student struggles in the classroom b. students ability to access and participate in the curriculum c. family's priorities for their child d. areas of delay identified in the OT eval.

b. students ability to access and participate in the curriculum Promoting students participation in activities and routines within the curriculum is priority for OTs in the education system. OTs must develop measurable annual goals designed to enable the student to have access to and make progress in the general education curriculum. One persons perspective is not intended to drive the process, so answers A and C are incorrect. Answer D is incorrect as the difficulties established in the OT evaluation are not the only basis of developing the IEP goals.

The OT has provided services to an individual with Medicare benefits on an outpatient basis of 10 times. The individual has achieved his initial self-care goals, and the OT expects him to progress further in the area of self-care performance. The claim for the initial session included a self-care G code and modifier, but no claims have included any G-code and modifier since then. According to Medicare guidelines, what does the OT now need to do? a. submit a claim with the final status G code/modifier and discharge the patient b. submit a claim including the new g-code/modifier for current status and g-code/modifier for new goals c. identify a new, different g-code set, such as mobility or carrying/moving/handling and establish related goals d. request an extension or exemption to continue to provide services

b. submit a claim including a new g-code/modifier for current status and g-code/modifier for the new goals Medicare requires providers to submit a claim at least once every 10 treatment days. If the therapist is not ready to discharge the patient, he or she would not submit a final discharge status. It would only be appropriate to identify a different g code set if the individuals self care goals had ALL been achieved and the patient would benefit from addressing a different deficit. Medicare does not exempt or extend services.

An individual with ID is living in a supervised community environment. Upon eval, the OT determines that the individual can perform work tasks involving repetitive processes and demos good social interaction skills. The family and client are most interested in an integrated work environment. What is the MOST appropriate service delivery model for the OT to recommend a. an adult activity center b. supported employment c. volunteer work d. sheltered workshop

b. supported employment supported employment refers to competitive work in an integrated work environment consistent with the strengths, resources, priorities, abilities, interests, and informed choice of the individual. This model provides a structure for persons with ID to work in an actual job site while also offering assistance from a job coach and would be more appropriate for an individual living in the community. Answer A focuses on leisure rather than employment. C could be a useful form of activity but would not address employment needs. Sheltered workshops are designed to help individuals master basic work skills but have been criticized for creating dependency.

A man with arthritis in his hands is attending outpatient OT to learn joint protection techniques. He wants to continue his hobby of needlepoint, which he does everyday during his train commute to and from work. What advise should the OT give him regarding his hobby? a. provide him with needlepoint designs that have a low level of complexity b. teach him to take breaks frequently when doing needlepoint and respect pain c. encourage him to take up a different hobby that does not require him to hold small items, such as a needle d. recommend he do only small needlepoint projects that can be completed in less than an hour.

b. teach him to take breaks frequently when doing needlepoint and to respect pain. Individuals with arthritis should minimize long periods of static holding. Designs with lower level of complexity could benefit an individual with cognitive deficits. A different hobby might be better for his joints in theory, but working with him to find ways that he can continue the hobby that he values is more client-centered. Because needlepoint can easily be stopped at any time, it is not necessary to limit him to short term projects.

Which adapted technique would the OT MOST likely select in order to teach independent dressing to an 8 y/o girl with hemiparesis? a. encourage the child to do most of her dressing while laying in bed b. teach the child to dress her hemiparetic extremities first c. educate the childs mother regarding how to assist with the dressing skills d. instruct the child to dress her nonhemiparetic extremities first

b. teach the child to dress her hemiparetic extremities first. Answer A is also considered an adapted technique, but would be most appropriate for a child with incoordination or poor balance. While answer C is something the OTA would do at some point in the process, it does not encourage independent dressing skills. D is not considered an adaptive skill because this technique will typically interfere with independent dressing.

a 7 y/o boy with hemiplegia has difficulty putting on his socks each morning before school. What should the OT recommend? a. encourage the child to wear tight-fitting socks b. teach the child to sit in a chair with a back support, lift the affected leg, and use socks with a wide opening c. encourage the child to request assistance from his mother when putting on socks d. teach the child to sit in a chair and lift and place the unaffected foot up on a small stool, and use socks with a wide opening

b. teach the child to sit in a chair with a back support, lift the affected leg, and use socks with a wide opening back support is the best starting position for this task. A wide sock opening can prevent frustration during the most difficult part of donning socks. C and D would not encourage independence. A would make the task more difficult

during the process of planning an intervention program, the therapist relies on data gathered through narrative reasoning. What info do these data provide to the OT? a. students knowledge of the classroom activities and routines b. teachers report of how the student has adjusted in this new classroom c. estimate of the students fine motor skills compared with his peers d. history of therapy services the student has received

b. teachers report of how the student has adjusted in this new classroom B is correct as narrative reasoning is a strategy for understanding the meaning of the experience from the child, family, or other care providers perspectives. A does not explain the meaning of these events. C and D represent factual info.

An OT is reviewing bathing skills with a 13 y/o boy when the teen asks questions regarding his sexuality. What should the OT do FIRST? a. refer the teen to someone who has increased knowledge regarding the subject of sexuality, such as a psychologist b. tell the client that you don't mid discussing sexuality related questions as long as his parents are comfortable with the idea c. attempt to change the subject because the child is too young to be educated on this topic. d. try openly and honestly address the client's questions to the best of your knowledge

b. tell the client that you don't mid discussing sexuality related questions as long as his parents are comfortable with the idea When sexuality questions arise and the child is under 18 years of age, therapists must obtain parental permission. Answer A would be an appropriate response if the parents agreed to the idea and if the OT did not feel comfortable or knowledgeable regarding the topic. C may cause the child to feel inadequate, frustrated, or more confused. D would only be appropriate after seeking permission from the parents.

an OT has been working with an individual who sustained a deep laceration of the median and ulnar nerves, resulting in complete sensory loss. Upon reeval, what sensation would be the first to return? a. vibration and pain b. temp and pain c. light touch and proprioception d. tactile localization and proprioception

b. temp and pain the sensations pain and temp are carried along small unmyelinated nerve fibers which recover more rapidly.

which best represents a school based consultative relationship between the OT and school teacher? a. the teacher identifies a childs needs so the OT can follow up with appropriate intervention b. the OT provides an in-service program to the teachers in the school c. OT reports students progress to other team members at the IEP. d. OT implements services that support the teachers classroom goals

b. the OT provides an in-service program to the teachers in the school the purpose of consultation is to identify and solve problems with engagement in occupations, prevent future problems, or achieve identified goals. The other answers do not reflect the mutual relationships and ongoing collaborative process that is a characteristic of the consultation approach.

An OT and OTA work in a collaborative relationship. The teamwork between the two professionals is best exemplified by which of the following? a. the OT completes the assessment and instructs the OTA to provide a specific intervention b. the OTA updates the OT on progress a patient has made in the past week, and both provide info to update the goals c. the OTA gives a progress note to the OT and the OT writes the discharge summary based off the progress note d. the OT tells the OTA what type of equipment to order for a patient and the OTA orders it from a medical equipment company

b. the OTA updates the OT on progress a patient has made in the past week, and both provide info to update the goals this description best represents a collaborative approach. A and D demo one-way communication. C the OT takes info from the note but does not receive input or recommendations directly from the source.

The OT has just observed a child eating lunch in the school lunch room. Which statement best describes an objective observation? a. the child did not appear to like the food presented b. the child demonstrated tongue thrust with food presented in 5 of 10 trials c. the child was uncooperative and unhappy d. the child was apparently not hungry at the time

b. the child demod tongue thrust with food presented in 5 of 10 trials this is an objective measurement. A infers the childs emotional reaction. C implies or judges the childs behavior and D interprets data based on insufficient evidence.

A local business has hired an OT to assist with needs related to the Americans With Disabilities Act to determine reasonable accommodations and whether an employee is considered as a qualified individual with a disability. Which of the following BEST represents an individual who would qualify? a. the individual has a broken leg from a car accident b. the individual has a physical or mental impairment that substantially limits major life activities and has a history of having had such impairment. c. the individual has a history of hip or knee replacement d. the individual has reported to be a user of illegal drugs and alcohol, which interfere with the ability to perform worker roles.

b. the individual has a physical or mental impairment that substantially limits major life activities and has a history of having had such impairment. The individual must meet one of these requirements in order to be considered IWD. If he or she does not, the employer does not have to accommodate. The other answers are items that disqualify an individual from ADA coverage.

using a top down approach for a middle school student, where should the school based OT process begin? a. school related performance skills b. the students participation in curricular and extracurricular activities c. client factors related to class participation d. an understanding of the head-toe and proximal-distal pattern of development

b. the students participation in curricular and extracurricular activities this is correct as practice models and theories that begin with the consideration of occupational performance lend themselves readily to a top down approach. Answers A and C represent a bottom-up approach and D refers to a traditional motor development theory.

When planning acute treatment for a patient who has recently experienced a traumatic amputation of the RUE at the below-elbow level, which of the following areas of patient education would the OT address first? a. teaching to put on/take off a prosthesis. b. training in residual limb wrapping c. practicing grasp and prehension functions. d. simulation to resume vocational activities

b. training in residual limb wrapping Shrinking and shaping the limb are necessary to allow the patient to tolerate a snug fitting socket. Activities A and C are premature at this time and will be performed after the prosthesis has been selected, prescribed, and fitted. Training to resume vocational activities would normally occur later, once the individual has mastered the basics of prosthetic use.

The IEP team is developing long-range planning for a student with moderate ID. Given the students learning capacities, what will MOST likely be learned? a. perform school skills at a middle school level b. understand basic concepts related to money c. perform mathematic operations that use simple division d. write an accurate short story about his summer vacation.

b. understand basic concepts related to money The other answers represent academic skills beyond a second-grade level and are most likely expected of a child with mild ID.

An OT has just been hired by an organization with locations in 2 states and is about to begin seeing patients in both states. When working with patients covered by Medicaid, what must the therapist be sure to do? a. obtain preauthorization b. understand how Medicaid coverage varies from state to state c. perform and document monthly reeval d. apply to the federal government for reimbursement

b. understand how Medicaid coverage varies from state to state. Medicaide is a cooperative venture funded jointly by the federal and state governments to assist states in furnishing medical assistance to needy individuals. Many healthcare plans use preauthorization as a cost containment measure. C is determined by the patients third party payer but not medicaid. Billing would be directed to the state, not federal, government.

An OT is working with an individual who has tested positive for HIV. The individual accidentally cuts his finger with a knife while helping with a meal preparation task. Which of the following are the MOST appropriate precautions to follow? a. suicidal b. universal c. escape from unit d. medical

b. universal health care personnel should follow universal precautions when blood or bodily fluids are present regardless of diagnosis. The other precautions are guidelines developed for individuals with risks not identified in this question.

which approach should the OT implement to best support learning and development for a child with autism? a. incorporate customized social stories as part of daily recess to promote the childs social skills development b. use a family-centered approach to learn about parents concerns and priorities and provide interventions that help them increase parenting skills and promote their childs learning c. help the family adapt the home environment to include strategies that support the childs individual sensory preferences d. emphasize therapist/child intervention sessions that provide ample opportunities to explore multisensory play

b. use a family-centered approach to learn about parents concerns and priorities and provide interventions that help them increase parenting skills and promote their childs learning Contemporary models of practice recognize the importance of the childs family as a consumer of pediatric OT. It is critical for parents to be informed about their children.

An individual confides to the OT that he is concerned that LE flaccidity may cause problems during sexual activity. After evaluating the client's tone, the BEST strategy to recommend is to: a. use a side-lying position b. use pillows to prop up body parts into the desired position c. incorporate slow rocking into movements d. avoid movements that elicit a quick stretch

b. use pillows to prop up body parts into the desired position. An individual with low tone may benefit from supportive positioning devices. Slow rocking and avoidance of quick stretch are both methods for reducing tone and would not be beneficial to someone with low tone. Side lying would be very difficult to maintain but may be preferable for someone requiring energy conservation or spasticity.

A child with CP and limited postural stability is developmentally ready for toileting. Which element of the treatment plan should be considered FIRST. a. training in management of fasteners b. utilization of foot supports c. provision of a seat belt d. training in climbing onto the toilet.

b. utilization of foot supports. This is the practitioner's first concern so that the child feels secure and when feet rest firmly on the floor, the abdominal muscles that aid in defecation effectively fulfill their function. A is incorrect because management of fasteners requires hand skills that depend on stable posture and balance. C is incorrect because provision of a seat belt may not be necessary. D is incorrect because climbing onto a toilet independently may be developed later.

a child has mastered brushing her teeth while the OT provides verbal and physical cues. TO progress with the process of reducing the intrusiveness of the cues, what is the OTs next step? a. verbal cues only b. verbal and gestural cues only c. physical cues only d. gestures and physical cues

b. verbal and gestural cues only Physical cues are the most intrusive and verbal cues only is the least intrusive. Answer B is a happy medium that downgrade the amount of cues currently being utilized.

when planning an intervention, what is an important consideration to help the OT establish a positive, therapeutic relationship with a group of middle school youth in an inner city after school program? a. contracts to establish expectations written in age appropriate language b. verbal and nonverbal language and physical contact with the students c. culturally sensitive rewards and reinforcements for student who attend groups d. inclusion of older students in the group to serve as role models for the middle schoolers

b. verbal and nonverbal language and physical contact with the students This focuses on the therapists behavior and concerns the therapists presence to help facilitate change. The other answers are potential considerations in the intervention plan but they do not influence or develop interaction between the group and the OT .

An OT is using a visual perceptual FOR to plan an intervention for a child with visual perceptual problems. the FIRST activities planned should address which type of visual perceptual skills? a. visual memory skills b. visual attention skills c. general visual discrimination skills d. specific visual discrimination skills

b. visual attention skills First, the child needs to have good attention to the task, which allows the visual info to be stored in memory. Answer A is incorrect because visual memory skills can only be developed after visual attention is established. Answers C and D are incorrect because they develop after visual memory.

a child with CP and visual impairment is learning to use a computer for the classroom work and is frustrated because it takes a long time for her to type a sentence. What is the best solution to this problem? a. a larger monitor b. voice recognition c. work prediction software d. masking some of the keys on the keyboard

b. voice recognition word processing software A is incorrect because the visual impairments are secondary to the motor impairments in this scenario. C anticipates the word desired and increases speed of input. This may help her complete the task, but not provide an optimal solution. D can limit the variety of words the student is able to see.

An OT working for an assisted living corporation needs to design programs to engage residents with dementia who wander and pace the halls throughout the day. What is the BEST type of movement-oriented programming to employ to engage these residents? a. reminiscing about previous jobs b. walking as a part of walking club c. singing oldies in a group d. participating in a craft activity requiring concentration

b. walking as part of a walking club walking can provide an outlet for the movement needs of some people with dementia. Walking in a structured way can be camping, provide an activity of exploration, and help refocus the resident. The other answers are good activities but would not provide the element of movement in the same degree.

a patient who had a CVA has difficulty using his LUE for reaching activities because of fluctuating muscle tone. According to NDT approach, one of the MOST effective ways to teach a person to normalize high tone in affected extremities prior to functional activities is by: a. placing a weighted cuff on the extremity b. weight bearing through the UE in sitting or standing c. using the unaffected arm for all reaching activities d. forced use of the affected extremity

b. weight bearing through the UE in sitting or standing weight bearing is the most effective way of normalizing tone according to the NDT approach. A would increase muscle tone making it more difficult. C would accomplish the reaching task but not normalize the tone. D is a concept not specific to NDT and not designed to normalize muscle tone.

A hospital based OT is working on discharge plans for a 2 y/o with paraplegic spinabifida who has just started using a power wheelchair. Which community resource recommendation is the MOST critical for this child? a. a social service agency b. wheelchair equipment vendor c. family physician d. early intervention program

b. wheelchair equipment vendor Although any of these community resources may be helpful, B is correct because of the possible breakdown of the piece of equipment. The other answers may serve as resources for other needs of the child, but only a specialist in wheelchair equipment would be able to solve any mechanical issues that arise.

a 2nd grade student with MD operates a manual wheelchair, but his mobility is slow because of muscle weakness. When should the OT consider a power wheelchair? a. in junior high school when child switches classrooms several times a day b. when the childs speed over long distances becomes less than that of a walking person c. after the home can be made accessible d. after it becomes impossible to propel a manual wheelchair

b. when the childs speed over long distances becomes less than that of a walking person Because the child will be experiencing progressive muscle weakness, EC is of primary importance. A and C address valid environmental considerations to be made after determining the general need for a powered wheelchair. Waiting until it is too late would be difficult.

a child has difficulty sitting due to inadequate postural reactions. What is the first activity the OT would use to promote the development of independent sitting? a. swinging on a playground swing with a bucket seat b. wide base sitting on the floor while reaching for a suspended balloon c. straddling a bolster while batting a ball d. bouncing on a ball with a handle using only one hand for support

b. wide-based sitting on the floor while reaching for a suspended balloon The child first practices skills in unsupported sitting on a stable surface using a wide base of support. As skills improve, the wide base is reduced to a more narrow one. Reaching activities are used to promote postural reactions. The other answers are all involving unstable support.

An OT learns that a nursing home resident fell during the night and may have sustained a new fracture. The individual still wishes to engage in therapy. What is the BEST course of action for the OT? a. provide treatment as originally planned based on the wishes of the individual but stop if increased pain is elicited b. withhold treatment, but gather info on the course of events for documentation and consultation with the treatment team c. provide treatment by observing self-feeding, an area not listed as an OT treatment goal d. withhold treatment and wait for the OT director to return from vacation to gather information and plan treatment

b. withhold treatment, but gather info on the course of events for documentation and consultation with the treatment team. Treatment plan may need to be revised as a result of change in status and treatments cannot be conducted unless used to achieve a goal. Therefore, answer C is incorrect. Waiting for the director to return delays services.

An individual is participating in a work hardening program after an accident resulting in physical limitations. The individuals goal is to return to his job as an auto mechanic. Which of the following BEST represents a work-simulation activity? a. lifting weights b. working on a mock car engine c. visiting the work site garage d. preparing lunch for work in a standing position

b. working on a mock car engine Work hardening includes all aspects required for the client to return to full function in employment, such as psychosocial, communication, physical, and vocational needs, and typically incorporates work simulation tasks as part of the treatment process. This activity would address endurance, productivity, and strength. Answer A is not a work hardening goal when performed in isolation from a simulated task. C would be part of the onsite analysis typically completed by the practitioner or vocational retraining counselor. D is not considered to be a demand required by this particular vocation.

An individual who lives in a group home and has a history of sexual abuse has identified relaxation through physical activity as a method she would like to use to manage anxiety. Which activity would be most appropriate for the OT to introduce? a. sewing and handcrafts b. yoga, initially in her own room c. progressive relaxation exercises in a group format d. aerobics class

b. yoga, initially in her own room yoga, progressive muscle relaxation, and aerobic exercise have all been shown to be effective in managing stress. However, clients with histories of sexual abuse and issues with body image may initially feel unsafe practicing these techniques in a group atmosphere. Although A can be relaxing, these would not address the goal of using physical fitness as a means for relaxation.

While preparing a presentation for a professional conference, an OT planned to copy and distribute an article but realizes the name of the author is missing. Which of the following is the MOST appropriate action for the OT to take? a. distribute the handout and apologize for not having the authors name b. show the handout with an overhead projector and apologize for not having the authors name c. .use the handout only as a resource while developing the presentation d. use the article without mentioning the author.

c. .use the handout only as a resource while developing the presentation According to the AOTA Code of Ethics, OT practitioners must give credit when using the work of others. The other answers do not give credit to the author.

An OT practitioner is planning to introduce CIMT for an individual who recently experienced a left CVA as she meets the motor criteria suggested in the hospitals protocol. What is the suggested motor control inclusion criteria for CIMT? a. 15 degrees wrist extension and 10 degrees of extension for each digit b. 5 degrees of wrist extension and 10 degrees of individual digit extension c. 20 degrees of wrist extension and 10 degrees of extension for each digit d. 30 degrees of wrist extension and 20 degrees of extension for each digit.

c. 20 degrees of wrist extension and 10 degrees of extension for each digit answer C is correct as control of the wrist and digits is necessary to engage in this type of intervention.

An individual with an ID has a job assembling packets that include a plastic knife, fork, spoon, and a napkin. He uses a visual sample for guidance. The OT notices that when he runs out of knives, he continues to assemble to packets anyway, even though they are incomplete. At which level of Allen Cognitive Level is this individual functioning? a. 2 b. 3 c. 4 d. 5

c. 4 clients functioning at level 4 perform goal directed actions in response to visual cues. These clients are able to complete short tasks. Attention is up to an hour.

the OT is working with a client with an SCI. Which level of injury would benefit most fro using a wrist-driven flexor hinge orthosis during a prehension activity? a. C1 b. C3 c. C6 D. T1

c. C6 an individual with this injury has some use of the abducutor pollicis longus, extensor pollicis longis, extensor digitorum communis, and extensor carpi ulnaris. A and B have higher level lesions that lack the wrist extension strength to operate this orthosis. D is able to grasp and manipulate utensils without difficulty.

An OT manager is preparing the outpatient OT staff for a visit from an accrediting agency the accrediting agency that surveys inpatient and comprehensive outpatient rehab programs is BEST represented by which of the following? a. AOTA b. JCAHO c. CARF d. NBCOT

c. CARF The Commission on Accreditation of Rehab Facilities is the regulatory agency for the provision of rehab services. AOTA was formed in 1917 and they were originally known as JCAHO. NBCOT is the agency that develops and administers the examination for registration as an OT.

During initial session with an individual diagnosed with major depression, the OT observes that the individual is unwashed and dressed in dirty clothing, suggesting obvious ADL performance deficits. Which is the BEST eval tool to use with this individual? a. FIM b. skilled observation and close-ended questions c. COPM d. family member interview regarding the individuals ADL performance

c. COPM In depression, the barrier is not lack of knowledge or ability to perform the ADL but lack of volition, interest, or drive. Therefore, the COPM, which is the most client-centered is the best choice. The FIM assesses ADL performance from a physical perspective. Skilled observation is an effective way of assessing ADL performance; however, open ended questions are more appropriate.

an individual has a history of substance abuse and poor social skills and is cognitively intact. She has enrolled in a program with an emphasis on developing skills to live independently in the community. The philosophy of the facility is guided by the recovery model. Which assessment should be part of the initial eval? a. FIM b. AMPS c. COMP d. Allen diagnostic model

c. COPM The recovery model emphasizes empowerment of the individual by providing opportunities for self-determination, access to resource, and the attention to the individual's values and goals. The COPM is consistent with this because it is self-directed and client-centered. The film evaluates performance of self-care tasks and was designed for physical rehabilitation populations. The amps measures ADL and ideal performance but is not meant to be used with individuals that have underlying psychosocial impairments. answer D is a cognitive evaluation used to determine cognitive disability.

An OT performs an initial eval on an individual who requires mod assistance for UB and LB dressing. The OT expects the individual to achieve independence in dressing, with occasional assistance for small fasteners by discharge in 2 weeks. The initial G-code and severity modifier are determined to be G8987-CK. How should the self-care goal status be identified? a. G8987-CI b. G8987-CN c. G8988-CI d. G8988-CN

c. G8988-CI Medicare G-codes and severity modifiers must be used when submitting outpatient claims to medicare. The g-code for goal status is G8988, and the G-code for discharge is G8989. Modifiers indicate level of functional limitation and range from CH (0% impaired, limited, or restricted), to CN (100% impaired, limited, or restricted)

An OT is working with an individual with depression who is cognitively intact but demoing difficulty carrying out self-care and other ADL tasks. The OT, who has no advanced certifications, would like to identify a standardized assessment to measure ADL performance. Which is the MOST appropriate tool for this purpose? a. Bay Area Functional Performance Eval. b. Routine Task Inventory- Expanded. c. Kohlman Evaluation of Living Skills. d. Assessment of Motor and Process Skills.

c. Kohlman Evaluation of Living Skills the KElS combines interview items with simulated performance to obtain information about 17 living skills in the areas of self-care, safety and health, money management, transportation, telephone use, and work and leisure. The AMPS assesses motor and process skills, sometimes embedding a task that is in an ADL category. In order to administer the AMPS, one must have special training. Therefore, the OT could not use this immediately. The routine task inventory uses observation to assess 14 different ADL aspects and is based on the cognitive disabilities model which would not be appropriate for this client. The bay area functional performance eval was designed to measure performance skills such as memory, organization, attention, motivation, and frustration tolerance. Although it can help the OT develop conclusions about ADL performance, it is not actually an ADL evaluation tool.

an OT working in an outpatient setting is considering the use of the Ayers SI for a child with a learning disability. Which intervention is most consistent with this approach? a. student is included in classroom-based group that includes a program of varied sensory stim activities b. student uses a small, weighted blanket on his lap during seat work periods in the classroom. c. OT encourages child to engage in a playful environment, with ramps, scooters, ladders, balls, manipulatives, and vibrating toys d. OT develops planned sensory diet that includes sensory stories to promote students participation in the classroom activities and routines

c. OT encourages child to engage in a playful environment, with ramps, scooters, ladders, balls, manipulatives, and vibrating toys A and B are incorrect because it discusses a more classroom or school based setting as opposed to outpatient therapy.

In order to determine oxygen saturation levels during an ADL evaluation, the OT should: a. count respirations b. measure HR c. Read the pulse oximeter d. Note the individual's breathing patterns

c. Read the pulse oximeter answers A and B may indicate the individual is not receiving enough oxygen, but they also occur commonly with increased activity levels. It is important to note breathing patterns when working with individuals with compromise lung capacity, such as fast or shallow breathing. although any of these behaviors may be indicative of decreased oxygen saturation, specific information about the saturation levels cannot be ascertained without a pulse oximeter

The OT asks an individual with schizophrenia to describe what brought her to the hospital for admission. The individual response by saying "I took a cab". How would the OT MOST likely identify this response? a. delusional thinking b. a distractible response c. concrete response d. an insightful response

c. a concrete response Literal responses reflect concrete thinking. Delusional responses would most likely be completely off topic. A distractible response would change the topic or stop in the middle of responding. An insightful response would include reasons that led up to being hospitalized.

The OT learns from reviewing the chart that an individual is exhibiting positive symptoms of schizophrenia. during the initial eval, what should the OT expect to encounter? a. a Thomas voice b. very little facial expression c. false perception of reality d. difficulty concentrating

c. a false perception of reality The symptoms of schizophrenia can be divided into negative and positive symptoms. negative symptoms result in a trait loss such as flat affect, impoverished thought process, lack of interest or energy, inability to experience pleasure, and inability to sustain attention. positive symptoms add traits such as delusions, hallucinations, disorganized speech, and disturb sleep patterns. answer c is a positive symptom. negative symptoms tend to persist after the positive symptoms which are treated with medications.

which would the OT recommend for a 12 y/o child with mild visual impairments who desires independent use of the home telephone? a. speaker phone b. hands-free phone c. a phone with extra large buttons d. phone with a receiver holder

c. a phone with extra large buttons The other answers would assist individuals who cannot hold the phone due to incoordination, weakness, or loss of hand/UE function.

an OT is working as part of a multidisciplinary team that contributes to an IEP. How is an IEP most accurately described? a. a contract that lists the students goals and services intended to help achieve those goals b. a document that outlines the districts educational curriculum together with the states learning standards c. a process and legal document that outlines strengths, needs, and educational services for a student with a disability d. a plan of services and supports for families and their infants and toddlers with disabilities

c. a process and legal document that outlines strengths, needs, and educational services for a student with a disability It is both a summary and collaborative team process that includes eval, planning, and program implementation.

an individual with an eating disorder attends a partial hospitalization program as part of an agreement to use a harm-reduction approach to avoid hospitalization. However, at the conclusion of the Healthy eating group that day, the client refuses to eat any of the food. Which is the best action for the OT to take in response to the situation? a. set limits by reinforcing the inability to return to the group unless there is an agreement to eat b. explain the importance of meal planning and meal preparation as key elements of recovery c. accept her choice and support the decision to at a minimum, avoid diet pills, laxatives, and diuretics d. encourage attendance at the stress management group that will be offered later that day

c. accept her choice and support the decision to at a minimum, avoid diet pills, laxatives, and diuretics the harm reduction model is used with individuals who may be unwilling to participate in treatment if the expectation is full recover. A would be inconsistent with this model. B and D are important interventions, however, these answers do not specifically address the issue of safety, which is primary in the harm reduction model.

an individual who demonstrates compulsive behaviors is repeatedly late for work and is in danger of job loss as a result of spending too much time n morning grooming activities. upon observing this, how should the OT respond? a. explain that washing and brushing so many times is unnecessary b. encourage speaking about his fear one-on-one c. acknowledge the use of rituals to cope with anxiety d. discontinue group work and provide individual treatment

c. acknowledge the use of rituals to cope with anxiety OCD is a type of anxiety disorder that involves the unnecessary repetition of actions. Demonstrating empathy and and acknowledging the challenges are essential components of a therapeutic relationship. A will not help modify the behavior. B is used for individuals with phobias. And D is indicated when an individual cannot tolerate a group environment

An OT is treating a child with autism spectrum disorder. To maximize the child's benefit from intervention using Ayers sensory integration, what does the therapist need to ensure? a. sensory strategies are provided to enable child to process proprioceptive and tactile stimuli b. only one option for activity is available at a time, to encourage focus. c. active participation and self direction in activities matched to identify needs d. highly structured session, facilitating step by step learning.

c. active participation and self direction in activities matched to identify needs Ayers sensory integration includes emphasis on the child's inner drive and active participation. Therapeutic gains are maximized if the child is fully invested . The other answers are incorporated as they create an intervention context that is counter to essential characteristics to this approach.

What is the MOST important aspect of administering and scoring a standardized test? a. judgment to determine how best to administer the test. b. previous experience as a way to gauge test results c. adherence to specific instructions for administration and scoring. d. practice in administering the test items.

c. adherence to specific instructions for administration and scoring. In standardized assessments, the instructions are detailed and fixed so that procedures are followed consistently. Following these instructions assures the highest level of reliability and validity possible. Subjective judgment and previous experience may be factors in administration of nonstandardized tests. Although practice of a attest can help determine competence in the use of a test, it would not influence how to administer and score the test.

When structuring an intervention to promote sensory modulation for a child with tactile defensiveness, what is the BEST initial approach? a. apply intense light touch stim, such as tickling the abdomen, for desensitization b. avoid all forms of tactile stim to accommodate the childs preferences c. allow the child to self apply tactile stim to maximize the childs tolerance d. eliminate all deep pressure tactile stim to decrease defensiveness

c. allow the child to self-apply tactile stim to maximize the childs tolerance A is incorrect because light touch is particularly disturbing to children with tactile defensiveness and may create feelings of overwhelming anxiety. B is incorrect because avoiding all forms of sensation is virtually impossible and the child would not be able to develop coping skills. D is incorrect because the therapist would use increased pressure touch and increased proprioceptive aspects to help the child manage stim.

In the comfortable but crowded OT clinic of a mental health partial hospitalization program, a client is having difficulty concentrating on problem-solving tasks. Which of the following environmental strategies would be most likely to enhance the clients ability to perform the problem solving tasks in this clinic setting? a. play pleasant background music b. decrease the intensity of the lighting in the clinic c. allow the person to perform tasks in a quiet, separate area d. adjust the temperature and ventilation in the clinic

c. allow the person to perform tasks in a quiet, separate area all of the answer choices are environmental strategies which can impact the ability to become engaged in activity. But answer C is most likely to allow concentration. A can be very effective to shift attention and decrease agitation but may provide too much sensory stim. B may help decrease stimulation in the room but encourage relaxation rather than concentration. D could enhance the environment but from the question the human environment seems to be the most distracting feature in the clinic, not the temperature.

an OT is teaching an individual who recently sustained an above-elbow amputation how to tie shoes with one hand. Which of the following methods would the OT most likely implement to facilitate success with shoe tying a. problem solving b. retraining c. altering the task d. compensation

c. altering the task when the task method is altered, the same task objects are used in the environment, but the method of performing the task is modified to make the task feasible given the clients impairments. A is the ability to organize info from several levels to generate a solution to a problem. B teaches the same activity to the individual who had previously mastered those skills. D would be avoiding performance of the activity entirely by using an alternative piece of equipment or method.

An OT/OTA team needs to report discharge info and document the info in the patients chart. At what level can the OTA participate in the discharge planning process? a. An entry level OTA may perform the task independently b. an intermediate level OTA may perform the task independently c. an OTA may contribute to the process, but not complete the task independently d. an OTA cannot perform the task owing to regulatory statutes.

c. an OTA may contribute to the process, but not complete the task independently Because of the analytical nature of provision of discharge recommendations, the OTA does not complete this activity independently, regardless of level of experience. Answer D is incorrect because it does not allow collaboration with the OTA.

the OT is observing a 3 y/o during toothbrushing. The child demos good bilateral UE/hand strength but decreased dexterity. Which piece of equipment would the OT most likely encourage the child to use during toothbrushing? a. small soft bristle toothbrush b. a toothbrush with a universal cuff c. an electric toothbrush d. a soft sponge tipped toothette

c. an electric toothbrush this allows more thorough cleaning and it is good for children with limited dexterity, but it may be difficult for children with weakness.A would most likely assist a child with tongue thrust. B would assist a child with decreased grip strength. and D is typically indicated for children with hypersensitivity.

An OT is working with an individual who is preparing for discharge from a psych facility. The patient has been repeatedly readmitted to the hospital when acute symptoms emerged after forgetting to take prescribed medication on a regular basis. Which of the following approaches to medication adherence would be most appropriate? a. a psychodynamic approach to help her address the stigma related to taking her meds b. a psychoeducational approach to teach her about the medication she needs to take c. an environmental approach that provides an auditory cue when it is time for her to take her meds d. a cognitive-behavioral approach that addresses the patients attitudes about taking meds with negative side effects.

c. an environmental approach that provides an auditory cue when it is time for her to take her meds Medication noncompliance is the primary factor related to frequent readmission. The reason this patient is noncompliant is because of poor memory. The other issues are not indicated in this scenario

Which of the following questions is most appropriate for evaluating self-awareness in an individual recovering from a TBI? a. how often do you find yourself forgetting appointments b. what would you do if you smelled smoke coming from the kitchen c. are you any different now compared to what you were like before your injury d. how would you find out which bus to take to get to work by 9 am?

c. are you any different now compared to what you were like before your injury Answer A would be useful to asses safety skills. D addresses problem-solving and planning. the individual would need self awareness to answer question A.

an OT who needs to transfer a morbidly obese man is not confident that it can be managed by one person. The best action for the practitioner to take is: a. use proper body mechanics b. request that a PT perform the transfer c. ask another person for assistance d. refrain from transferring the patient

c. ask another person for assistance trying to attempt this transfer alone could result in injury to the patient or therapist. Asking someone else to do a difficult task is not professional. D is not an option.

An OT is offered a job upon completion of fieldwork and accepts the position even though she has not yet applied for her license. The employer wants her to start working immediately. What would be the BEST action for this therapist to take under the circumstances? a. schedule an immediate start date and send for a license hoping it will arrive in time b. confide in the rehab director and follow her recommendation to start as scheduled c. ask whether the company can delay the start date until her license arrives d. start the job knowing that no one with the company will ask to see her license.

c. ask whether the company can delay the start date until her license arrives All OT practitioners must hold a current, updated state license if residing in a licensed state. Students completing fieldwork II should apply for a temporary license prior to beginning their job search. The other answers would all be unethical and illegal.

An OT is working with his first level I fieldwork student. What should the focus of this experience be? a. facilitating clinical reasoning b. achieving entry-level competence c. assisting the student in developing a general understanding of client needs d. encouraging the student to partake in ethical and reflective practice.

c. assisting the student in developing a general understanding of client needs According to AOTA, the goal of level I fieldowkr is to introduce students to the fieldwork experience, to apply knowledge to practice, and develop understanding of the needs of clients. Although the other answers are all significant aspects of student development, they are most representative of expectations during level II fieldwork experience.

a 3 y/o demos the ability to use the toilet independently except for wiping and readjusting clothing afterward. This behavior indicates that the child is performing at which level? a. significantly below age level b. slightly below age level c. at age level d. above age level

c. at age level At 3 years, a child is expected to know when he or she has to use the toilet and be able to get on and off the toilet. Complete independence is usually achieved by 4 or 5 years. By the age of 2 years, most children have daytime control with occasional accidents.

An adolescent with a history of shoplifting and gang violence has been hospitalized with a diagnosis of conduct disorder. During a task group, what are the most important performance skills for the OT to evaluate? a. perceptual motor performance b. leisure and vocational interest c. attention span and social interaction skills d. interest in performing and ability to perform multiple roles.

c. attention span and social interaction skills. Conduct disorders often involve aggression towards people or animals and destruction of property. The individual with conduct order violates rules, age appropriate norms, or the rights of others, evidenced by acts of aggression against people or animals, property destruction, lying, or theft. These symptoms cause impairment in job, school, or social life. When working with this population, the OT typically addresses areas of attention, impulse control, and social skills. Children with developmental delays typically require intervention for perceptual motor skills. Multiple role performance is more developmentally relevant to adulthood than adolescence. Although leisure and vocational interests may be relevant, the other issues will be more significant for a person with this diagnosis.

A child with learning disabilities resulting in low frustration tolerance and poor self esteem is learning how to tie shoelaces. Which method is the MOST appropriate for the OT to introduce to this child? a. physical guidance b. verbal cues c. backwards chaining d. forward chaining

c. backwards chaining In this method, the OT completes all of the steps to a task except the last one. As the child becomes competent, the OT completes all but the las two steps, and so on until the child is able to perform the entire activity. This method is helpful for children with low frustration tolerance. A requires the least amount of cognitive ability and provides the child the opportunity to learn through a sensory motor experience. Verbal cues may be perceived as intrusive and critical. Forward chaining begins with the first step and the practitioner finishing the rest. The is method is helpful for children with difficulty sequencing and generalizing skills.

An OT is planning a meal prep activity for an individual with cognitive deficits in the areas of attentional and organizational skills. What is the most appropriate activity to use FIRST in addressing sequencing skills? a. setting the table b. planning a meal c. baking cookies following a recipe d. preparing a shopping list

c. baking cookies following a recipe this is a well-delineated meal prep activity that provides structure with a specific sequence of tasks. Setting a table of preparing a shopping list do not necessarily require sequencing of tasks. Planning a meal involves a great deal of organizational ability, and would not be appropriate as an initial activity to address goals related to sequencing tasks.

a child with significantly low muscle tone caused by Duchenne MD is losing trunk control when sitting. The OT assesses the childs ROM, strength, and endurance, which indicates that the OT is MOST likely relying upon which FOR? a. NDT b. SI c. biomechanical d. visual perceptual

c. biomechanical this reference is applied for positioning children when a person cannot maintain posture through appropriate automatic muscle activity because of neuromuscular or musculoskeletal dysfunction. A is incorrect because NDT is concerned with improving posture and movement but supportive equipment is prescribed for that purpose. B is incorrect as sensory integration concerns interaction between the sensory system to provide information that results in complex behaviors. D is incorrect because it is concerned with receiving and understanding visual stim and is used to guide intervention

The OT fitted a 6 y/o with an adapted seat to use during mealtime and other tabletop activities at home. What info is MOST appropriate to convey to the parents? a. use the seat as the child is willing b. bring the seat in for each weekly therapy session c. bring the seat in for reeval in 4 to 6 months d. keep the seat until the end of the IEP.

c. bring the seat in for reeval in 4-6 months Once a child receives a seating mobility system, the therapist must reevaluate fit every 4 to 6 months to account for the child's growth, changes in posture, comfort, and functional abilities. A is incorrect because it should be regularly used. Weekly adjustment is not necessary and the end of the IEP may be more than 6 months away.

When developing play activities for a child for a child with acute juvenile RA, what types of activities should the OT avoid? a. light touch b. vestibular c. resistive d. elevated temperatures

c. resistive Resistive activities are contraindicated for acute stages of RA. Light touch is likely to be disturbing to a child with tactile defensiveness. Rapid vestibular stimulation is contraindicated for a child who is prone to seizures. Temperature is more relevant to clients with MS because of high temperatures exacerbating symptoms.

an OT is seeing a home health care client who is in the moderate stages of Alzheimers and whose memory loss is now interfering with the performance of daily self-care activities. What is the MOST relevant OT intervention for the client? a. memory retraining activities for the client b. ADL retraining program for the client c. caregiver instructions in problem solving d. leisure activity planning

c. caregiver instructions in problem solving The most appropriate intervention at a moderate to moderately severe level of alzheimers is to help the caregiver problem solve and recognize need for initiation and cueing during tasks. At this stage, new learning is not possible, therefore A and B are incorrect. Leisure activities that are structured to meet the needs of the client could be helpful, but will not address the primary problem of performane of self-care activities.

And OT is evaluating a child's shoulder stability as part of an assessment of pastoral control. which method is most likely to be used for this assessment? a. child on stomach with shoulders abducted and legs extended and time how long the child can maintain this position b. child on back with arms crossed across the chest and time how long the child can maintain this position c. child in an unsupported chair and ask the child to grab the OT sums, push and pull, and see whether the child can remain relatively still d. child imitates OT and raises both arms forward overhead out to sides return upward and repeats for one minute

c. child in an unsupported chair and ask the child to grab the OT's thumbs, push and pull, and see whether the child can remain relatively still. The correct response is answer c because stability results from the ability of opposing muscle groups to contract at the same time to provide stability around a joint. they approach given an answer see enables the therapist to observe if muscles around the glenohumeral joint contract together so the left and right shoulder girdles provide stability to the upper torso, enabling the child to resist the therapist force and retain the upright seated position. answer A is the ability to maintain the prone extension position, while answer B is an example of supine flexion. answer D may be used to examine range of motion, strength, and symmetry of both upper body sides. however, it is incorrectly assessing shoulder stability.

an OT is working with an older adult who has been admitted for depression with concerns about upcoming retirement. The individual has progressed from the precontemplation stage to the action stage. Which is the best action stage activity for this individual? a. record the pros and cons of retiring b. list 5 concerns about retiring c. choose 1 goal to address next session d. identify potential community groups of interest to join

c. choose 1 goal to address next session B would be an activity for the precontemplation stage. A would be for the preparation stage, and

In planning OT services using a behavioral approach, what are the best intervention activities to include? a. opportunities for the child to express his or her interests b. play activities that encourage interaction between all children c. clear expectations and reinforcement for desired performance d. role play to rehearse and practice desired performance

c. clear expectations and reinforcement for desired performance positive behavior is fabricated when children understand expectations and simple rules and are reinforced for desired performance. The other answers do not reflect these concepts.

a client who is diagnosed with depression participates in the group activity for 10 min and then states she is just too tired to continue. how would the OT document this in the subjective portion of the SOAP note? a. fatigue limits the clients participation in the therapy process b. low level of motivation will limit client in therapy c. client stated she was too tired to continue d. client appears unmotivated

c. client stated she was too tired to continue One of the main symptoms of MDD is decreased energy. The subjective portion contains info verified from the client. Clinical judgement or factors contributing to impaired performance belong in the assessment part of the SOAP note.

during intervention planning, before the OT develops goals and objectives for a 6-year-old child with autism, what other steps need to be completed? a. identify a practice model and frame of reference and select intervention activities b. engage in the clinical reasoning process to identify priorities for this student c. collaborate with other team members to develop long-term goals and project duration of services d. review schools requirements to understand the preferred format and style for goal writing

c. collaborate with other team members to develop long-term goals and project duration of services Intervention planning should be a collaborative effort. Answer A is incorrect as a practice model and specific activities are not selected before intervention goals are established. B is incorrect as the OT uses clinical reasoning throughout the OT process. D should already be understood before this point.

An individual with a history of homelessness, substance abuse, and mental illness has begun to participate in the OT program in a shelter for individuals in recovery. the individual has indicated an interest in employment as a primary goal and reports previous work as a carpenter 10 years ago. What is the FIRST action of the OT in addressing work potential? a. observe the individual during a morning cleaning activity b. identify a low-demand volunteer job within the shelter c. conduct a readiness assessment d. determine qualifications for supported employment programs

c. conduct a readiness assessment this individual has a history of chronic mental illness. according to the transtheoretical model, a person will not change until he is ready to change. a readiness assessment is an important first step in any rehabilitation assessment process and must come before conducting functional and resource assessments. answer a and answer B qualifies for any type of work program which will occur later.

An OT is considering the use of an electronic documentation system within a private practice setting. Some of the advantages to electronic health records are increased legibility, speed of documentation, and improved organization. What are the disadvantages of such a system? a. issues related to storage capacity b. retrieval of info c. confidentiality, time, and money d. determining a system for remote site access

c. confidentiality, time, and money These are legitimate concerns. The other answers are all potential advantages to using this type of documentation system.

A hand therapy research group is attempting to determine whether the experiment caused a change in the dependent variable in a study that used orthoses as the OT intervention in relation to contracture prevention, or whether issues related to the actual passage of time and or accuracy of the tool used to measure the contractures were responsible for the results. The research group is looking at which of the following? a. study bias b. external validity c. internal validity d. context specificity

c. internal validity Internal validity is the ability of the research design to answer the research question accurately.

an outpatient OT has administered, scored, and interpreted data from interview, observation, and assessments. what is the first step in developing recommendations for the child? a. describe and prioritize the child's problems and needs b. document evaluation findings and language and a manner that are useful for team members, including the child's parents c. consider the rules and occupations required for this child and develop recommendations that increase the child's competence in these areas d. analyze the effects of the child's difficulty on developmental tasks and performance

c. consider the rules and occupations required for this child and develop recommendations that increase the child's competence in these areas This is the first step to ensure recommendations are functional for the child. Answers A and D are part of the eval processes. Answer B is the final step in the eval.

A child has considerable difficulty with problem solving when playing with interlocking blocks, becomes frustrated, and gives up easily. An OT would MOST likely suspect a problem in what area? a. sensorimotor play b. pretend play c constructional play d. play with rules

c. constructional play constructive play is described as manipulation of objects to create something. Sensorimotor play is sensory exploration characterized by practice and repetition. Pretend play involves manipulating people and objects to create situations. Game play involves the acceptance of prearranged rules.

An individual with cognitive deficits exhibits little transfer of skills from one activity to the next. Which intervention would be BEST to assist this individual in performing the steps of doing laundry? a. performing memory drills of the steps involved in doing a laundry activity b. placing serial pictures of a laundry activity in sequence c. consulting a checklist of steps while doing laundry in context d. reading a story about a person doing laundry with the individual and then discussing the story.

c. consulting a checklist of steps while doing laundry in context. Making a checklist would provide an external memory aid during practice of the functional activity. The other answers are methods that require an individual to be able to transfer learning of skills from one context to another.

The OT has completed and eval on an individual with serious and persistent mental illness observed during a music therapy group. The individual actively engaged with other group members and was able to problem solve with other group m embers when they were asked to select their instruments. It was clear the individual was interested in participating in a performance scheduled for a week later. However, some assistance was needed to sustain appropriate social interaction for setting limits. Which group level would BEST enable social participation for this individual? a. parallel b. associative c. cooperative d. mature

c. cooperative The individual exhibited eagerness to interact with other members, express ideas, problem solve, and sustain interest in longer projects, which are characteristics associated with expectations of a cooperative group. Individuals participating in groups at associative levels have less well-developed social skills, fewer problem-solving abilities, and shorter attention spans. Individuals working in parallel groups would not be interacting with others and individuals in mature groups would not need the level of intervention required by this individual.

an OT/OTA team begins to provide OT services through a certified home health agency. What is the most critical component for establishing a successful collaborative relationship in this treatment setting? a. determining reimbursement when completing joint visits b. establishing a system for the OT to countersign the OTAs documentation c. creating effective communication via weekly staff meetings, voice mail, and faxes d. developing a handout to educate clients regarding how the team will provide services

c. creating effective communication via weekly staff meetings, voice mail, and faxes A may provide useful info but is not critical in establishing a collaborative relationship. A would be something that would transpire as a result of effective collaboration. D could be done collaboratively, but it is a one time event and is not as effective as answer C.

during the interview with parents of a 3 y/o child with mild CP, the OT learns that the child is regularly fed by his grandma and does not have any independent feeding skills. Before setting self-feeding goals, what should the OT explore? a. degree of abnormal muscle tone in the childs UEs b. possibility of DD c. cultural context and family interaction patterns d. need for AE

c. cultural context and family interaction In some cultures, children are fed by a caregiver throughout the preschool years. The other answers may be valid issues as well, but should be addressed after the OT has familiarized himself or herself with the families cultural context/

As the school team meets to develop the IEP for a 3rd grade student with autism, the OT helps to project longer term academic and functional performance outcomes for the student in middle school and high school. What is the MOST significant benefit of this approach for program planning? a. assurance that services will be available for the student in future years b. likelihood for optional progress toward relevant post-high school goals. c. current services that focus on steps to achieve targeted long-range outcomes. d. avoidance of unnecessary duplication of services from year to year.

c. current services that focus on steps to achieve targeted long-range outcomes Answers A and B are incorrect, as targeting future outcomes does not ensure services are available or goals are achieved. Services duplication is avoided by effective collaboration.

The OT is considering the use of paraffin with a client who has chronic arthritic condition. paraffin has been shown to be most efficacious with: a. assisting with tumor management b. assisting with tendon repair c. decreasing stiffness and improving ROM d. decreasing deep vein thrombosis

c. decreasing stiffness and improving ROM paraffin is primarily used to decrease stiffness and improve rom, and it is frequently used to treat chronic arthritic conditions and also offers pain control as a primary advantage. other answers are situations in which superficial heat agents should not be used.

A construction worker reports experiencing severe anxiety when climbing ladders. The OT has only received authorization for one session to work on self-management strategies that the individual can use to reduce anxiety while on the job. what is the most appropriate strategy to teach this client? a. autogenic training b. meditation c. deep breathing d. progressive muscle relaxation

c. deep breathing All four answers are examples of self-management techniques however deep breathing is most appropriate because it can be taught quickly and used in a relatively short amount of time.

a LTG for an individual following a hip arthroplasty is independence in LE dressing with AE. the most relevant STG for the OT to work on would be for the individual to: a. increase standing tolerance to 10 min b. increase hip flexion to 90 degres c. demo adherence to prescribed hip precautions d. apply EC techniques during dressing activities

c. demo adherence to prescribed hip precautions A and B is not necessary for independence in dressing. C however, is mandated by the physicians for the individual to perform the activity safely. D is appropriate for individuals with low endurance, but individuals are not necessarily going to have this issue after a hip arthroplasty.

As a fieldwork assignment, a level II student introduces a new assessment tool for evaluating work skills of individuals with ID to the fieldwork educator. The fieldwork educator wants to test the reliability of the instrument. What is the BEST way to do to this? a. ask several experts within the department for their opinion and document them b. ascertain that a new therapist is able to obtain the same results as those of an experienced therapist c. demo that a patient will perform in a consistent manner when being evaluated twice over the course of 2 weeks d. determine an area to eval, assess the need for an eval tool, and determine the eval format

c. demo that a patient will perform in a consistent manner when being evaluated twice over the course of 2 weeks. an instrument is considered to be reliable when a patient will perform consistently, producing the same response. A is a method for testing validity. B is a method for demonstrating service competence. D are the beginning steps for designing a new eval tool.

The director of an assisted living facility has asked the OT to help them develop a falls prevention program. The OT should FIRST: a. evaluate residents of the facility who have fallen in the past 6-12 months b. interview facility administration and staff to determine goals for the program c. determine how frequently falls occur, how many individuals are affected, and how the problem is currently being addressed d. provide the director with a menu of recommendations that will address the problem of falls in the facility and determine a starting point.

c. determine how frequently falls occur, how many individuals are affected, and how the problem is currently being addressed The first step in program development is to profile the community. This includes collecting demographic info about the population. Next consultant should perform a needs assessment by collecting data from individuals with a variety of viewpoints. Goals are established and recommendations are made once the needs assessment is complete.

Which of the following is MOST important to include at the onset or, acute-inflammatory stage for an individual with complete UE and LE paralysis as a result of Guillain Barre syndrome? a. ADL training b. balance and stabilization activities c. developing proper communication tools as needed, and proper positioning of trunk/head and UEs. d. resistive activities for the intrinsic hand muscles

c. developing proper communication tools as needed, and proper positioning of the trunk/head and UEs. The acute inflammatory phase manifests itself as acute weakness in at least two limbs that progresses and reaches its maximum in 2-4 weeks with increasing symptoms. Therefore, answer C is imperative. The next step is answer A. Then Answers B and D are implemented later, during the recovery stage, after strength begins to improve.

An OT performing a motor skills eval observes that a child is awkward at many gross motor tasks. Though able to skip rope forward, the child is unable to skip rope backward, even after several attempts. This information would lead the therapist to be particularly observant for which additional signs? a. delayed reflex integration b. inadequate bilateral integration c. developmental dyspraxia d. general incoordination

c. developmental dyspraxia. Dyspraxia is a deficit in learning new motor skills, planning new motor actions, and generalizing motor plans. Children with dyspraxia often learn tasks such as jumping role with great difficulty, effort, and considerable practice. However, when the task is altered, the child is unable to adapt to the task for a long period. Answer A is incorrect because there is no description to suggest reflex activity affects posture and movement. B is incorrect because the child coordinates the two sides of the body to skip rope forward. D is also incorrect because general incoordination would likely affect performance during both forward and backward jumping.

An OT is running a group for women with eating disorders. The activity is to create a gift box with a message inside about what they appreciate about another individual in the group. After accidentally tearing the side of her gift box, one client looks very distressed and stops participating. Which response is BEST for the OT to enable the client to continue working on her goal of self-acceptance? a. set up a time to work with her one-on-one b. take the box, make the necessary repairs, and encourage her to continue writing her message. c. discuss the symbolism of how we are all experiencing breaking and healing d. bring her the supplies to start over again, and leave out the message part of the activity.

c. discuss the symbolism of how we all experience breaking and healing The OT must be supportive and empathetic and help the individual to allow herself to make mistakes. Repairing the box would send the wrong message, and working with her in a one-on-one meeting would remove her from the group. Downgrading the activity by removing the message would remove a key part of the activity.

When performing a preprosthetic eval of a patient who had a long above elbow amputation, the OT should measure the circumference of the residual limb at the: a. distal radio-ulnar joint b. proximal radio-ulnar joint c. distal humerus d. proximal humerus

c. distal humerus an individual who has undergone this procedure looses the limb at the level of the distal humerus resulting in loss of hand and wrist function.

An individual with a low back injury lives alone and must be able to do laundry independently. to Prevent reinjury, what should the OT instruct the individual to do? a. place the clean laundry basket on the floor next to a chair and sit for folding b. stop the activity when pain becomes severe c. divide the laundry into several smaller loads for carrying d. carry the laundry in one or two large loads

c. divide the laundry into several small loads for carrying. this will decrease low back strain. Answer A would require bending and twisting, two movements that people with back pain should avoid. Pain should be prevented as much as possible.

and OT has recommended strategies for use throughout the school day to support a young child with autism to participate in learning activities. when designing the plan to collect data regarding the effectiveness of the intervention, what must the OT include? a. confirmation that the teacher approves the plan b. a timeline to re-administer standardized tests c. documentation that the intervention was implemented as planned d. criteria for modifying the intervention

c. documentation that the intervention was implemented as planned. Although the teacher is in agreement, this is part of collaboration between the OT and the teacher and does not contribute to the measurement plan. B is not a necessary element in progress monitoring, as many standardized tests do not reflect changes in function and are not appropriate for all students.

during the OT session, a child begins a grand mal seizure. What is the most important action for the OT to take during the seizure? a. check breathing and administer mouth to mouth resusitation if necessary b. attempt to restrain the childs movements to prevent injury and place something in the mouth c. ease the child to a lying position, remove or pad objects nearby, and loosen clothing d. observe the child, document behavior, and record the duration

c. ease the child to a lying position, remove or pad objects nearby, and loosen clothing A is incorrect because checking breathing would not be done til the seizure has stopped. B is incorrect because attempting to restrain the child could lead to injury. D would not help to protect the child in the moment but would be necessary after the incident.

An individual with ALS and mild dysphagia becomes extremely fatigued at meals. Which is the FIRST intervention the OT practitioner should consider recommending? a. speak with the physician about tube feedings b. sit in a semireclined position during meals c. eat six small meals a day d. substitute pureed foods for liquids

c. eat six small meals a day This is the more simple, achievable option before considering surgical procedures or diet changes that may decrease QOL. An upright position is optimal when feeding individuals with dysphagia, and answer B can make swallowing more difficult or dangerous.

An OT is selecting activities for a childs treatment plan to achieve an IEP goal concerning the students completion of written expression during independent seatwork in the language arts curriculum. To best support the students engagement and provide optimal therapeutic value, the OT should: a. have the student complete tracing activities in progressively smaller physical areas to refine pencil control, increase visual-motor accuracy, and promote writing endurance. b. interview the student to learn how seatwork is most challenging and then engage the student in skill-building activities to increase self efficacy in that specific component c. embed self in the classroom during language arts seatwork performance to assess performance and trial multiple strategies as student participates in assigned work d. use teacher eval data to define students specific areas of need and implement activities that target practice in the teachers highest priority area

c. embed self in the classroom during language arts seatwork performance to assess performance and trial multiple strategies as the student participates in assigned work whole activities with multiple steps and a meaningful goal versus repetition of activity components elicit the child full engagement and participation. The other answers are incorrect as repeating a single component has minimal therapeutic value.

An OT is planning a program to address the needs of persons with Alzheimers disease, and their families, as part of a hospital outreach program. What area of intervention would be MOST beneficial to maintaining safety and supporting function at home for people in the moderate to severe stages of Alzheimer's disease? a. strength and endurance activities b. cognitive rehabilitation techniques c. environmental modifications d. assertiveness

c. environmental modifications this is the area of intervention that can best assist in maintaining safety and supporting function in the home. At this stage in the disease process, cognition will not be rehabilitated. Strength and endurance does not support safety. Assertiveness would be inappropriate for dealing with safety.

an individual is participating in an assertiveness training group within a psychiatric rehabilitation program. and what areas is the individual most likely to improve? a. engaging in relevant conversations with coworkers b. using appropriate facial expressions when disagreeing with coworkers c. expressing disagreement with coworkers in a productive manner d. using courteous behavior when disagreeing with coworkers

c. expressing disagreement with coworkers in a productive manner assertiveness is a social characteristic that empowers people to stand up for themselves. The other answers are examples of additional social interaction skills, however, they do not address assertiveness. A and B are examples of actions taken when engaging in conversation. and C is an example of proper social conduct.

in order to promote tenodesis when performing PROM, how should the OT position the wrist? a. neutral to promote finger flexion and extension b. flexion to promote finger flexion and extension c. extension to promote finger flexion and flexion to promote finger extension d. flexion to encourage finger flexion and extension to promote finger extension

c. extension to promote finger flexion and flexion to promote finger extension tenodesis is the reciprocal motion of the wrist and fingers that occurs during active and passive wrist flexion and extension.

An individual is able to complete the full ROM of shoulder flexion while in a side-lying position during an eval. However, against gravity, the individual is not quire able to achieve 75% of the ROM for shoulder flexion. This muscle should be graded as: a. good (4) b. fair (3) c. fair minus (3-) d. poor plus (2+)

c. fair minus (3-) fair minus is the grade given when an individual moves part through incomplete range of motion against gravity or through complete range of motion with gravity eliminated against slight resistance. grade 4 indicates ability to move their full range of motion against moderate resistance. A fair grade would be the ability to move through the full range of motion against gravity but not take any additional resistance. A grade of poor plus would move through full range with gravity eliminated and take minimal resistance before suddenly relaxing.

Following radiation therapy for breast cancer, an individual develops lymphedema in the right (dominant) UE. She is experiencing functional deficits and is referred to OT. Treatment to reduce the swelling will begin with: a. change of dominance training b. RUE exercise using isometric contractions c. fitting of compression garments d. a heat modality, such as a warm compress.

c. fitting of compression garments lymphedema can result in swelling that can be painful and stigmatizing. Manual massage can help reduce swelling. Change of dominance is not usually necessary. B or D would not be useful in this case.

an OT is ordering a wheelchair for an individual with progressive MS who complains of chronic overall weakness. The most important consideration the practitioner can make is the adaptability of the wheelchair in anticipation of: a. gradual gains in strength b. growth of the individual c. fluctuating weakness or further decline d. improved wheelchair mobility.

c. fluctuating weakness or further decline generally, equipment, environmental and behavioral mods help compensate for weakness. A and D are not characteristics of progressive degenerative diseases. Growth of a client is important to consider for pediatric needs.

During an infants OT session, the mother reports she has observed that her baby has difficulty with swallowing and frequently chokes. To reduce the risk of aspiration and facilitate swallowing, the OT should keep the head in what position? a. neutral b. slightly flexed c. slightly extended d. rotated toward the feeder.

c. foam tubing around the utensils C is correct because foam tubing increases the diameter of the utensils and larger grip diameters may help the child feed more independently. Swivel utensils are most appropriate for children who experience incoordination or tremors, whereas pediatric universal holders are commonly used when a child has no grip at all. Weighted utensils assist with motor incoordination.

a parent of four who was diagnosed with a right CVA is receiving home health OT to improve left UE function and sitting and standing balance. What is the most appropriate activity for the OT to recommend? a. stacking cones b. manipulating a door pulley c. folding laundry d. throwing a ball

c. folding laundry This challenges balance and UE functions in ways that are more functional than stacking cones or throwing a ball.

A participant in a study requires an MRI of the brain 4 times a month in a study that strives to combat the symptoms of depression. The lead investigator of the study informs the participant of the potential risks and anticipated high level of involvement required by the study. This interaction, in which the lead investigator of the study comprehensively informs the participant about her involvement in the study, is commonly referred to as a: a. review of remediation of risks b. confidentiality of study participant c. full disclosure d. withdrawal of rights for participation.

c. full disclosure any person who participates in a study has the absolute right to full disclosure of the purpose and procedures of the study. The other answers are all individual components of full disclosure.

The BEST way for an individual with hemiparesis and mild perceptual deficits to button a shirt is to: a. button all the buttons before putting the shirt on b. get the shirt all the way on, then line up the buttons and holes and begin buttoning from the top. c. get the shirt all the way on, then line up the buttons and holes, and begin buttoning from the bottom. d. use a buttonhook with a builtup handle

c. get the shirt all the way on, then line up the buttons and holes, and begin buttoning from the bottom It is easier to see the buttonholes at the bottom. Buttoning the shirt first may result in ripping the buttons out when donning the shirt. A buttonhook with a built-up handle is more helpful for an individual with finger weakness or incoordination.

An individual with a panic disorder feels so overwhelmed he cannot get himself from his room to OT group each morning. Which of the following strategies would be most helpful? a. reduce distractions and keep lights low b. provide a stimulating environment with real life opportunities c. give him a tour of the OT department and a schedule of activities d. leave doors open and avoid being alone with the individual

c. give him a tour of the OT department and a schedule of activities The CBT approach to panic disorder involves having the individual approach situations that cause anxiety and reduce the catastrophizing associated with the event. A may be useful environmental adaptations for individuals with mania or hyperactivity. B is recommended for individuals experiencing delusions. D is for individuals who are hostile or violent

an individual exhibits limited awareness of functional limitations resulting from his recent brain injury, attempts to perform transfers unsafely without assistance, and says he doesnt see the need for therapy. What is the best way for the OT to promote awareness and insight for this patient? a. have the individual explain why he believes he is not impaired b. provide a checklist of skills that must be present to perform various activities and review it with the patient c. have the individual predict his performance before an activity, then have him self evaluate his performance d. disregard the individuals perceptions and proceed with therapy.

c. have the individual predict his performance before an activity, then have him self evaluate his performance this would provide meaningful self-initiated feedback and would be the best way to increase awareness. A would not provide concrete immediate feedback. B would likely not be effective because the individual already feels that he possesses these skills. D does not address the needs to increase the patients awareness.

an individual with hand weakness has difficulty holding a fork. Using a biomechanical FOR, the OT should: a. elicit functional grasp using reflex-inhibiting postures b. stimulate the hand flexors using quick stretch to promote functional grasp c. have the individual repeatedly squeeze with the hand increasing amounts of resistance d. build up utensil handles

c. have the individual repeatedly squeeze with the hand increasing amounts of resistance this approach remediates deficits in strength, endurance, and ROM. A is a neurophysiological approach. B is an NDT approach. D is an example of a rehabilitative approach.

The OT is working with an individual who is in recovery and about to be discharged from an inpatient substance abuse program. The patient schools are to find a place to live and abstain completely from alcohol use. what is the most appropriate information for the OT to provide as part of the discharge planning process? a. how to manage medication and a schedule b. The elements of the recovery model and it's import c. healthy leisure participation in alcoholics anonymous d. group homes that use a harm reduction model and availability

c. healthy leisure participation and alcoholics anonymous. Providing this info is an example of CBT. A nurse would most likely be the individual involved with medication management. Information about the recovery model would have been introduced earlier. The hard reduction model is not in alignment with this individuals goals.

an OT writing a job description for an aide position in the OT department. According to AOTA guidelines, which of the following is beyond the scope of practice of an aide? a. cue an individual with schizophrenia to maintain attention to task during a weekly cooking group led by an OT b. practice use of a sock aid and shoehorn in the OT department with a patient who has had a hip replacement and is able to perform the task safely but takes an excessive amount of time c. help place food on the spoon for a patient learning how to use a universal cuff in the patients room at lunch time, while the OT supervisor runs a lunch group in the dinning room d. help a child maintain correct positioning during paper and pencil activities next to the OT who is working with another client

c. help place food on the spoon for a patient learning how to use a universal cuff in the patients room at lunch time, while the OT supervisor runs a lunch group in the dinning room An aid does not provide skilled OT services. It is trained by the OT to provide specific delegated tasks. The aide may be delegated to tasks when the outcome is predictable, the clients situation and environment are stable and will not require judgement or adaptations, the task or routine has been previously established, and the client has demonstrated ability in executing the task. this answer states that the patient is learning the universal cuff and therefore judgement would still be necessary for the OT.

a child with a learning disability has significant problems with visual memory that limit performance during test activities. how can the OT enhance visual memory? a. provide memory tasks in mass practice trials b. decrease demands for visual attention just prior to memory tasks to prime visual memory abilities c. help student identify simple rhymes to remember steps to complete homework independently. d. repeat visual memory activities so student can anticipate next steps.

c. help student identify simple rhymes to remember steps to complete homework independently. The OT embedded a recommended memory strategy into the childs daily routines to help him organize his work and enhance the retrievability through the use of language cues. A is incorrect as the mass practice is not optimal for generalization into functional tasks. B is incorrect as visual attention is a prerequisite to visual memory and D is incorrect because serial or varied repetition enhances visual memory

In preparation for an annual individualized Family Service Plan (IFSP) meeting, the OT is developing ideas for intervention to develop a child's fine motor performance for independent play participation. To match the principles of Part C early intervention programs, what would be the OTs BEST recommendation? a. family encouragement of the child to engage in play with toys he has at home, without concern for normal development. b. a periodic reassessment using the Peabody fine motor subtest to measure progress in fine motor skill achievement. c. home visit, during which OT will identify and encourage opportunities for fine motor practice within the family's preferred routines. d. increased weekly home visits so OT can implement therapy activities to promote child's fine motor skill development, using a remedial approach.

c. home visit, during which OT will identify and encourage opportunities for fine motor practice within the family's preferred routines. C is correct as it reflects the use of the natural environment. A is vague and does not help the parent identify toys most matched to the child's specific fine motor needs. B is incorrect as it describes an evaluation rather than an intervention. D is incorrect as it is counter to the principles of IEP programs designed to support learning and development within the context of children's everyday activities and routines.

An OT is conducting a perceptual function screening with an individual who has had a CVA. Which of the following informal screening activities would the therapist ask the individual to perform in order to identify the presence of agnosia? a. demonstrate common gestures such as waving. b. name objects through touch only c. identify or demonstrate the use of common household objects. d. read a paragraph and explain its meaning.

c. identify or demo the use of common household objects visual agnosia is the inability to recognize common objects and demo their use in an activity. Asking the individual to demo common gestures such as waving is a technique to screen for apraxia or the inability to perform purposeful movement on command. Answer B is asking the individual to perform stereognosis. Asking the individual to read a paragraph an explain its meaning would be a screening method for alexia, or the inability to understand written language.

An OT is determining the level of cognitive function in an individual with an ID, using the Allen Cognitive Disability model. In which way will this be MOST helpful to the therapist? a. identifying difficult behaviors that may interfere with intervention b. planning a training program to improve prevocational skills c. identifying the type of environmental supports that can maximize the clients level of function d. developing an educational plan to improve social skills.

c. identifying the type of environmental supports that can maximize the clients level of function OTs use tools such as the Allen Cog levels to determine what level of structure and environmental cues are needed to engage the client in task completion. A cognitive level will not give information on behaviors. Answers B and D are incorrect because the cog level assessment evaluates how a person processes info cognitively rather than evaluating specific prevocational or social skills.

an OT observes a child with a learning disability use an unusually tight grip when writing with a pencil. The child also frequently breaks the pencil point by applying too much pressure on the paper. What is the most likely cause of these observations? a. inadequate sensory processing related to the vestibular system b. limited motor planning c. inadequate sensory processing related to the kinesthetic system d. difficulty with visual system processing

c. inadequate sensory processing related to the kinesthetic system Good kinesthetic info enhances speed, reduces the need to visually monitor the hand during writing, and influences the amount of pressure applied to the writing instrument. A is not the correct answer because it primarily affects balance and general motor coordination. B is not correct because it refers to the ability to conceive of, plan, and carry out unfamiliar motor tasks. D is incorrect because the use of a pencil requires unconscious awareness of body position and pressure.

An individual with borderline personality disorder has been referred to OT. Which of the following would be most important to evaluate? a. ADLs b. IADLs c. Interpersonal skills d. sensorimotor skills

c. interpersonal skills the most common area of difficulty across all personality disorders, is interpersonal skills. Specific personality disorder categories indicate that there is some variation among the types of relationships that are impacted. For example, authority relationships seem particularly dysfunctional in those with antisocial personality disorders. ADLs and IADLs are often problems for individuals with mood and thought disorders. Sensorimotor deficits are more likely to be observed in individuals with schizophrenia.

An infant born 12 weeks prematurely has a hx of multiple medical issues, including retinopathy of prematurity, mechanical ventilation for 5 weeks, and poor feeding skills. The infant is now chronologically 4 months old (1 month adjusted age). She is medically stable and engaging, with a G-tube and 2 L of O2 by NC. What is the most appropriate intervention provided at this time? a. teach the parent positioning and handling to promote motor development b. instruct the parent how to perform PROM of all extremities c. include family's concerns into recommended caregiving practices d. provide interventions that incorporate multi-sensory stimuli.

c. include familys concerns into recommended caregiving practices It embraces the principles of family-centered early intervention. A incorporates only one area of the infants development and does not reflect teaching the parents how to position the infant within her regular routines. B is incorrect as family should be encouraged to help this infant actively move. D is incorrect as it does not reflect the therapist-parent collaboration.

An OT is exploring AT options to help a student with autism produce written assignments in the classroom. The teacher is concerned that because of the students rigid and perseverative behaviors, additional computer use in the classroom will intensify maladaptive behavior and promote his isolation from peers. What is the best option for this child? a. do not implement increased computer use in the class at this time b. emphasize handwriting over this marking period and determine the students progress c. include specific strategies in the plan to incorporate variable use and avoid isolation d. allow student to use the computer only in a group setting with other students.

c. include specific strategies in the plan to incorporate variable use and avoid isolation Individuals with autism may develop habits, routines, or inflexible behaviors when presented with the consistent repetition of computer based activities. However, computers can be helpful for word processing tasks. The OT can recommend activities and environments to promote participation through computer use. A and B are inappropriate because the OT should carefully plan the technology activities to meet the unique needs of the individual. D does not address any specific individualization

an individual with Alzheimer's disease has limitations and shoulder ROM. The OT goal for this individual is to improve active shoulder range of motion in order to resume self-care activities. which strategy would be the most effective and actively engaging the individual? a. telling the individual to perform repetitions of activity UE ROM independently. b. training the individual to use long-handed adaptive devices to compensate for decreased shoulder motion c. incorporating simple, familiar activities such as hanging up clothing or catching a ball d. instructing the caregivers to perform PROM exercises on the individual

c. incorporating simple, familiar activities such as hanging up clothing or catching a ball The primary goal for OT for clients with this disease is to maximize QOL and engagement in occupation. A might not be effective because it can be confusing and will likely be forgotten. B would not increase active shoulder motion and would be confusing. D would not lead to improvement in AROM.

an individual with ALS swims 3 times a week to maximize strength and endurance. initially able to swim for only 10 minutes, the individual is now able to swim for 20. what is the next step to take to increase endurance? a. continue the program of swimming 20 min, three times per week b. decrease swimming frequency to two times a week c. increase swimming time to 25 minutes or to tolerance d. provide AE that will enable the individual to swim using less energy.

c. increase swimming time to 25 minutes or to tolerance Although ALS is a progressive degenerative disease, improvements in strength and endurance are possible if the individual was not previously functioning at maximum capacity. C increases duration while recognizing importance of fatigue.

During a mealtime feeding eval of an individual who sustained a TBI, the OT observes that the client picks up his fork but does not attempt to eat, though the food is accessible and he has expressed that he is hungry. In what area is the individual MOST likely expressing problems? a. attention b. concentration c. initiation d. apraxia

c. initiation an individual with initiation problems may be able to plan or carry out activities but may be unable to begin until prompted by another person. Problems with attention, concentration, or apraxia would be evidenced if the eating activity were performed incorrectly or incompletely.

A very confused long-term care facility resident is frequently found in the rooms of other residents in the middle of the night. What of the following environmental adaptations would MOST effectively prevent wandering? a. apply wrist restraints after the client has fallen asleep b. keep hallways clear of obstructions to prevent injury c. install an alarm on the clients door d. more the individuals room close to the nurses station

c. install an alarm on the client's door This would be the preferred intervention. Other strategies include, regular exercise, putting up stop signs, and diversion. Moving the individuals room would likely add to confusion. Keeping hallways clear results in safer environments but does not stop the patient from wandering. Applying restraints would be the last choice because it interferes with the clients functional independence.

an OT has been asked to design a group to promote self-efficacy for women in a domestic violence shelter. The task part of the activity involves the women giving each other facials and manicures. How should the OT follow up the activity for these women, who are functioning at a cooperative group level? a. identify the steps for planning a fashion show b. have the participants take glamour shots of each other c. institute a participant led discussion of how the activity made them feel and identification of reasons they deserve to feel so good. d. solicit program feedback from the group members and encourage them to plan how to implement this type of group again

c. institute a participant led discussion of how the activity made them feel and identification of reasons they deserve to feel so good. at the cooperative group level, members are encouraged to identify and gratify each others social emotional needs in conjunction with task accomplishment. The therapist acts primarily as a role of an advisor and may not be present at all meetings. D involves the therapist more intensely. A and B could be good follow up actions but they do not incorporate the social support and involves the therapist as a leader instead of advisor.

an OT evaling a child notices a bruise on the childs shoulder that looks like an adults handprint and fingerprints. Which is the most critical step for the OT to take? a. discuss this with the family member who picks up the child b. observe for additional injuries c. make a report to appropriate authorities d. avoid becoming involved in personal family matters

c. make a report to appropriate authorities OTs are mandated federal reporters. The other answers delay or prevent proper assistance to the child or family.

An OTA frequently administers the Allen Cognitive Level Test and then discusses it with the supervising OT. Which of the following MOST accurately describes the OTs role during these discussions? a. determine the OTAs service competency b. collect data on the patient's performance c. interpret the results based on data collection d. develop the treatment plan

c. interpret the results based on data collected An OT initiates and directs the screening, eval, and re-eval process and analyzes and interoperates the information in accordance with federal and state law. Service competency would need to be established prior to the OTA administering the eval. Collecting data is the responsibility of the OTA in the scenario. Developing the treatment plan would follow the analysis of the data.

A team composed of an experienced OT and experienced OTA have worked closely together for 5 years. Of the following, which tasks must be performed by the OT? a. activity programming, environmental adaptations, and caregiver and staff education b. ADL training and running feeding and leisure activity groups c. interpreting results of assessments for the purposes of treatment planning d. positioning, providing adaptive devices, and instructing in the use of orthotic devices.

c. interpreting results of assessments for purposes of treatment planning An OT initiates and directs the screening, eval, and reeval. All other answers may be performed by the OTA in a long term care facility

An OT interviewing an individual diagnosed with a TBI about his ADL performance realizes that the client is confabulating. What is the best way for the OT to gather the needed information? a. complete the interview using close-ended questions b. stop the interview and complete it the next day c. interview a reliable informant instead of the individual d. administer a written questionnaire using a checklist format

c. interview a reliable informant instead of the individual Individuals who are unable to think clearly, or who are experiencing memory loss often confabulate, or fill in the missing gaps with erroneous information. When this occurs, it is necessary to get reliable information from another source. A would not necessarily yield more correct information. D is effective for individuals with requisite cognitive skills, but the individual is unlikely to do any better with this method. There is no reason to believe the patient would be better the next day.

the parents of an adolescent with a mental health disorder are meeting with the OT in an outpatient center to plan areas for eval. What will the OT most likely want to include as part of the eval? a. assessment of the childs visual motor integration related to schoolwork performance b. assessment to measure childs cognitive and visual perceptual skills c. interview about challenges and successes within the familys current daily activities and routines d. involve other members of the school team for the purpose of carryover

c. interview about challenges and successes within the familys current daily activities and routines this answer is correct because when a child has a mental illness, the typical roles and family occupations of all family members are challenged. Answers A and B are incorrect as there is no evidence in this question to drive specific evals in either of these areas. D is incorrect as parents choose resources to help them parent their children with or without disabilities, and without further indication that the contact with childs school will help parents achieve their goals, this action is inappropriate

An OT is evaluating a school age child with CP. Which procedure should DEFINITELY be included in the process? a. screening of gross and fine motor performance b. the COPM c. Interview with the childs parent d. BOT-2

c. interview with the childs parent. Interviews with parents are an essential part of every eval regardless of setting. They enable the OT to gather vital info about the clients expectation related to OTs involvement. Answer A is inappropriate as the childs diagnosis already suggests that assessment of fine and gross motor is likely, and further the info from parent and teacher will help the OT address this concern. B and D suggest specific eval tools that the therapist may use with some children but they are not essential for ALL students.

When an OT selects a standardized test to assess a child, what can the therapist OT assume about the test? a. it is valid b. it has normative data c. it has a standard format d. it is reliable

c. it has a standard format Standardized tests include a manual, fixed number of items, and fixed protocol for administration. The other answers may often be provided with standardized tests but they are not assumed to be part of the test.

An older adult has been hospitalized with a diagnosis of major depressive disorder following an overdose of sleeping pills. In obtaining an occupational profile, the OT discovers the patient has knitted and crocheted and enjoyed cooking. What activity should the OT recommend FIRST to achieve the goals of increasing a sense of self confidence? a. crocheting a sweater for her teenage granddaughter b. making spaghetti and garlic bread for her husband c. knitting a small hat for her newborn grandson d. planning meals for the week in anticipation of discharge.

c. knitting a small hat for her newborn grandson all the choices are client centered and could contribute to a sense of self-esteem and positive outlook toward the future. However, the small hat is the simplest and most short-term activity listed. It will not require new learning. All other activities could be appropriate as the patient's symptoms lessen.

The teacher notes that a student with quadriplegia frequently slumps to the side when sitting in a wheelchair. What would the OT MOST likely recommend to enable the child to maintain optimal wheelchair positioning for seat work? a. reclining wheelchair b. an arm trough c. lateral trunk supports d. lateral pelvic supports

c. lateral trunk supports these would help maintain correct alignment of the pelvis and trunk. additional seating supports should be used as needed to improve posture, restrict abnormal movements, and promote head control and voluntary movement of the limbs. Answer A would shift the child's weight posteriorly and not prevent lateral shifting. Answer B would contribute to lateral shifting. Lateral pelvic supports would be too low to prevent the trunk from shifting.

A homemaker with weak grip strength wishes to prepare a muffin mix but cannot open the bag. Which of the following would the OT MOST likely recommend? a. a hand powered mixer b. loop handle scissors c. an electric knife d. prepare slice cookies instead of muffins

c. loop handled scissors This device would be much safer than an electric knife. Answer A does not address how the patient will open the bag, and answer D is not coincide with the client's desire to make muffins.

and OT is working with a group of 3-year-old children in a preschool setting. The OT has decided to initiate a group that facilitates symbolic play. which would be the most appropriate activity to introduce first? a. scissors and cutting activities with construction paper b. jump rope games with complex footwork c. Make believe groups with stuffed animals and imaginary friends d. building towers with blocks that resemble pictures in books

c. make believe groups with stuffed animals and imaginary friends Symbolic play is imaginative in nature. A would most likely facilitate fine motor skills. B would facilitate gross motor skills. D would resemble constructive play.

A patient with poor visual acuity is about to be discharged after completing a rehab program following a total hip replacement. The MOST appropriate environmental adaptation to ensure the individual can go up and down stairs safely is a. installing a stair glide b. mounting handrails on both sides of the steps c. marking the end of each step with highlighting contrast tape. d. instructing the patient to only take one step at a time when going up or down

c. marking the end of each step with high contrast tape. Installing a stair glide or handrails are a more costly adaptation that do not address the problems of decreased visual acuity. Answer D may cause the individual to be unnecessarily slow, and does not address the issue of poor visual acuity.

an OT is assessing an individual with dysphagia. Which of the following should the OT address FIRST? a. jaw pain and tooth-grinding habits b. cranial nerve function assessment c. mental status, oral structures, and motor control of the head d. muscle length control via finger-to-nose tests

c. mental status, oral structures, and motor control of the head A is most related to an evaluation of TMJ. B would by typically performed by a physician or neurologist. D would be related to dysmetria and not dysphagia.

which scar management technique is most appropriate to utilize during the rehab phase of treatment with an individual who sustained a partial thickness burn 6 months ago? a. preventing scar development through a static use of orthosis b. controlling edema to prevent loss of ROM c. minimizing scar hypertrophy through compression garments and proper skin care d. promoting self-care skills in order to resume the role of the homemaker

c. minimizing scar hypertrophy through compression garments and proper skin care ideally clients should be fitted with custom garments not later than 3 weeks after wound healing. These should be work 23 hours per day and removed only for bathing, massage, skin care, or sexual activity. A is typically performed during the surgical phase. B and D are not goals directly related during scar management and are more during acute care intervention

an individual requires 40-50% of assistance from the OT to transfer from a wheelchair to a slidingboard and to lift his legs from the wheelchair into and out of a bathtub. Accordint to the FIM levels, the individual will require which level of assistance to complete the transfer? a. total b. min c. mod d. max

c. mod moderate assistance is defined as having the ability to complete the task with supervision and queuing while requiring physical assistance for approximately 50% of the task. The individual who requires men assist is able to complete the task 75% or more.

an OT receives a referral for an infant in the NICE whose mother has a history of drug use during pregnancy. What is the first action the therapist should take? a. determine the mothers current drug use, medical status, parental involvement and support systems b. recommend a social work referral, provide support and community program info, and refer to the Department of Human Services c. modify the environment to protect the infant from excessive and or inappropriate sensory stim prior to direct intervention d. assess infants motor and behavioral skills and begin gentle handling to promote positive sensory experiences

c. modify the environment to protect the infant from excessive and or inappropriate sensory stim prior to direct intervention C best represents a developmentally supportive practice that views protecting the newborn from excessive stimuli as urgently important. A and B are important in determining appropriate treatment plans, they are team managed activities that are not priority of just the OT

An individual who recently experienced an MI has been evaled by the OT and is currently receiving intervention. A primary concern of the cardiac team is to prevent overexertion in therapy. What is the BEST way to address this concern? a. teach the client independent med management strategies b. retrain the client in ADLs to prevent excessive strain c. monitor the clients HR and BP regularly throughout intervention d. teach EC techniques to decrease stress

c. monitor the clients HR and BP regularly throughout intervention Monitoring occurs constantly during the acute phases of cardiac rehab. The other answers all pertain to typical OT interventions for cardiac rehab, however answer C is more preventative for overexertion.

An individual is learning how to perform transfers into a bathtub 2 weeks after a TKA, but is still unable to extend or flex the knee more than 20 degrees. Which of the following would MOST likely allow for safe tub transfers? a. wait another 2-4 weeks, because this activity is contraindicated until 4 to 6 weeks after surgery b. use a handrail attached to the side of the tub c. obtain a tub transfer bench and leg lifter d. use a low kitchen stool with rubber tips

c. obtain a tub transfer bench and leg lifter Answer A is not considered a standard course of treatment. B would assist with transfers but would not address the limitation of the knee movement. D would not be considered a safe selection for transfer training.

a home health OT has received a referral for an individual with medicare coverage. Which one of the following actions must occur before the therapist can initiate eval or treatment? a. identify the deficits that impair functional abilities b. establish short and long term goals c. obtain physicians POC identifying services to be provided d. collect the individuals history of current illness.

c. obtain physicians POC identifying services to be provided Within the home care setting, the therapist must have a physicians order that identifies the services to be provided. After eval, A, B, and D can be determined.

The OT and client have agreed on a baking activity to work on decision-making skills, but when the time comes, the individual is reluctant to participate. Which of the following approaches would be MOST appropriate? a. premeasure all the dry and liquid ingredients b. use a recipe with no more than three steps c. offer a choice of slice and bake cookies or a cake mix d. create a checklist to use as each ingredient is added

c. offer a choice of slice and bake cookies or cake mix Strategies to address impairment in decision making skills include limiting the number of options. Answer A could save time and be a useful strategy for an individual with a short attention span. Limiting number of steps and creating checklists could be useful for individuals with memory deficits.

a 10 y/o with a diagnosis of athetoid CP would like to be able to dress herself independently. Which clothing features could the OT recommend that would be most useful in facilitating self-dressing? a. mini T shirts made of elasticized fabric b. dresses with side zippers and zipper pulls c. oversized t-shirts and elastic top pants d. front opening shirts with snaps or buttons

c. oversized t-shirts with elastic top pants for a child who experiences difficulty with coordination, loose clothing is preferred to tight fitting garments. Elastic waist bands are more functional than zippers or buttons.

the OT is working with an older adult diagnosed with diabetes and peripheral neuropathy following an LE amputation. The client has indicated interest in doing a craft activity while increasing standing tolerance with the new prosthesis. Activities of interest include needlepoint, scrapbooking, woodworking, and painting. Which activity is the most appropriate for the OT to recommend first? a. scrapbooking b. woodworking c. painting d. needlework

c. painting this is the safest activity. Needles would not be a good choice, splinters from woodworking would be a risk factor and paper cuts would be a risk factor with scrapbooking.

An OT is working with an individual with a visual field deficit who neglects the left side of the plate when eating. Based on the clients interests, the OT has the client working on a mosaic activity to address this deficit. When documenting the clients performance in the objective part of the SOAP note, which example is most likely to justify reimbursement to a third party payer? a. patient has progressed from moderate assist level to occasional verbal cues for left neglect while eating b. patient would benefit from continued skilled OT in order to achieve preadmission level of independence. c. patient required occasional tactile cues to look to the left to compensate for visual field deficit, and was able to acquire and correctly place all 50 tiles from the container placed on the left side. d. patient demos ability to pick up half inch tiles using pincer grasp and accurately follows a simple repeating pattern for 10 minutes with occasional verbal cuing

c. patient required occasional tactile cues to look to the left to compensate for visual field deficit, and was able to acquire and correctly place all 50 tiles from the container placed on the left side. It is important to document that a client is learning or gaining the skill. C includes level of assistance and skill contribution of the therapist, measurable data, and specific component being addressed. A is a summary of the patients progress that belongs in the assessment portion of the note, as do references of progress such as B. D incorporates appropriate measurable data, built does not address the visual field deficit which is the primary focus of this question.

What would an OT who is fitting a first grade child for a desk and chair recommend to promote optimal hand function during classroom seat work? a. placing the writing surface at a level slightly below the childs elbow b. providing a wrist weight for writing activities c. placing the writing surface at a level slightly above the childs elbow stabilizing the childs trunk against the seat back.

c. place writing surface at level slightly above the childs elbow. This promotes use of shoulder abduction and IR of the arms and this position enables proximal and optimal positioning of the hand for skilled activity. Answer A is incorrect as a surface lower than the elbow promotes use of body flexion. There is no indication that the child requires any external weight for hand function. Stabilizing the trunk is incorrect because unless the pelvis is stabilized, arm movements may still be compromised.

a 15 month old child is demoing motor performance similar to that of an 8 month old child. In which activities would the OT most likely engage the child to elicit developing protective reactions? a. play while prone on a mat, with child reaching to grab toys suspended forward and out to childs sides b. play while floor sitting with toys placed on the floor in front of child, or suspended for childs reach to front c. play while floor sitting with toys placed on the floor to childs sides, or suspended for childs reach out to the sides d. play while in supported quadruped position, with child reaching for toys placed on floor more than arms reach from childs body

c. play floor sitting with toys placed on the floor in front of the child, or suspended for childs reach to front Protective extension is an automatic response in which the childs arms extend and to support him or her to prevent falling. Forward protective extension develops by 8 months in typical children and begins progressing to side protective extension. Answer A is incorrect as a child with this level of skill has already surpassed that level.

An OT is working with an individual who is at risk for aspiration during swallowing. The BEST way for the OT to assist the client during feeding is for the OT to: a. place the client in slight neck extension b. recline the client in bed c. position the client by using a side or front hold d. facilitate the client to move into full neck flexion

c. position the client by using a side or front hold using this hold position may be beneficial in decreasing aspirations in people with delayed pharyngeal swallow and reduced airway closure. D may make it too difficult to swallow and A may increase the likelihood of aspiration. A sitting position is preferred to lying down.

an OT is positioning a child with low muscle tone and postural instability into a prone stander so she can participate in activities in the art studio. The child rapidly fatigues. How can the OT best adjust the stander to promote head righting and enable to student to participate in the class? a. place the child in prone on the floor b. position the stander at 45 degrees from the floor c. position the stander at 75-80 degrees from the floor d. position the child in an upright prone or supine stander

c. position the stander at 75-80 degrees from the floor by adjusting it to the nearest vertical, the child can work on head righting. The child cannot maintain a righted head in full prone may be more successful at a less upright position. A is incorrect because the head and neck are doing the most work against gravity. The gravitational demand on the child is greater at 45 than 75 making it more difficult. D is incorrect because the head and neck work less against gravity in an upright position.

An OT is working with a group to develop grocery shopping skills. In addition to classroom teaching and homework exercises, what is the MOST important component to include? a. participating in simulated grocery shopping activities b. finding the lowest priced items c. practicing in an actual grocery store d. knowing where and how to find items

c. practicing in an actual grocery store. This is important because evidence has shown that situated learning is more effective than simulation. Answer B and D are specific skills that should be taught during the classroom teaching modules.

The OT is working with an individual with mental illness and cognitive deficits who has expressed interest in learning to cook in order to move into a less restrictive environment. The OT observes as the client prepares mac n cheese from a box. The individual is able to open the box and combine the ingredients, but is unable to turn on the heat or recognize when the food is ready. The OT decides to downgrade the activity for the next session. What is the MOST appropriate way to downgrade the activity? a. baking chicken and mashed potatoes b. assembling a PB and J c. preparing a frozen dinner d. making instant pudding

c. preparing a frozen dinner preparing mac and cheese, a hot meal is a multistep activity requiring sequencing and the use of heat. Preparing a frozen dinner requires the use of heat but has less steps. Answer A would be upgrading the activity. B and D would be downgrading the activity more than necessary.

A family and their young child are ready to transition from Part C early intervention to a preschool program. Which option will MOST likely support the family in the process? a. home-based OT advises family that services and paperwork differ in preschool so they are prepared to expect changes. b. home based OT contacts OT at preschool agency and offers to answer any questions c. preschool staff invite the parent and child to come and visit the preschool before they end their current services. d. IFSP team ensures that all paperwork is complete and in the childs file before the anticipated transition date.

c. preschool staff invite the parent and child to come and visit the preschool before they end their current services. When staff in the preschool reaches out to families they facilitate relationship building. A is not constructive and highlights differences in the upcoming change. B is incorrect, as filling reports is a procedural task and by itself does not support collaborative relationships.

What is a PRIMARY goal for providing a hand orthosis to a child with active juvenile RA? a. inhibit hypertonus b. increase ROM c. prevent deformity d. correct deformity

c. prevent deformity A is not a characteristic of this condition. The correction of deformity and increasing ROM may be contraindicated due to the disease.

an OT is developing a prevention program for a local company with many workers who have developed tendonitis, nerve compression syndromes, and myofascial pain. A primary focus of the prevention program should be to: a. educate company workers about osteoarthritic conditions b. develop a pamphlet on peripheral vascular disease c. produce a flyer identifying risk factors for cumulative trauma disorders d. offer screening for neuroma-related conditions

c. produce a flyer identifying risk factors for cumulative trauma disorders A is typically presented by stiffness, redness, and edema and are not caused by cumulative trauma. B is unrelated to the diagnosis in question and D is specifically related to an amputation, nerve injury, or suture.

An OT practitioner wishes to assess outcomes for life skills training services provided to individuals at a shelter for women who experience domestic violence. Which of the following methods would be the MOST comprehensive method for obtaining this information? a. final eval of each client involved b. client satisfaction survey c. program eval d. utilization review

c. program evaluation This is an outcome-monitoring system that reflects the results of services on consumers by defining and reviewing the outcomes of care. Final evaluations of clients involved in the program and client satisfaction surveys may both be components of the program eval. Utilization review evaluates the care that is provided to ensure services were provided in a necessary, appropriate, and efficient manner.

The mother of a 3 y/o with spastic quadriplegia wants her son to walk independently around the house. The child is not yet ready to achieve this goal; however, his mobility would be helpful to the mother who is finding her child difficulty to lift. What is the BEST way to assist in development of family centered intervention? a. support and work on the parent's goal as she has stated it b. suggest an alternate goal of improving sitting balance for play c. propose a modified goal that still meets the parent's needs d. include the child in the goal setting process.

c. propose a modified goal that still meets the parents needs developing an alternate goal is a family centered intervention that would address both the parents needs and the childs abilities. A would not meet the child needs for developmentally appropriate intervention. B and D do not help the concerns identified by the family.

an OT provides consultation to a school district to help maximize the learning environment for students with ADHD. Which environmental adaptation recommendation would best promote optimal learning for students with ADHD without affecting other students? a. use dim lighting and reduce glare by turning down lights b. remove all posters and visual aids to reduce visual distractions c. provide a screen to reduce peripheral visual stimuli d. restructure classroom activities into a series of short term tasks

c. provide a screen to reduce peripheral visual stimuli although all the answers given describe techniques that could assist the student, this is most appropriate. The others could have a negative impact on other students ability to learn

the goal for an elderly individual with LE weakness is to be independent with bathing, but this requires the tub to be more accessible to the individual, who uses a walker. which environmental adaptations would the OT MOST likely recommend to achieve this? a. tear out the tub and install a walk in shower b. install shower doors for privacy and for support getting in and out of the tub c. provide a transfer tub bench and install grab bars d. place nonskid decals in the tub and mats on the floor to prevent slipping on the wet floor.

c. provide a transfer tub bench and install grab bars The best adaptation to achieve access to the tub would be providing the individual with a transfer tub bench and grab bars for safety. A would be an unreasonable expense. Shower doors would make it more difficult to transfer into the tub.

While performing endurance training activities, an individual on a cardiac rehab unit begins to slow down, using progressively smaller movements to perform the activity. Which of the following is the MOST appropriate action for the OT to take? a. stop the activity immediately b. upgrade the activity for the next session c. reassess the patients vitals and consider modifying the activity to make it less challenging d. replace the activity with isometric exercises

c. reassess the patients vitals and consider modifying the activity to make it less challenging The OT should recognize subtle signs of fatigue such as frustration, slowing down, hurrying to finish, and substitution movements. Stopping the activity is necessary if the individual experiences symptoms such as dyspnea, chest pain, light headedness, or diaphoresis. The activity should be upgraded when the individual is able to perform the activity without cardiac signs or signs of fatigue. Isometric exercises which interfere with blood flow are contraindicated for this population.

An OT in a state psych hospital, using the recovery model, is working with a group of older adults with severe and persistent mental illness. They have been hospitalized for decades and will be moved into community living when the hospital closes in several months. Which of the following discharge planning activities is most appropriate for these individuals? a. using a map, show them where the new house will be in comparison to where the hospital is b. allow them to select their roommates c. provide catalogs from which they can select bedspreads and curtains d. provide collage activity to explore/express feelings about moving.

c. provide catalogs from which they can select bedspreads and curtains interventions centered on the recovery model provide individuals with choice, support them in achieving self-identified goals, address all aspects of life, and empower them in decision making. Answer A is too advanced for an individual at this level. Considerable judgement and safety considerations must go into roommate selection. D could facilitate exploration and expression of feelings but does not address choice or empowerment of the individuals.

and OT is working on keyboarding activities with an individual with asymmetrical muscle tone who keeps falling to the side while sitting in a wheelchair. which adaptation would most effectively stabilize the upper body and midline position? a. change to a reclining wheelchair b. use an arm trough c. provide lateral trunk support d. attach lateral pelvic support

c. provide lateral trunk support This will help maintain correct alignment of the pelvis and drunk. Answer A is incorrect because the individuals weight would be posterior but would not prevent lateral shift. B may help maintain a more centered position of the trunk but the weight of the affected extremity would result in instability of the shoulder. D would be too low to stabilize the trunk

an OT consulting in a supervised living environment is teaching the residential staff strategies to help minimize the effects of hallucinations for residents with schizophrenia. What is the most effective strategy for the OT to teach the staff? a. avoid disturbing persons while they are experiencing a hallucination b. move individuals experiencing hallucinations to areas where they can be isolated c. provide meaningful activities that will divert attention from the symptoms d. move the individuals experiencing hallucinations to more stimulating environments

c. provide meaningful activities that will divert attention from the symptoms this is the best approach because people who display positive symptoms benefit from activities that divert attention from the symptoms. In the process, the individuals can learn self-help coping strategies to minimize intrusiveness of positive symptoms. A would have no benefit to reducing attention to the hallucination. B is not recommended because interpersonal contact can be beneficial for reinforcing reality. D can have the effect of increasing hallucinations if they find the environmental stimuli too stressful.

An OT practitioner is working in a sheltered workshop with adults with ID functioning at Allens Cognitive level 4. The upcoming job requires an assembly-line approach. Which of the following is the BEST method for introducing an assembly activity to this population? a. provide simple, repetitive tasks b. visually and verbally demo a 2 or 3 step task c. provide one-step directions and samples for each individual to duplicate d. provide written directions for individuals to follow.

c. provide one-step directions and samples for individuals to duplicate Individuals functioning at this level have difficulty making corrections and require visual demonstration and limited instruction to one step at a time. Individuals at level 3 are capable of using their hands for simple repetitive tasks. Level 5 can generally perform a task involving several familiar steps and one new one. Level 6 individuals can anticipate errors and plan ways to avoid them. These individuals could follow written instruction.

A child with behavioral problems has difficulty with peer interactions. The OT's intervention plan is MOST likely based on which approach? a. provide small group occupation-based activities in an authoritarian environment with clear expectations. b. begin with 1:1 occupation-based activities to develop a child's social skills, then introduce group activities. c. provide small group occupation based activities that encourage exploration and interaction d. provide small group occupation-based activities with rules to define acceptable play guidelines

c. provide small group occupation based activities that encourage exploration and interaction It is unlikely that the child will initiate and develop social interaction in an environment that inhibits independence, interaction, and exploration.

Which of the following are the MOST appropriate interventions for the OT to plan in treating a person following a total knee replacement? a. using ROM and strengthening for UEs b. providing ROM and strengthening activities for LE c. providing ADL training in use of AE techniques for LE dressing and transfers d. administering an eval of homemaking activities

c. providing ADL training in use of AE techniques for LE dressing and transfers Decreased knee movement may require the person to use AE for transfers and LE dressing. Answer A improving strength of UEs would not necessarily be an aspect of treatment for all individuals. Answer B would be an area that the physical therapist would primarily address. An eval would not be an intervention.

An OT is working with a 12 y/o whose UE physical disabilities limit the ability to independently read books while lying in bed. What would be the BEST adaptation for this child who enjoys reading books before sleep? a. teach the child how to use a book holder and mouth stick b. fabricate an adapted book holder for use in his bed c. purchase talking books related to the students areas of interest d. have the childs parent read a book the child selects each night

c. purchase talking books related to the students area of interest Accommodations to enable reading include the use of commercially available talking photo albums, specific software, e-text, audio recorders, screen readers, and multimedia. A is not a practical strategy while the child lays in bed. B is incorrect because the child has limited UE ability to turn pages. D does not address the Childs desire to independently read.

an OT providing accessibility consultation services to a local library finds a reference room with a doorway that has a threshold height of 2 inches, making wheelchair access difficult. Which of the following recommendations would be most appropriate? a. hang signage to increase awareness of the threshold b. provide a throw rug that covers the threshold c. remove the threshold altogether d. install a ramp over the threshold

c. remove the threshold altogether this would be the simplest and safest solution. A throw rug is more dangerous, signage doesnt fix the problem, and installing a ramp is likely more expensive and unnecessary if the ramp can be removed.

an OT is attempting to determine the best model to support the idea related to a falls prevention program in a well elderly setting. During the planning, the OT reviews the ethical principles of the Belmot report to be sure which of the following is adhered to in relation to the participants? a. HIPPA and guidelines b. completion of annual review to maintain approval of participants c. respect, beneficiene, and justice d. The neuremburg code, confidentiality, and privacy

c. respect, beneficence, and justice the nuremberg code was pat of the impetus for the development of the belmont commission.

an individual with a diagnosis of lung cancer is admitted to a rehab unit following a lobectomy. The OT should design and implement a treatment plan that focuses on: a. palliative care b. prevention and changing habits c. restoration of function d. symptom management

c. restoration of function individuals admitted to a rehab unit need to be able to participate in 3 hours of therapy a day. Therefore the goals need to align with that. A is provided to individuals in the advanced stages of cancer and focus on comfort and symptom management. B is for educating people about risky behaviors and focus on developing healthy habits.

An OT observes a child move from a completely prone position to a prone-on-elbow position. In reporting the childs progress, the OT documents the child is gaining control in the midline position through the development of what type of reactions? a. amphibian b. prehensile c. righting d. equilibrium

c. righting righting reactions are designed to align the head with gravity. A occurs when the child is in prone position, lifting the pelvis causes flexion of the arm and leg on the same side. This appears at 6 months and is maintained. Prehensile reactions are referred to grasping patterns and reach. Equilibrium reactions develop after righting reactions and allow the child to maintain standing or walking.

in a social skills group, members have learned about social skills and their components, and the OT has spent time in modeling and demonstrating ways to perform social skills. based on the social skills training approach, which of the following would be the NEXT step in the process of developing social skills? a. self evaluating ones social behavior b. independently practicing in real life situations c. role playing social situations d. providing feedback on the clients hygiene

c. role playing social situations this highly structured approach to teaching skills involves breaking skills into discrete steps, rehearsing the behaviors through role plays, and providing feed back. The first step in social skills training is to identify interactional challenges, followed by teaching skills, practicing and developing these skills, then feedback and reinforcement of the skills. Then answer C occurs followed by independent practice and feedback.

a young adult on the autism spectrum has obtained a job at a coffee shop. Social skills are good, as evidenced by getting along well with the boss and coworkers, but the client has difficulty noticing when orders are ready, when areas need to be tidied up, and when the coffee machine needs to be refilled. What is the most appropriate recommendation to address potential sensory issues? a. provide noise cancelling headphones b. move to a quieter work area c. rotate frequently from one station to another d. provide a job coach

c. rotate frequently from one station to another this individuals behavior suggests that he has low registration. People with low registration tend to miss inut that others take in. They may be slow to respond or require repetition and cues. Yet, these individuals are highly flexible because they are not bothered by sensory stim and can typically manage distracting environments quite well. A and B would be appropriate for individuals with sensory sensitivity. Although a job coach could be appropriate, it does not address sensory issues.

A 16 y/o boy who had a TBI 5 years ago is ready to begin shaving, but has difficulty as a result of limited ROM in his shoulders and elbows. Which is the BEST adaptation for him to use? a. electric razor attached to a universal cuff b. safety razor with built up handle c. safety razor with extended handle d. safety razor with attached universal cuff

c. safety razor with extended handle This allows the individual to overcome limited shoulder and elbow ROM. Answers C and D would benefit an individual who is unable to grasp a razor, but would not enable to individual to reach his face to have. Answer B would benefit an individual with limited finger flexion or strength.

An individual with TBI is impulsive during feeding. which is exemplified by placing too much food in the mouth at one time. what method would most effectively develop safer eating habits? a. cut the food into smaller pieces prior to serving b. count to 10 between bites of food c. set down the utensil til the mouth is cleared d. serve the food in seperate containers on the meal tray.

c. set down the utensil til the mouth is cleared this method of imposing a restriction on the behavior can develop into an established routine for pacing. A and D are examples of environmental adaptations not habit development. B will not assure that the mouth has been cleared in time for the next bite.

a young child with hypertonicity is unable to bring his hands to midline to reach for a toy while in supine and sitting positions. What is the best position to use to reduce the effects of abnormal patterns and facilitate midline grasp? a. supported standing b. prone c. side lying d. quadruped

c. side lying this position reduces the influence of tonic labyrinthine reflexes, extensor tone, and gravity, all of which make protraction of the shoulders and forward reach difficult. A is incorrect because standing would not reduce extensor tone. B and D incorrect because in the prone position, the UEs are involved in weight bearing.

When planning intervention to build self-esteem for an individual demoing positive symptoms of schizophrenia, what is the OT most likely to use as a treatment format? a. projective media to facilitate expression of feelings b. work in an isolated area away from the group c. simple and highly structured activities d. discussions about the individuals delusions

c. simple and highly structured activities Structured tasks provide habit training, diversion, coping skills, and time management training. Answer A is not most useful for encouraging expression of feelings. It may be appropriate to separate individuals who are violent or unable to tolerate the presence of others,but this would not be desirable as part of the regular group routine. D is undesirable because it is likely to reinforce them.

A homeless individual with mental illness has recently begun coming to a shelter. which type of group is most likely to engage this individual? a. highly structured craft group b. volunteer activity group c. simple meal prep group d. social skills group

c. simple meal prep group obtaining food is a basic survival need and is often the focus of a homeless individuals day. Craft groups may be useful in developing healthy leisure interests. Volunteer activities can help develop work habits. Ans Social skills groups address interpersonal skills necessary for living in society.

an OT is considering seating options for a student with extensor tone triggered through LE WBing. What should the therapist recommend to inhibit the childs extensor tone and enable his participation during seat work in the classroom? a. provide lateral trunk supports and a headrest during all seat work periods b. install a chest seat belt on the chair to maintain upper body support c. sit on a wedge-shaped seat cushion that is higher in the front and includes an abduction post between the knees c. use a wheelchair with custom-molded seat cushion and lap tray for all seat work.

c. sit on a wedge-shaped seat cushion that is higher in the front and includes an abduction post between the knees. The angle of the hip joint is key to limiting extensor tone while sitting. Children with extensor tone may have a reduction in muscle tone with less than 90 of hip flexion combined with hip abduction. A is incorrect because the trunk and lateral supports prevent side moment only. A seat belt promotes stability above the pelvis to reduce sliding upward or forward. B would be addressed by following the support of the childs pelvis. Although answer D provides individualized positioning, it would limit students inclusion with peers in the classroom.

An OT working at a veterans administration hospital has been asked to participate in developing a new program for individuals with PTSD. the PRIMARY focus of treatment planning and intervention will most likely be on which of the following areas? a. pain management and balance b. memory and concentration c. sleep and emotional regulation d. communication and social participation

c. sleep and emotional regulation individuals with PTSD have been exposed to traumatic events and persistently have flashbacks or dreams. The DSM-5 identifies five areas of persistent symptoms including difficulty following or staying asleep, irritability or outbursts of anger, difficulty concentrating, hypervigilance, exaggerated startle response. problems with concentration may occur, but not necessarily with memory, pain, or balance. social participation may be impacted but the ability to communicate is not typically an issue.

In order for an individual sitting in a wheelchair to achieve maximal postural positioning, the OT should position the individuals pelvis in what position? a. moderate posterior tilt b. neutral c. slight anterior tilt d. slight posterior tilt

c. slight anterior tilt generally after the acute onset of a disability or prolonged time spent in bed, a client assumes a posterior pelvic tilt. In turn this posture moves the center of mass back towards the buttocks. The other answers would not be conducive to maximal postural positioning.

The OT is planning to review kitchen activities with an individual who underwent a total hip replacement and is to return home tomorrow. Because the cart says that this person is TTWB status, how will the weight bearing restriction impact participation in kitchen related activities? a. the person should perform all kitchen activities from a sitting position b. standing kitchen activities can be performed by placing about 50% weight on the affected leg. c. standing kitchen activities can be performed by placing the majority of weight through both arms, using toes for balance (about 10% of weight) d. perform all standing activities without restriction

c. standing kitchen activities can be performed by placing the majority of weight through both arms, using toes for balance (about 10% of weight) Answer A would reflect NWB. B would reflect PWB, and D would be FWB.

An OT is planning to begin work on self care activities with an individual who recently had a TBI and who has cognitive and visual perceptual impairments. What is the MOST effective way for the OT to present directions during the activity? a. tell the patient about the activity that he will be working on and ask him to begin b. rely on facial expressions and body gestures, rather than words, to get the patient started. c. state simple, concrete directions allowing time for delayed responses. . d. provide a written list of steps with pictures for the patient to follow

c. state simple, concrete directions allowing time for delayed responses Most people with TBI experience some degree of difficulty processing external info from the environment. Answer A would require initiation and abstract thinking. B and C rely on gestures and providing written documentation which may not be effective if visual perceptual impairments are evident.

an individual has been referred to OT following UE injury resulting in partial paralysis. Which of the following assessments would be most important for an OT guided by a biomechanical FOR? a. AE needs b. tone, reflex development, automatic reactions c. strength, ROM, and coordination d. habits, roles, and values

c. strength, ROM, and coordination A represents the rehabilitative approach which emphasizes independence. B demos the NDT approach for individuals with CNS function. D emphasizes MOHO.

The OT who provides info and resources that are helpful to the family of a young child with multiple disabilities MOST likely includes what type of information? a. updates about their child's developmental age according to standardized test results b. explanation of how OT services differ from those provided by other teammates c. suggestions for activities in which parents can engage with their child to help the child practice developing skills d. reports of the child's behavior during therapist-implemented interventions

c. suggestions for activities in which parents can engage with their child to help the child practice developing skills Parents typically hope to receive recommendations for activities that help the child play. For most pediatric standardized tests, developmental age scores for children with multiple disabilities highlights their limitations. Answers B and D may be provided to families at some point during the intervention process, yet they are not relevant to the needs parents generally have for information to help them support their childs development

and OT is working with an individual who has dysphagia. what should the OT anticipate that the individual will have difficulty doing? a. coordinating the two sides of the body b. expressing him or herself orally c. swallowing crushed ice d. speaking clearly

c. swallowing crushed ice Dysphagia is the difficulty with any stage of swallowing that interferes with functional independence. A is representative of bilateral integration. B is the inability to express oneself orally, D is the inability to speak clearly.

An older adult day care facility has contracted an OT practitioner to develop programing for its clients at risk for falls. the BEST type of program to provide regular group physical activity and enhance balance would be: a. safe-transfer training b. walking c. tai chi d. gardening

c. tai chi This would be the best activity because it incorporates slow stretching and provides graduated challenge to coordinated movements which can help improve balance. Answer A is useful to teach safety precautions but would not have much impact on motor skills. Walking is a good general exercise but would not provide stretching or challenge balance. Gardening would be performed in a more stationary position.

An OT is working with a client who recently had a MI and fatigues easily. The long term goal is to be able to dress independently while applying energy conservation techniques. Which statement is the BEST example of a short-term goal? a. the patient will demo energy conservation techniques during performance of ADLs 100% of the time. b. the patient will perform LE dressing with assistance for shoes and socks c. the patient will perform LE dressing with verbal cuing for energy conservation techniques 50% of the time d. instruct the patient in use of energy conservation techniques that apply to dressing.

c. the patient will perform LE dressing with verbal cueing for energy conservation techniques 50% of the time. goals should be functional, measurable, and objective. B does not provide measurable criteria. Short term goals must relate to the long term goal. Answer D describes what the practitioner will do.

an OT student has been assigned the task of creating a reality orientation board for a dementia unit. What are the key elements to include? a. illustrated step by step instructions for the days activity b. pictures of family members now and when they were younger c. the time, date, next meal, and the weather d. a list of the individuals medications and times they should be taken

c. the time, date, next meal, and the weather this is an intervention used for adults with dementia . The other answers are all interventions that can be used to modify the task or environment with individuals with dementia, but the board for the unit should be information that applies to all residents.

an individual demos the ability to pick up a penny from a flat surface. The OT would document this as which type of prehension? a. lateral b. palmar c. tip d. spherical

c. tip the IP joint of the thumb and the DIP and PIP joint is used to pick up objects such as a pin, nail, or coin. Answer A is for holding a pen, key, or utensil. B is AKA the 3 jaw chuck for lifiting items off a flat surface or tying shoes. D is typically used for holding a ball or other round objects.

an OT in the NICU is instructing the parents to bottle feed their infant. TO facilitate suck-swallow-breathe coordination, the OT would instruct the parents to hold the baby comfortably in the breast-feeding position and do what next? a. stroke the infants cheek before feeding her a bottle b. gently touch the infants lips before introducing the bottle c. tip the bottle back to allow the infant to include a breath in her suck-swallow pattern d. gently rub the infants gums and cheek simultaneously before feeding the baby her bottle.

c. tip the bottle back to allow the infant to include a breath in her suck-swallow pattern This pattern can be coordinated by providing secure positional support, reducing stim, using a slow-flow nipple, and pacing to allow breathing. A would facilitate the rooting reflex. B would facilitate the sucking reflex but does not incorporate the breathing. D would likely be over stimulating and frustrating to the infant.

which of the following devices is required for an individual with C7 to C8 quadriplegia when performing oral hygiene activities? a. mobile arm support with utensil holder b. universal cuff c. toothbrush with built up handle d. wrist support with utensil holder

c. toothbrush with a builtup handle a person at this level has the hand strength to hold a toothbrush with a built up handle. A is for a C5 injury. C5 or C6 may use a universal cuff or wrist support.

a teen with fine motor incoordination reports difficulty with self-care. Which option would this individual find most beneficial? a. wash mitt b. spray deodorant c. toothpaste with a flip open cap d. toothbrush with a built up handle

c. toothpaste with a flip open cap C is correct because the child could manage a cap that flips much more easily. Also, toothpaste tubes with these caps are larger in diameter. A and D are good options for individuals with weak grasp. B has a small button to push which would be difficult for someone with incoordination

An individual with right unilateral neglect is able to track from the left side to the midline of the body on paper and pencil tasks. What is the BEST treatment activity to promote crossing the midline to improve writing skills? a. practice wheeling a wheelchair following a taped line on the floor b. place commonly used self care items on the left side c. trace lines across the page with the right index finger from the left to the right side d. place playing cards in a horizontal row from right to left in sequence

c. trace lines across the page with the right index finger from the left to right side B is a compensatory technique that does not involve crossing midline. C would require cueing, also the person would have difficulty accurately completing a sequence task on the affected side. When tracing the finger across the page, the individual receives the same proprioceptive and visual input. This task makes the transfer of skills easier when performing writing.

specific recommendations have been made for electronic assistive technology for an adult with muscular dystrophy. after the devices are ordered and modified as necessary, the next step in the process of implementation for the OT practitioner is: a. assess how well the whole system works b. evaluate whether the assistive technology devices match the needs of the client c. train the client in the operations of the assistive technology system and in strategies for its use d. determine whether funding is available for the recommended assistive technology

c. train the client in the operations of the assistive tech and in strategies for its use. This is essential because e of the complex nature of assistive tech. A usually occurs after training is completed during the follow-up phase. B is done earlier in the process to ensure maximum success. D is also determined prior to ordering the device.

an individual is about to be discharged to home following a brief hospitalization for substance abuse. The family asks the OT what they should do the first weekend at home. Which of the following suggestions is MOST appropriate? a. throw a welcome home party for some close friends b. go out to hear a favorite band c. go on a minivacation d. attend an AA meeting

d. attend an AA meeting an individual who has recently been discharged is likely to need support. A and B are often closely associated with access to substances and should be avoided. C would probably be too stressful for his or her first weekend home.

What is a priority in the OT intervention plan for a new student who travels to elementary school using a wheelchair? a. opportunities to practice maneuvering the wheelchair through an obstacle course made of cones. b. plans for routine wheelchair maintenance and subsequent repairs. c. training for the school bus driver to ensure that the student travels to school safely and comfortably d. plans to design and create a writing surface that can be attached to the wheelchair for students use during seated work

c. training for the school bus driver to ensure that the student travels to school safely and comfortably the OT should ensure that the driver follows specific positioning and strapping procedures for the students safety. There is no evidence that the student needs to practice maneuvering the wheelchair in a busy environment, so answer A is incorrect. Plans for routine maintenance will be an ongoing concern for the family but not an immediate concern. Answer D is not a preferred option as it does not support inclusion with peers.

An OT has been asked to develop a stress management program for clients with eating disorders. For this population in particular, which is the MOST important strategy to include? a. a physical activity such as yoga or tai chi b. diaphragmatic breathing or progressive relaxation techniques c. training to acknowledge feeling and express emotions. d. role-playing stressful situations

c. training to acknowledge feelings and express emotions this promotes development of better coping strategies, decreases inner conflicts, and limits disordered eating habits. The other options are effective coping strategies.

an OT is evaling developmental stages in a 1o month old baby who is beginning to cruise while holding onto furniture. How would the OT most likely describe the childs development? a. compensatory b. advanced c. typical d. delayed

c. typical children between the ages of 10-12 months may take first steps and walk with one hand held or behind a push toy and begin to cruise using furniture.

the OT working with an infant observes the presence of the first stage of voluntary grasp. how would the OT describe the grasp pattern exhibited in documentation? a. radial palmar grasp b. pincer grasp c. ulnar palmar grasp d. raking grasps

c. ulnar palmar grasp grasp development progresses from ulnar grasp to palmar to radial.

An OT is fabricating a resting hand orthotic device for an individual with extremely fragile skin. Which of the following areas will the OT have to inspect MOST frequently? a. metacarpal heads, pisiform, and trapezium b. volar PIP joints, medial fifth digit, and thumb MP joint c. Ulnar styloid, distal head of radius, and thumb CMC joint d. thumb PIP joint, pisiform, and hamate

c. ulnar styloid, distal head of radius, and thumb CMC joint These prominent sites are common sites for pressure. The other answers are also areas that could potentially be susceptible but are a secondary worry.

An OT arrives for a home health visit, but the individual is not feeling well so the visit is rescheduled for the following day. THe charges for the week are due that afternoon. Anticipating a follow up visit with the individual the next day (1/15/15) and reluctant to wait another 2 weeks to submit for payment, the practitioner bills for treatment using the 1/14/15 date. How would the OTs actions be described? a. acceptable because treatment has been scheduled for the next day b. acceptable if the agency she works for allows it c. unacceptable because it violates the AOTA code of ethics d. unacceptable because if the individual is still ill and unable to participate in therapy on 1/15/15, a delay of more than 1 day is unacceptable for billing purposes

c. unacceptable because it violates the AOTA code of ethics Whether or not an alternate date has been arranged, stating that services were provide on a wrong day is falsification of documentation. Answer D would only further complicate the issue.

the OT is educating the parents of a young child with sensory defensiveness regarding hair grooming. The parents report that each time they shampoo their childs hair she becomes anxious, restless, and agitated. What would the OT most likely recommend to the parents? a. wash the childs hair with water only and avoid shampoo b. have the childs hair washed thoroughly at each haircut c. use a calm voice and explain each step of the hair wash prior to doing it d. use cool water when shampooing the childs hair

c. use a calm voice and explain each step of the hair wash prior to doing it. A and B are inappropriate strategies because it would not maintain proper hygiene. Use of cool water can be contraindicated. Water should be warm to increase relaxation.

The OT is working with a patient who needs to be independent in medication management prior to discharge. What is the most effective strategy for the OT to teach the patient to remember if medication has been taken? a. establish a routine of taking meds the same time every day b. keep the medications in a special, labeled location c. use a diary to record each dosage after it is taken d. arrange for a caregiver to remind the patient when meds should be taken

c. use a diary to record each dosage after it is taken The use of memory notebooks have been documented to improve orientation as well as support everyday living tasks such as remembering simple IADLs. A and B could be helpful in reminding the patient to take the medication, but would not be as effective. D does not facilitate the patients independence.

One of the key positioning strategies an OT plans to provide for a patient with a hip replacement while the patient is sitting or supine is: a. place pillows on the lateral side of the hips b. elevate the foot on the side of the hip surgery c. use a foam wedge between the legs of the patient d. insert a roll under the patients knees

c. use a foam wedge between the legs of the patient A key aspect when working with patients after hip replacement surgery is to provide positioning strategies to prevent postsurgical complications. Leg adduction must be avoided as a surgical precaution. Answer A would not prevent adduction. B and D would increase hip flexion, which is contraindicated.

an OT is working with a child whose ability to perform schoolwork in a busy 2nd grade classroom is affected by his limited visual attention. What is an adaptation of the sensory environment that would most improve attention during a visual task? a. work with soft music playing near his desk b. place childs seat work against a pattered background c. use headphones during seat work d. attach a small reading lamp to his desk

c. use headphones during seat work This helps to filter outcome of the auditory input from other students in the classroom. It can be helpful for some students with poor visual attention., The other answers may reduce visual attention.

A child is on a pureed diet because of an inability to chew food. What would be the MOST effective method to facilitate the childs ability to chew? a. encourage removal of food from spoon with teeth b. stimulate texture management with vegetable soup c. use infant mesh feeder bag d. place a raisin between the childs teeth

c. use infant mesh feeder bag the child can practice chewing foods encased within a mesh feeding bag to experience repetitive chewing with less risk for gagging or choking. A is incorrect because scrapping off food from a spoon with teeth does not encourage any voluntary motor control. B is incorrect because it combines two textures which would be unpredictable and increase risk of aspiration. D is incorrect because a raisin is too great a step from pureed food in terms of texture and may lead to choking.

An individual diagnosed with cancer has developed chemotherapy-induced neuropathy resulting in wristdrop and decreased sensation in the RUE. The individual is right handed. Which treatment approach is MOST appropriate for optimizing UE functional performance. a. meaningful activities to improve right handed pinch and grasp b. RUE sensory re-ed c. use of orthosis to provide support until strength of the wrist returns d. change of dominance training

c. use of orthosis to provide support until wrist strength returns Chemotherapy induced neuropathy usually causes wristdrop and footdrop which are transient. Because the condition is temporary, positioning the wrist in a functional position is the best approach. Answers B and D are for permanent conditions. Wrist drop is a result of radial nerve involvement, which would not affect the muscles of the hand needed for pinch and grasp.

Which of the following is the BEST bathing technique for an individual with COPD and low endurance? a. tub bathing with hot water b. standing for a quick shower. c. using a bath chair with the door open d. tub bathing using lukewarm water

c. using a bath chair with the door open People with COPD have difficulty breathing when the environment is hot or humid. A and B are incorrect because the energy demand. D is incorrect because the transfer in and out of the tub would be taxing.

An individual with an SCI has an indwelling catheter and is participating in a group trip to a movie. He is self-conscious about the catheter bag and wants to be sure it is not obvious during the trip. What is the best action for the OT to take? a. use a condom catheter for the duration of the trip b. place the cath bag in a backpack hung from the handles of the wheelchair c. utilize a leg bag for the trip d. hide the cath bag under a backpack on the individuals lap

c. utilize a leg bag for the trip a leg bag can be hidden under most pants. A urine collection bag must not be placed above the level of the bladder for more than a few minutes. A condom cath still requires some type of bag, so it does not solve the problem.

An OT working in acute care sees several patients bedside each hour. When moving from one patients room to the next, what is the MOST important action to take to prevent the spread of infection? a. place all lines and equipment in the correct location b. wear protective clothing c. wash hands between patient contacts d. dispose of any waste in proper containers

c. wash hands between patient contacts this is the most significant way to reduce the spread of infection. B and D are additional examples of standard precautions. A has nothing to do with infection control

a school aged child with multiple disabilities is beginning to develop some controlled movement in the UEs. When would it be most appropriate to introduce switch operated AT? a. once the child develops the ability to sit upright for 10 minutes b. after the child can reach and point with accuracy c. when the child demos any reliable controlled movement d. once the child develops isolated finger control to push the buttons.

c. when the child demos any reliable controlled movement as long as the child can produce any reliable movements, switches can be adapted to meet positioning and mobility needs. B and D are not necessary to use simple switches. A would not be required if the child needed to be positioned in a reclining or side lying position

when adapting a toilet for use by a 6 y/o child with limited postural control, what should the OT primarily pay attention to? a. whether the toilet paper can be reached without a major weight shift b. whether the flush handle is easy to manipulate c. whether the childs feet reach the floor d. whether a nonskid mat is available on the floor to prevent slipping

c. whether the childs feet reach the floor a relaxed position during toilet use is essential for success in elimination training. Plus, the feet can be used to help with postural stability. The other answers are useful considerations that should be addressed after the issue of support has been resolved.

A school based OT is working on hand function with a school-age child diagnosed with juvenile RA. To prevent hand fatigue, what piece of AE should the OT recommend? a. reacher b. jar opener c. wide barreled pencil d. plate guard

c. wide-barreled pencil writing muscle tension and fatigue may be reduced for some children by using a wider barreled pencil. The other answers are all adaptations that can be used with a child with this condition. However, A and B are infrequently performed by the child during the day and the pencil is more beneficial for his school routine.

During a task group, an individual diagnosed with borderline personality disorder tells one group member that their project looks like it was made by a little kid and tells another group member that their project looks stupid and tosses the project across the room. Which is the BEST approach for the OT to take with the person who is displaying this behavior? a. ban the individual from group activities until her behavior improves b. tell the individual how disappointed you are in her behavior c. work with the individual on appropriate communication skills d. ask the individual how she thinks the other group members felt when she did that

c. work with the individual on appropriate communication skills characteristics associated with personality disorders include excessive and unstable expression of emotions and disregard for the rights and needs of others. Because individuals with BPD are often manipulative, practitioners need to be aware of these facts and monitor their own emotional state. Expressing feelings of disappointment would not be beneficial. Answer A would deprive the necessary intervention.

An OT is explaining to a teacher about high technology that can be used to help a student with multiple disabilities to communicate in the classroom. Which interventions would the OT MOST likely recommend for this child? a. environmental control unit b. wanchik writer c. head pointer d. augmentative communication device

d. augmentative communication device. This is a high-tech aid that can compensate for expressive deficits. A is incorrect because an ECU is a device that allows people with disabilities to operate appliances or other devices. B and C are incorrect because they are both low-tech aids for communication.

An individual with a C6 SCI has been referred to OT 2 days postinjury. Immobilized with a halo brace, the individual demos fair plus wrist extension and poor minus finger flexion. Which of the following interventions should be implemented FIRST? a. volar resting hand orthoses to prevent flexion contractures b. wrist support with universal cuff to promote independence c. wrist orthoses to promote development of tenodesis d. instruction in bed mobility techniques to prevent decubiti

c. wrist orthoses to promote development of tenodesis This type of orthoses is implemented in the acute phase of rehab because tenodesis grasp is developed by allowing the finger flexors to shorten. A volar hand orthosis would interfere with the development of tenodesis. Interventions related to promoting independent performance should begin as soon as possible, but issues related to positioning must be addressed first. An individual would not be instructed in bed mobility until after the acute phase.

An OT is developing a pilot program to promote positive behavior in students in the local elementary school. The program will address behavior needs of students with sensory processing challenges and will provide prevention support for all students in the classroom . Recognizing the setting and the students capacities, which approach is MOST likely the OTs first consideration? a. an in service training program for parents to help them learn about healthy limits in computer and tv time at home b. consultation with the phys ed teacher to develop a warm up and cool down exercise routine that promotes self-regulation c. classroom screenings to identify students who need additional OT support d. instruct teacher and students to use the Alert program and include routine follow-up visits to classroom to colead sessions with the teacher

d. . instruct teacher and students to use the Alert program and include routine follow-up visits to classroom to colead sessions with the teacher The alert program is a cognitive intervention that is well suited for use with a whole classroom. The program likens arousal levels to an engine running. Answer A does not have a direct connection with the students classroom behavior and does not give the student any strategies when they need help to focus or remain alert during the day. B may provide some limited benefit, however, the strategies would be limited to when the children are in gym class. C is not an appropriate activity in this pilot program.

The OT is planning intervention for a 10 y/o child with learning disabilities and significant difficulties accomplishing writing and drawing tasks, secondary to perceptual motor dysfunction. When selecting a service delivery approach, which is the BEST choice? a. consultation with classroom teacher so student stays in least restrictive environment b. 1:1 intervention from the OT to focus on the student's skill development before middle school. c. intervention from the OT in a small group with other students having similar needs. d. A combination of service delivery models so the OT can address student, teacher, and task needs.

d. A combination of service delivery models so the OT can address student, teacher, and task needs. the aim of OT services is to increase the childs participation in important childhood occupations. Service delivery in the schools relies on multiple models to meet student's needs across activities and routines. The other answers are incorrect as they each offer only one service delivery model, therefore limiting the OTs ability to support the students in various ways.

A client sustained a hand injury months ago while at work. The individual is now diagnosed with CRPS and is experiencing pain that interferes with work and self-care. Which of the following would be the best pain management modality option to discuss with the client? a. hot packs b. cold packs c. fluidotherapy d. TENS

d. TENS This is an intervention technique thought to stimulate the afferent A nerve fibers in the high-frequency mode and stimulate the release of morphine-like neural hormones, the enkephalins, in the low-frequency mode. Its efficacy as an intervention for pain control is well documented in literature. The other answers may assist more with joint stiffness and cumulative trauma disorders but not necessarily for the pain experienced with complex regional pain syndrome.

Which BEST depicts the recommended service approach when the IEP team has included OT services to support the student's participation in the school curriculum? a. the OT engages the student in activities outside of the classroom to avoid distraction to peers. b. the OT works with the student in the classroom when all students are completing seat work. c. the OT arranges to work with the student in the classroom during after-school hours. d. The OT recommends strategies that are easily embedded within the students activities and routines in the curriculum and promote his or her participation.

d. The OT recommends strategies that are easily embedded within the students activities and routines in the curriculum and promote his or her participation. IDEA mandates that intervention should be provided in the least restrictive environment. Answers A and B represent approaches that remove the student from his peer group and this is acceptable only when support cannot be provided in the context of the classroom. C is incorrect as it does not represent the opportunities and challenges present in the students regular classroom experiences.

An OTA with previous school based OT experience has joined the school district staff. The supervision OT is developing the OTAs workload. Which is the MOST likely action taken by the OT? a. evals the OTA completed in her previous setting are identified, and students needing these evals completed are assigned to the OTAs caseload b. OTA will not complete any evals until the probationary period of employment is complete c. OTA will observe the OT completing each eval once, then the OTA will administer the measure for the next student identified by the OT d. The OTA demos administering assessments completed in previous setting, OT reviews OTAs scoring, and then determined which ones the OTA could complete in this setting.

d. The OTA demos administering assessments completed in previous setting, OT reviews OTAs scoring, and then determined which ones the OTA could complete in this setting. D reflects the OTA providing competency. The other answers omit the necessary step of confirming competency.

an individual with a C6 SCI is unable to button his shirt. The OT would be most likely to select which type of AE to assist the individual with buttoning? a. a buttonhook with an extralong flexible handle b. a buttonhook with a knob handle c. a buttonhook on a .5 inch diameter, 5 inch long wooden handle d. a buttonhook attached to a cuff that fits around the palm

d. a button hook attached to a cuff that fits around the palm. Individuals with C6 SCI may have tenodesis grasp or no available grasp. Therefore a cuff is the appropriate choice.

Which of the following would the OT MOST likely recommend and individual use at home after discharge following a total hip arthroplasty (posterior lateral approach) for ADL completion? a. wire basket attached to a walker b. padded foam toilet seat 1 inch in height c. short handled shoe horn d. long handled reacher

d. a long handled reacher A person with a THA must avoid hip flexion of 90 degrees or greater, adduction, and IR. A reacher would allow the person to adhere to the precautions. A would not allow the person to come close to a counter without having to side step. B or C would be inadequate because they would cause the person to flex the hip past 80 degrees while performing self-care tasks.

an OT is filling out an incident report regarding a patient who was seen on the rehab unit. Which of the following scenarios is the Most likely to be the subject of an incident report. a. patient complained of nausea during a standing activity b. a patient with a SCI indicated that his hand orthoses were uncomfortable c. a patient who had a THR did not follow precautions while completing a dressing activity but did not complain of discomfort d. a patient with a CVA and left neglect caught his left arm in the wheel or the wheelchair resulting in a cut and bruise

d. a patient with a CVA and left neglect caught his left arm in the wheel or the wheelchair resulting in a cut and bruise in general, the therapist should complete an incident report when a situation occurs that is harmful to the patient or OT. This includes cuts, burns, falls, and contact with hazardous materials. The other answers are worthy of report in documentation but not necessarily an incident report.

what is the most appropriate adaptation for gardening for an individual with a back injury? a. ergonomically correct hand tools b. a wheelbarrow with elongated handles c. a 12 inch high seat with tool holders d. a raised garden bed

d. a raised garden bed this prevents prolonged spinal flexion. Answer B would be more difficult to control while pushing than a wheelbarrow with normal handles. Answer C would benefit an individual with low endurance. Answer A would assist for joint protection.

An OT has been asked to design a health promotion group for individuals with cognitive impairment functioning at a parallel task group level. What is the MOST appropriate activity for the OT to use? a. planning a dinner party b. a healthy eating board game c. a weight watchers support group d. a therapist led aerobics class.

d. a therapist-led aerobics class. a parallel task group involves individuals performing there own task and provides opportunities for sharing or social interaction. The other answers describe activities at higher levels of interactions.

an OT working in a state-owned outpatient facility for adults with ID receives a referral for an individual recently admitted to the facility. While observing the individuals performance on the job line in the sheltered workshop, the OT notices the individual appears board, hits his legs with his fists, and talks loudly. Which type of environment would best meet this individuals needs? a. a quiet workroom with minimal noise and lowered blinds to reduce lighting b. a job that offers structure and repetition c. rotation between several job stations, with an alarm to alert the individual when it is time to rotate d. a workroom with upbeat music and a variety of jobs from which to choose

d. a workroom with upbeat music and a variety of jobs from which to choose this individuals behavior is typical of someone who is sensation seeking. A and B would be for someone who is sensory avoidant. People who are low registration are more likely to prefer C.

A student working at her desk has difficulty when alternating visual focus on a blackboard 20 ft away and then on items on the desktop area to complete mathematics problems. This most likely indicates a problem with: a. ocular motility b. binocular vision c convergence d. accommodation

d. accommodation answer a refers to the ability to pursue an object visually in an efficient and smooth manner. answer B is the ability to focus the eyes on an object at varying distances and on seeing a single object clearly. answer c is the ability to move eyes inward or outward with continued focus on the object

during the course of treatment, an individual diagnosed with depression tells the OT that he leads a very isolated lifestyle and often feels alone and afraid. What is the best therapeutic response for the OT to make? a. reassure the client that we can be friends or you will be his friend b. tell the client i know how you feel c. encourage the client to socialize more often d. acknowledge the feelings of isolation

d. acknowledge the feelings of isolation communication is enhanced when the individual believes he is being heard. D is an example of empathetic listening. A and B can be counterproductive to developing a therapeutic relationship. C is not helpful, because the individual would probably do that if he were able

Participants from a partial hospitalization program are taking an outing into the community. One program member complains of SOB. The OT practitioner determines the individuals resting HR is 128 bpm and BP is at 230/180. What is the MOST appropriate action for the OT to take? a. continue the community re-entry outing, watching to see if any additional symptoms occur b. immediately return the rest of the clients to the day program c. help the patient lie down and wait until vital signs return to normal d. activate the emergency response by calling 911

d. activate the emergency response by calling 911 The numbers are above safe ranges for vitals and evidence that the patient is medically unstable. The other answers do not recognize the seriousness of the situation and may delay the necessary medical attention.

In planning a therapeutic dressing activity for a first grade child who has down syndrome, what should the OT consider first? a. adaptive equipment b. adaptive clothing c. proper positioning d. adapted teaching techniques

d. adapted teaching techniques A child with ID characteristically has difficulty with processing info, applying concepts, and organizing and sequencing tasks. The other answers are of secondary importance.

an OT has received an order to evaluate an individual who is 2 days post total hip arthroplasty and expected to be discharged home in 2 days. what areas are most important to address prior to discharge? a. cognitive and perceptual functioning b. work, self care, and leisure performance c. upper and lower extremity strength, range of motion and coordination d. adaptive equipment needs and assistance from other

d. adaptive equipment needs and assistance from others For at least 6 weeks, movement will be restricted. Although leisure and work performance are important, they are secondary to the safety of following precautions. Answer C is also important but is secondary to safety.

through the school based eval process, the OT may consider many possibilities for intervention; however, the determination that OT will be included in the students IEP is made at which point in the IEP process? a. after the OT reviews and interprets all the eval data b. during the interview with the classroom teacher who referred the child for eval c. at the IEP meeting once the OT eval results are summarized to team members d. after the students goals and objectives are determined for the IEP.

d. after the students goals and objectives are determined for the IEP. After IEP goals and objectives are correct, the team is able to discuss which professionals should address particular goals, when they will be addressed, and where they will be addressed. Each of these decisions is made on the basis of the students need, not the personal preferences of the professionals.

A first grade student has difficulty completing seated tabletop activities in the classroom because of the limited sitting balance. Which should the OT provide to promote postural adjustments in sitting? a. sturdy chair without armrests while the child completes seat work b. corner floor seat with built in desk surface while the child eats lunch c. bolster for back support while the child colors d. air filled cushion on the childs seat during seat work periods.

d. air filled cushion on the childs seat during seat work periods. this modifies the position in which the child balances to change stability and improve balance control whether the child is stationary or moving through space. It requires the child to continually adjust to the subtle movements of an unstable surface. The other answers provide additional external support.

An individual with MS reports exhaustion after cleaning the house but cannot afford a house cleaner. Which of the following strategies is MOST appropriate to recommend to this individual? a. suggest that the client hire outside help with the financial support of family members b. prescribe activities that will increase strength c. use the largest joints available for the task d. alternate tasks that requires standing with those that can be performed sitting.

d. alternate tasks that requires standing with those that can be performed sitting. When fatigue impedes occupational performance, energy conservation techniques should be considered. Convincing the individual to do something she cannot afford is not an example consistent with collaborative decision making. B may ultimately be useful but is secondary to energy conservation. C is a joint protection technique more suitable for individuals with arthritis.

An individual with mental illness has accepted a secretarial position but is concerned that her high levels of distractibility may interfere with concentration and job performance. Which of the following interventions is MOST appropriate for this individual? a. arrange for the individual to have a job coach b. suggest that the employer provide more frequent breaks c. explain the problem of distractibility to the employer. d. ask the employer to provide an isolated cubical as the work space

d. ask the employer to provide an isolated cubical as the work space an isolated cubical is a reasonable accommodation for this individual. The ADA supports the individuals right to work by prohibiting discrimination. A job coach is more appropriate for an individual who needs frequent cueing or assistance. Frequent breaks are for individuals with anxiety who need to manage stress. Although explaining distractibility to the employer is good, it does not offer a solution.

A client arrives to exercise group wearing traditional garments consistent with religious practices that cover the body from head to toe, restricting movement while exercising. The OT observes the individual's discomfort and is concerned about participation, but also wants the client to be comfortable in an environment that is obviously culturally different for her. What is the BEST action for the OT to take, demoing cultural competence in this situation? a. when the opportunity arises, discreetly explain to her that dressing more appropriately for the group will allow for better participation. b. comment on the beauty of the clothes, asking for affirmation from the rest of the group. c. gently suggest that next time the client wear loose-fitting garments that will be comfortable to exercise in. d. ask the individual if there are any cultural guidelines for how to dress for an exercise group

d. ask the individual if there are cultural guidelines for how to dress for an exercise group. Singling her out could be embarrassing and making suggestions of how to dress (B and C) without first obtaining culturally relevant information is not a culturally sensitive approach.

An OT in the hospital outpatient department meets with the parents of a 7 y/o boy with developmental coordination disorder (DCD). The OT believes his performance challenges his participation in school and brings this up. What is the MOST appropriate way for the OT to address the parents? a. request permission to send progress report to the school district. b. ask parent for permission to call the school district's OT to learn about the IEP referral process. c. Suggest that the school district pay for the child's outpatient therapy, as it likely relates to the child's school performance. d. ask whether the school district has addressed his coordination difficulties and if not, discuss further whether the parent wishes to raise this issue with the child's teacher.

d. ask whether the school district has addressed his coordination difficulties and if not, discuss further whether the parent wishes to raise this issue with the child's teacher. This answer allows the opportunity for the parent to learn about the OT's concerns and make decisions about the next steps. the other answers do not respect the collaborative team process embraced by OT.

what is the appropriate level of involvement for an OT within a parallel group in order to address the psychosocial needs of the group members? a. provide verbal encouragement to support participation b. observe from the sidelines and refrain from taking a directive role c. participate as an active member, allowing participants to run the group d. assist clients in the selection of simple, short-term tasks

d. assist the clients in the selection of simple, short term tasks parallel groups are the lowest group. THe OT defines the task and provides clients with the necessary assistance and empotional support. It would not be required that the OT provide encouragement for experimentation. OT participates as a member at a mature level.

A newly referred patient complains of frequently dropping lightweight items and reports a numb feeling in both hands. Which of the following instruments is MOST important to use when evaluating the individual? a. goniometer b. dynamometer c. pinch gauge d. aesthesiometer

d. asthesiometer this is an instrument designed for sensory testing. The term usually indicates a tool consisting of a ruler type scale and two points that can be moved closer of farther apart for testing two point discriminations. The dynamometer and pinch gauge both measure strength. A goniometer is used to measure ROM.

During an initial interview, parents describe their child as having difficulty communicating and interacting with others. The OT observes him repeatedly gazing upward and scanning the ceiling or quickly patting his hip. These behaviors described are MOST likely to be associated with what disorder? a. ADHD b. Childhood conduct c. Obsessive compulsive d. autism spectrum disorder

d. autism spectrum disorder. ASD is characterized by severe and complex impairments in reciprocal social interaction and communication skills, and by the presence of stereotypical behavior, interest and activities. Children with ADHD display behaviors of inattention, hyperactivity, and impulsivity. Children with childhood conduct disorder display severe forms of chronic misbehavior such as physical aggression or lying. OCD is characterized by intrusive thoughts and repetitive behaviors that have little or no functional purpose beyond decreasing tension.

the family of a client receiving OT asks the OT to write a recommendation to terminate the clients drivers license. The OT is uncomfortable taking this action and perceives it as an ethical dilemma. Which principle of the AOTA code of ethics could such an action most clearly conflict with? a. social justice b. veracity c. fidelity d. autonomy

d. autonomy the OT would be directly impacting self-determination by reporting concerns about the individuals ability to drive. Social justice requires that OT personnel treat people fairly. Veracity requires the accurate and objective truth be told and documented. Fidelity states that OT personnel should treat other colleagues and professionals with respect.

an individual with Parkinsons disease has particular difficulty with both starting and stopping movements. What is the best strategy for the OT to teach the individual and his or her caregivers? a. perform a deep-breathing exercise when movement is frozen b. practice the starting phase of the movement repeatedly c. encourage the client to mentally identify the series of steps needed to initiate the movement d. avoid crowds and potential distractions, and perform tasks slowly while walking.

d. avoid crowds and potential distractions, and perform tasks slowly while walking. it is important to educate the client who is experiencing freezing to avoid distractions and crowds. Additionally, he should avoid corners and narrow spaces. The other answers will not provide the sensory information needed at the movement to evoke movement or change a frozen movement pattern

the OT program manager of a local hospital must determine the cost-effectiveness of services provided. Which of the following methods would most effectively obtain this info? a. outcomes measurement b. utilization review c. program eval d. benefit or cost center analysis

d. benefit of cost center analysis budgets include revenues and expenses which are commonly referred to as cost centers. Outcome measurements are taken at the completion of service intervention and are used to evaluate the effectiveness of intervention. Utilization review assess the care that is provided to ensure that services were appropriate. Program eval is a method to determine how well the programs goals have been achieved.

an OT for a school-age child with visual perceptual deficits is making recommendations to compensate for visual figure-ground challenges in the classroom. Which activities would the OT most likely recommend to the childs teacher? a. place a red line on the left side of the page b. use a timer for certain activitiies c. teach the child to use lists and color coding of books and folders d. block out areas of a page to expose material that is the immediate focus

d. block out areas of a page to expose material that is the immediate focus. D is correct because this compensatory technique is a way of dealing with visual figure ground issues by giving specific attention to distinctive features through highlighting. A is incorrect because the technique is used for visual tracking or neglect. B is incorrect because it is a technique for visual attention. C is incorrect because it helps children deal with visual memory.

An individual presents with a visual impairment and complains of cloudiness in the lenses of the eye, dulling colors, and overall blurry vision when performing all tasks. Which of the following is the individual most likely experiencing? a. age related macular degeneration b. glaucoma c. diabetic retinopathy d. cataracts

d. cataracts Age related macular degeneration is a disease that affects sharp central vision whereas B causes gradual failing of peripheral vision. C results in blurred vision or spotty areas called scotomas.

a childs long term goal is to increase participation in classroom learning activities that require fine motor skill. The assessment has revealed a deficit in tactile discrimination, specifically stereognosis. What is the most relevant STG? a. child will correctly identify 5 of 5 fingers touched when given tactile stimuli b. child will correctly identify 5 of 5 shapes drawn on the dorsum of her hand c. child will correctly identify 5 of 5 matching textures d. child will correctly identify , only by feel, 5 of 5 common objects

d. child will correctly identify, only by feel, 5 of 5 common objects stereognosis is the ability to identify objects through proprioception, cognition, and the sense of touch. A describes localization of tactile stim, B demos graphesthesia, and C demos tactile discrimination.

which child with neuromotor impairment would benefit MOST from using a prone scooter for exploratory play? a. child with CP with predominant extensor tone b. child with generalized low muscle tone who is easily fatigued c. a child with cognitive limitations and poor sensory awareness d. a child with spina bifida with LE paralysis

d. child with spina bifida with LE paralysis Prone scooters require the use of the arms and ability to lift the head while moving. Typically, children with this condition have enough UE coordination and strength to propel themselves on a scooter. They usually have the cognitive and sensory awareness to negotiate the scooter within the environment. This type of scooter is contraindicated for the children in all other answers. Neck hyperextension is required for exploratory play and would further increase extensor tone in the child with CP. A child with low tone would be unable to maintain this position for long, and a child with cognitive limitations and poor sensory awareness would be at a risk of injury.

What is the NEXT step for an OT to follow after initial eval of a 1 y/o requiring early intervention services? a. independently develop an IEP b. collaboratively develop an IEP c. independently develop an IFSP d. collaboratively develop an IFSP

d. collaboratively develop an IFSP The service plan required by federal law and provided through early intervention programs is called an individualized family service plan. This plan is developed by the family and other team members once evaluations are complete. Answers A and B refer to the IEP, a service plan for children ages 3-21 years. IEPs or IFSPs should be completed in a collaborative manner.

A parent refuses to allow a level II fieldwork student to lead his sons therapy session, stating he only wants someone whos qualified. He insists the OT student must observe the OT lead the session. What action should the OT take first under these circumstances? a. attempt to persuade the parent to allow the OT student to lead the session b. advise the parent that the OT student is finishing her fieldwork next week c. cancel the session for that day and document that the parent refused OT d. comply with the parents request and allow the OT student to observe the session

d. comply with the parents request and allow the OT student to observe the session OT should always respect the recipients of their services. Attempts to persuade conflict with this ethical principle. Canceling the session is punitive and witholds services from the child.

an OT is initiating training with a worker with an ID. The game packaging task involves first putting a pencil in a box, followed by putting a score pad in the box. Which of the following reinforcement schedules would be used initially to achieve the goal of learning the task sequence. a. intermittent reinforcement with correct responses b. reinforcement every 10 minutes c. reinforcement for every fourth correct response d. continuous reinforcement of correct responses.

d. continuous reinforcement of correct responses This is most often used at the beginning of treatment. The other answers are examples of intermittent reinforcement which is more effective at maintaining the desired response.

an OT consulting with a community center is developing a club for youth at risk for violence, with the primary goal of engaging the children and meaningful occupations designed to build their participation and community enhancement. which activity is most consistent with this goal? a. develop a community-based chess competition b. plant flower seeds in small pots to be taken home once a sprout c. provide a psycho educational intervention on anger management d. convert an empty lot filled with trash into a community garden

d. convert an empty lot filled with trash into a community garden This modifies the environment and impacts occupational performance. A promotes competition and B promotes individual reward. Anger management skills are important for children at risk but a psychoeducational format will not directly contribute to community building

during which meal preparation tasks would symptoms related to deficits and executive functioning the most evident? a. making a strawberry, banana, and mango smoothie and a blender b. using a sharp knife to chop items for a salad using six ingredients c. baking and icing cupcakes using a conventional oven d. cooking hamburgers, french fries, and green beans on the stove and in the oven

d. cooking hamburgers, french fries, and green beans on the stove and in the oven executive function refers to higher order mental capacity such as decision-making, problem solving, planning, task switching, modifying behavior in light of new information, self-correction, generalizing strategies, formulating goals, and sequencing complex actions. this answer requires planning, sequencing, timing, and judgment to make sure all items are cooked in different ways and for different amounts of times. All other tasks are less cognitively complex and require the individual to follow straightforward directions that are not sensitive to a particular order, or only require timing for one item.

when evaling light touch of the primary somatic system, the OT will most likely use: a. tunning fork on the skin b. alternating pressure of both ends of a safety pin c. warm and cold water tubes placed on the skin d. cotton swab to apply light touch to a small area of the skin

d. cotton swab to apply light touch to a small area of the skin A assesses deep pressure, B pain perception, and C temperature perception.

the OT is planning intervention to promote developmental skill acquisition for an infant in the NICU. which action will have the most therapeutic impact? a. adjust the infants positioning to promote physiological development b. recommend early intervention referral to assess the infant when home from discharge c. complete the neurobehavioral assessment and identify intervention d. create a comfortable collaborative relationship with parents and promote parenting skills.

d. create a comfortable collaborative relationship with parents and promote parenting skills development with families in the NICU requires facilitation of the familys active role with their infant and the NICU team. This approach provides the parents with effective tools to best nurture and care for their infant at any time and in any environment and has permanent impact on the developmental outcome of the infant. Answer A only provides temporary change. B and C are strategies that can impact the infant but are less important at this time.

an OT is working on money arrangement skills with an individual with cognitive deficits following a stroke. Which of the following components would be least amenable to errorless learning approach? a. identifying coins b. making change c. writing a check d. creating a budget

d. creating a budget The errorless learning approach may be used with individuals with memory impairments. The other answers all have correct methods for accomplishing the tasks and therefore lend themselves to an errorless learning approach. D can be accomplished so many different ways so it would not be appropriate for errorless learning

A rehab manager is attempting to find a way to have financial success in the department while ensuring patient satisfaction. Which of the following is MOST likely to be implemented to assess patient flow, develop critical pathways, and cut costs? a. quality improvement b. peer review teams c. cost accounting d. cross training

d. cross training Cross training is the training of a single rehab worker to provide services that would ordinarily be rendered by several different professions. Multiskilling is sometimes the term used. Arguments have been made against this method. Quality improvement is a systematic approach to monitoring patient care. Peer review is a component of quality improvement. Cost accounting is a method of tracking the costs of specific services or diagnostic groups.

A child with DD in kindergarten class has just developed the strength and stability in his right hand to hold scissors properly and make snips in paper. Which activity would help the child to develop the next level of scissor skill? a. cut cloth and cardboard b. cut along curved lines c. cut along straight lines to cut out a triangle d. cut the paper into two following a straight line

d. cut the paper into two following a straight line Scissor skills develop from first snipping to cutting a single straight line. The ability to cut heavier materials develops last in the sequence of scissor skills. Ability to cut curved lines develop after the ability to cut straight lines.

An individual arrives late to the morning social skills groups, sits slumped in a chair, and appears to have trouble staying alert and engaged. The OT notices the same is true for several other individuals. Which preparatory activity would be the MOST effective in promoting client's engagement in the group? a. slowly rocking in a rocking chair b. listening to a CD of ocean sounds c. wrapping themselves up in heavy, warm blankets d. dancing to a strong, fast beat

d. dancing to a strong, fast beat. These individuals are demoing under-responsiveness which is characterized by difficulty paying attention and engaging. The other answers are calming responses that will likely make the clients more tired.

when designing and implementing an assertiveness training program, what should be the focus of the FIRST session? a. strategies for making a direct request for desired change b. how to make I feel statements using the when you ______ I feel ____ format c. how to stand up for yourself in order to have more control in your life d. defining the 3 types of responses: passive, assertive, and aggressive

d. defining the 3 types of responses: passive, assertive, and aggressive Assertiveness training programs begin by defining the types of behavior. In order to understand their own behavior and the behavior of others, group members must know the different types of behavior. The other answers are all skills that the group can begin to practice once they understand the different types of responses.

An OT has been hired to consult in a residential setting that provides housing and support for individuals with histories of substance abuse. Children under the age of 2 may reside with their parents, who have a max LOS of 9 months. After administering the parenting well strength and goals assessment, the OT found that commonly identified areas of need included setting limits, having positive interactions, finding fun things to do, expressing anger without hurting anyone, and identifying their childs strengths. How can the OT most appropriately address this concern? a. eval each parent and design individualized interventions to promote parenting and play skills. b. provide educational materials about the impact of parental substance abuse on children c. provide a course to the day-care staff about age appropriate expectations and play activities d. design a program focusing on developing parenting skills, beginning with a module on age appropriate expectations

d. design a program focusing on developing parenting skills, beginning with a module on age appropriate expectations Understanding child development would allow the parents to have more realistic expectations of their children. Future modules could address additional parenting skills. A would likely not be realistic. B and C may be activities performed by the OT, however, answer D is most appropriate to perform first.

When completing a screening following a teacher's concern over a third graders hand writing skill, what is the PRIMARY goal of the OT? a. provide strategies to promote the students fine motor development b. document the appropriateness of the teachers referral c. establish the students hand skill and penmanship baseline d. determine need for further evaluation

d. determine the need for further formal eval A screening includes observation of skills to determine need for what standardized test and further analysis should be pursued.

An OT is planning a group program in an acute care psychiatry setting for severely mentally ill individuals who display disorganized thinking and difficulty functioning in many areas. What is the MOST appropriate type of group to use with these patients? a. activity b. psychoeducational c. neurodevelopmental d. directive

d. directive The directive group meets the needs of the most severely and acutely mentally ill and most minimally functioning patients. The environment is actively structured in organization and leadership to assure maximum participation. Activity groups require a higher level of task behavior and ability to engage in occupation to enable skill development. Psychoeducation groups are based on CBT and focus on teaching info that requires a level of learning capacity that may be impaired with acute illness. NDT enhances sensory integration in persons with long histories of chronic schizophrenia.

on arrival to the NICU, the OT finds the infants parents present. Which is the optimal intervention to assist the parents in responding to their infants behaviors? a. review the chart to complete birth history information and speak to the infants primary nurse b. introduce yourself as their OT, explain your role in the childs developmental care, and leave to review the medical record c. provide parent friendly written positioning recommendations for the parents to review. d. discuss parents concerns and provide appropriate info and resources

d. discuss parents concerns and provide appropriate info and resources NICU approaches embrace developmentally supportive care that acknowledges that parenting an infant in the NICU is stressful. The other answers are all strategies used in intervention, however, answer D is clearly the optimal strategy for fostering parental observation and interpretation skills, building positional and handling skills, and responding to their infants behaviors. Initial chart review and updating with nursing staff are essential but they are the least optimal strategies to pursue while the family is there.

Based on review of eval results, the OT believes that school-based services to support a student with autism should include education for the teaching staff concerning ways to promote positive behavior throughout the school day. In preparation for the IEP meeting, what is the best plan to address this concern? a. collect handouts with strategies teachers can use to help alert the child in the classroom and distribute them at the meeting. b. recommend the staff attend an in-service program to learn ways to help all students promote self-regulation c. demo for the teacher how the student can use fidget toys and other sensory strategies to help him stay focused during seat work d. discuss the benefit of OT provision of staff education and determine how this can be implemented and documented in the students IEP.

d. discuss the benefit of OT provision of staff education and determine how this can be implemented and documented in the students IEP The other answers may be components of the service delivery, however, in isolation, they do not represent approaches that promote a collaborative approach to program development for this student.

An OT is formulating a home program of play activities for parents of a 4 y/o with DD. What type of activities would be BEST for developing symbolic play skills? a. building blocks b. board games c. paint by number kits d. doll house and dress up clothes

d. dollhouse and dress up clothes This encourages symbolic play, and the other answers are not only representative of the younger child or older child. They also encourage more defined, close-ended play with predictable results.

which activity should the OT suggest to promote handwriting readiness skills for children in a preschool classroom? a. hammering dowels into a styrofoam board b. creating buildings with wooden blocks in the block center c. rolling clay into many different-sized balls d. drawing lines and shapes using shaving cream or sand

d. drawing lines and shapes using shaving cream or sand Activities to develop handwriting readiness skills aim at improving FM control and isolated finger movements, promote rewriting skills, and enhancing left-right discrimination. None of the other answers emphasizes isolated finger movements.

an individual who works as a nurse reports difficulty squeezing the bulb of the sphygmomanometer when taking BP readings and difficulty opening pill bottles. Which of the following instruments would be the most appropriate for assessing this individual? a. goniometer b. anesthesiometer c. volumeter d. dynamometer

d. dynamometer this individual demos deficits in strength which is measured by a dynamometer. goniometer measures ROM, anesthesiometer measures sensation, and volumeter measures edema.

eval results for a person with arthritis will most accurately reflect true functional abilities if scheduled to take place at what time of day? a. early morning (8am-10am) b. afternoon and evening c. late morning (10 am to 11 am) d. early morning and again at another time of day

d. early morning and again at another time of day as with assessment of function, the time of day and medications should be noted because these factors may influence results. Evaling only in the morning accurately reveals function only at one time of the day. People with arthritis may have changes in functional status once morning stiffness has disappeared,

An individual with depression finishes making a poorly constructed christmas tree ornament and tells the OT to throw it away because it is such a sorry looking thing. What is the best way for the therapist to respond? a. suggest giving the ornament to a family member b. state that it is beautiful and ask to keep it c. ask whether it is okay to hang it on the tree in the unit d. emphasize the importance of participation rather than the end product

d. emphasize the importance of participation rather than the end product individuals with depression are aware when they are not performing at their usual level and recognize when they have not done a good job. It is important to be understanding and empathetic. Answers C and B will be recognized as false.

An OT is conducting an ongoing assertiveness training group. Which of the following strategies would be MOST helpful in facilitating group cohesion and exchange of feedback among group members? a. define assertiveness, passivity, and aggression for the group members b. encourage all group members to release their aggressive feelings physically and verbally toward inanimate objects c. demo commonly used assertiveness techniques to the group members d. encourage group members to share similar experiences and reactions with each other

d. encourage group members to share similar experiences and reactions with each other. Participants can learn about themselves through identifying with others. A and C are designed to impact information, not to promote group cohesion. B is an example of catharsis which may not be helpful to all members.

and OT is working with a medically stable child who sustained bilateral UE, partial thickness burns 3 days ago while playing with a lighter. which ADL intervention should the OT introduce first? a. instruct the child to use adaptive equipment for all ADLs b. encourage independent compression garment application c. performed bilateral UE PROM exercises twice a day d. encourage independent ADLs with minimal reliance on adapted utensils and tools

d. encourage independent ADLs with minimal reliance on adapted utensils and tools Intervention goals for a child with burns include optimizing ROM, ensuring good use of hands, and optimizing skills for self-care and school activities. It is important to avoid over reliance on AE. B is typically contraindicated with open wounds and is implemented once the wounds close. C is not considered an ADL intervention.

An OT provided info about adaptations that will assist in resuming sexual activity to a patient with a SCI. Afterward, the patient confides in the OT that there are serious personal issues affecting the sexual relationship with his spouse. What is the best action for the OT to take? a. encourage the patient to explain further about the problems b. explain that this is normal and that divorce rates are actually higher after serious injuries c. direct the patient to speak with a physiatrist about the concerns d. encourage the patient to speak with the rehabilitation psychologist

d. encourage the patient to speak with the rehab psychologist. If the OT is not the one to educate the client or partner, the therapist should anticipate the need for further information. Therapists can provide counseling for issues such as sexual function, sexual abuse, and values. This Psychologist's role is more extensive, which appears to be what is required. Answer A may require skills beyond the scope of OT.

Using the MOHO FOR, eval of an individual should focus primarily on which of the following? a. an identification of problem behaviors that need to be extinguished b. clarification of thoughts, feelings, and experiences that influence behavior c. assessment of cognitive function, including assets and limitations d. eval of effects of values, routines, performance skills, and the environment on role performance.

d. evaluation of effects of values, routines, performance skills, and the environment on role performance The concept of this model address the motivation for occupation, the routine patterns of occupations, the nature of skilled performance, and the influence of the environmental occupation. answer a is the behavioral frame of reference. answer c evaluates cognitive disability frame of reference.

To improve written communication, an OT practitioner would be MOST likely to recommend a large keyboard to enhance computer access when an individual: a. has limited UE ROM, but adequate fine motor coordination b. fatigues rapidly when reaching for the keys c. uses only one hand to access the keyboard d. has good UE ROM, but difficulty accessing small targets

d. has good UE ROM but difficulty accessing small targets A person with limited ROM but adequate fine coordination and a person who fatigues quickly would benefit from a smaller keyboard. A large keyboard would be more difficult for a person using one hand.

A parent of 3 young children complains of severe low back pain. What is the BEST way to get a toddler up from the floor onto changing table for a diaper change? a. carry the child as close to her body as possible. b. lift the child from the floor using a wide BOS, one foot slightly in front of the other c. lift the child from the floor by bending the knees, not the back. d. have the child climb up onto her lap.

d. have the child climb up onto her lap all answers describe the use of good body mechanics. Answer D is preferable because it minimizes the amount of bending and lifting required.

an infant is beginning to sit and lean forward onto his arms. The OT considers recommendations to the family to increase opportunities for the infant to practice developing postural control and balance reactions during their everyday routines. Which is the most likely suggestion given to this family? a. during trips to the mall, the family should carry the infant in an upright position, holding the childs lower torso enabling the child to practice balance while looking at the variety of stimuli in this environment b. each day as she watches the morning news, mother should continue to practice the trunk control and balance activities modeled by the OT during home visits. c. extended family members who spend time with the child should attend the OTs home visit to learn ways the parent embeds new strategies that support childs development into their daily routines d. family should use cushions to support the child in seated play on the living room floor, with toys present, while parent watches nightly new or folds laundry

d. family should use cushions to support the child in seated play on the living room floor, with toys present, while parent watches nightly new or folds laundry This reflects the use of the natural environments as a context for intervention and embedding therapeutic intervention into the families everyday activities. A is incorrect as it provides only limited opportunity. With B and C there is no assurance that the intended learners understand, can replicate, or even if the opportunity will arise.

a long-term goal for an individual with progressive weakness is for the family to carry out his feeding program. Which statement is the MOST appropriate short-term goal? a. patient participate in feeding program b. patient will feed himself with mod A c. family will feed patient safely and independently 100% of the time d. family will demo independence in current positioning and feeding techniques 50% of the time.

d. family will demo independence in current positioning and feeding techniques 50% of the time. goals must be measurable, obtainable, objective, and functional. Answer A does not provide measurable criteria.

An OT is working with an individual who is experiencing a manic episode and is highly excitable. Given that this individual has expressed interest in all kinds of craft activities, which type of craft activity would the OT be MOST likely to select to provide external structure for the client? a. doing a detailed needlepoint project requiring fine stitches b. using clay to shape an object of one's choice c. doing a watercolor paint by number project d. finishing a prefabricated wood birdhouse from a kit.

d. finishing a prefabricated wood birdhouse from a kit An individual who is manic rarely completes projects because attention is fragmented. Interventions should be short, simple, and success enhancing. A person experiencing a manic episode is likely to exhibit high energy levels, short attention span, poor frustration tolerance, difficulty delaying gratification, and difficulty with decision making. A would require too much attention. C is an unfocused activity involving creativity and skill performance that could lead to frustration. And B uses unpredictable materials and requires creative decisions.

An OT is instructing an individual with left hemiplegia how to remove a pullover shirt. The correct sequence is: a. remove shirt from unaffected arm; remove shirt from affected arm, gather shirt up at the back of the neck, and pull gathered fabric overhead b. remove shirt from affected arm, remove shirt from unaffected arm, gather the shirt up at the back of the neck, and pull gathered back of fabric off over head c. gather shirt up at the back of the neck, pull gathered back fabric over head, remove shirt from affected arm, and remove shirt from unaffected arm d. gather shirt up at the back of the neck, pull gathered back fabric off over head, remove shirt from unaffected arm, and remove shirt from affected arm

d. gather shirt up at the back of the neck, pull gathered back fabric off over head, remove shirt from unaffected arm, and remove shirt from affected arm The other answers would most likely result in failure

While participating in activities to promote strength, an individual with MS who was recently admitted to the hospital complains of fatigue. Which of the following actions is MOST appropriate for the OT to take? a. instruct the individual to work through the fatigue to complete the session b. instruct the individual to work through fatigue for another 5-10 minutes c. discontinue strengthening activities d. give the individual a rest break

d. give the individual a rest break Rest breaks need to be scheduled to avoid fatigue with MS and therefore should be encouraged. Activities do not need to be discontinued but should be designed to benefit the patient without undue fatigue. Answers A and B would be contraindicated.

An OT is developing a measurement plan to track changes in social participation of several students with autism spectrum disorder. Which is the most likely measurement strategy the OT selects. a. school function assessment b. documentation of change in narrative progress notes c. photos taken over 2 month period at recess d. goal attainment scaling

d. goal attainment scaling goal attainment scaling can be effective for both developing and measuring progress toward social skills goals. These can be highly individualized nd used systemically to measure progress over time. Narrative notes are not a systematic measurement approach. Photos may contribute to measuring changes, however, they only represent a moment in time. The SFA does include items that measure aspects of social function, however, it is not a comprehensive assessment and may not include items that reflect priority areas for each student.

An OT is working with a group of individuals who present with decreased strength due to a variety of issues related to deconditioning. The OT plans to invite them to participate in aquatic therapy once a week as the environment of a pool has the potential to decrease: a. hypersensitivity b. proprioception c. edema d. gravitational force.

d. gravitational force Aquatic rehab approaches may be employed to remediate strength. The other answers do not relate to the overall goal of increasing strength, owing to deconditioning.

An individual with an L4 SCI wishes to become independent in driving an automobile. The MOST appropriate piece of AE for this individual is: a. palmar cuff for the steering wheel b. spinner knob for the steering wheel c. pedal extensions for acceleration and breaking. d. hand controls for acceleration and breaking.

d. hand controls for acceleration and breaking. An L4 SCI would result in paraplegia. Hand controls use hand motions to control the accelerator and brake mechanisms, limiting the need for LE function. answers A and B are options for individuals who need to steer single-handedly. Pedal extensions can be installed for individuals with limited LE reach.

an elderly man is caring for his elderly wife who had dementia. He becomes frustrated when she repeatedly asks for her mother and then becomes upset when he tells her that her mother is not there. Which strategy should the OT recommend to the husband to help manage the situation? a. use reality orientation by explaining to her that her mother has been dead for a long time b. set limits by firmly telling her to stop asking for her mother c. use therapeutic fibbing by telling his wife that her mother will be coming shortly d. help him to identify the feelings she is having but is unable to express verbally

d. help him to identify the feelings she is having but is unable to express verbally The residents searching for her mother may reflect a sense of loneliness. Rather than reacting to the behavior as a negative attribute of the individual, one must consider the motive behind the action. A can have the effect of unnecessarily confronting the person and the individual may react to the news as if she is hearing it for the first time. Telling the person to stop will not address the need that is being expressed verbally. C might work temporarily but may backfire if the person continues to question the story given or if it causes sadness or anger.

an OT observes a child with autism flapping his right hand in front of his eyes repeatedly. This behavior most likely serves which purpose? a. helps focus vision at close range b. relieves pain in his right hand c. relieves paresthesias in his fingers d. helps child maintain state of arousal

d. helps child maintain state of arousal These repetitive acts are believed to help the individual maintain a state of balance, calmness, or arousal.

an OT is instructing a family how to observe for stress in their preterm infant in the NICU. What are the MOST important autonomic responses to watch? a. frantic movements in the UEs and shaking b. limpness in UE and LE and crying c. twitches or jerky movements while the infant is asleep d. hiccups of skin color changed to flushed

d. hiccups of skin color changed to flushed these are automatic nervous system signs reflection physiological instability or stress. A and B are changes in motor behavior that reflect stress. C reflects disorganization in the infants sleep state.

An OT is working with a 4 y/o child who has significant hearing loss. The child also demos decreased fine motor coordination when compared with typically developing children her age. Which activity would the OT MOST likely implement to address these needs? a. parachute activities to provide vestibular input, alert the child, and encourage gross motor practice b. dressing practice with childs right and left shoes color coded to compensate for decreased reception of verbal cues c. introduce the child to the other children via socialization groups to increase social interaction and game playing d. play hide and seek with clay balls in which the child digs for various coins and then places them in to a piggy bank.

d. hide and seek with clay balls in which the child digs for various coins and then places them into a piggy bank. Answer D is correct as it encourages the child to work on fine motor skills. Answer A is indicated for gross motor or kinesthetic needs but are not necessarily related to fine motor limitations. B is an appropriate choice when addressing self-care skills. Although C would be appropriate because many children with hearing impairments feel isolated from the hearing community, the primary goal is fine motor skills.

an OT is working with an adult diagnosed with dementia who is experiencing moderately severe cognitive decline and who was admitted to the hospital after accidentally setting a kitchen fire. What are the most appropriate follow up services to identify relative to this patients meal planning needs after discharge? a. OT services to teach the patient to cook safely b. volunteer companion services to supervise cooking at home c. transportation services to bring the person to a community meal site d. home delivered meal services

d. home delivered meal services This individual is past the new learning stage and requires meals be prepared for her in order to remain safe. B provide companionship but not skilled supervision of activities. C would be inconvenient and challenging.

The OT is developing a survey to ask about child care patterns, health , and wellness. The researcher should attempt to avoid the use of double barreled questions. Which of the following is MOST representative of a double barreled question? a. how many times do you bathe your child each day? b. how many times a month do you seek outside assistance for emotional support? c. how many times a day do you feel frustrated and/or discouraged by your encounters with your child d. how many times a week do you complete a.m care (hygiene and dressing) and transportation to school for your child

d. how many times a week do you complete a.m. care (hygiene and dressing) and transportation to school for your child A double-barreled question is one that has more than one question embedded with it.

The OT's intervention plan is focused on developing executive function to help a 17-year-old student with autism increases organization and work habits for greater success in the high school curriculum. what type of activities would the OT program most likely include? a. visual attention, memory skills, and problem solving b. tactile and proprioceptive input while the student makes a goal-directed, adaptive response c. leisure and high interest activities into which problem solving tasks are embedded d. identification of own strengths, anticipating challenges and strategies that support performance

d. identification of own strengths, anticipating challenges and strategies that support performance enhancing metacognitive skills to increase task performance is among the approaches recommended for students with ASD. The other answers are incorrect as they do not directly relate to the executive function challenges that are the focus of this question.

After attending several OT group sessions, a client with a hx of low self esteem is showing more self confidence and asks for the OTs phone number. What is the MOST appropriate response for the OT to make? a. give him the phone number and instruct him to call when he is feeling depressed b. ignore the request, but reinforce what progress he is making. c. explain that the OT has someone special in her life d. in private, explain the nature of the client therapist relationship

d. in private, explain the nature of the client therapist relationship it is inappropriate for the OT to have more than a professional relationship. While self disclosing some info to clients can be a therapeutic tool, there is some info that would be misinterpreted or inappropriate. Ignoring the clients request constitutes avoidance which is inappropriate. Whether or not the OT has a boyfriend is immaterial and should not be used as an excuse. It may be an easy way out but is not professional.

An OT is applying PNF techniques for weight shifting during an activity that requires an individual to use the right hand to remove groceries from a bag on the floor to the right. The MOST benefit would be gained from this activity by then placing the groceries: a. on the counter directly in front b. on the counter to the left side c. in the upper cabinet to the right side d. in the upper cabinet to the left side

d. in the upper cabinet to the left side Placing groceries in the upper cabinet to the left will promote the greatest degree of weight shift to the affected side. Putting groceries on the counter or upper cabinet to the right side would not allow for enough weight shift.

when assessing the sense of proprioception at an individuals joint, the OT would hold the lateral aspect of the elbow, wrist, or digit, and then move the body part into flexion or extension. The response of a client would then be to: a. identify where the pain is b. demonstrate the stretch reflex c. demonstrate full AROM d. indicate whether the body part is being moved up or down

d. indicate whether the body part is being moved up or down The other asnwers are not indicative of a proprioceptive assessment. C

An OT is working with an individual who is s/p RUE shoulder replacement. The client is right hand dominant and an avid woodworker. Currently, limited RUE ROM prevents participation in woodworking, which requires at least 120 degrees of shoulder flexion. The OT and the patient collaboratively develop a plan that incorporates woodworking to increase RUE ROM. After establishing the just right challenge at 40 degrees of shoulder flexion for sanding the pieces of a wood project, how should the OT progress the patient in order to achieve his goal? a. progress him to the use of power tools. b. apply increasingly heavier wrist weights during sanding and staining activities c. provide him with tasks that require bilateral coordination d. involve him in sanding and staining increasingly larger pieces

d. involve him sanding and staining increasingly larger pieces. The larger the project the greater the ROM required. A would not necessarily promote greater ROM but would involve increasing complexity of the task. Weights could be used to promote strength but not ROM. and Bilateral coordination would be more useful for individuals with coordination and perceptual deficits.

A homebound individual has been receiving OT for self-care and home management training and PT for ambulation and stair training following a stroke. THe individual reported to the OT that since the last visit, she walked to the bus stop independently y herself. This statement is significant in that it indicates that the individual: a. is making progress toward her goals and home therapy should be discontinued b. has achieved her PT goals c. has achieved her OT goals d. is no longer home bound

d. is no longer home bound After the individual can leave the residence, he or she cannot continue to receive home care and would be referred to outpatient services. Without knowing the individuals goals, it is not possible to say he or she has met them.

The administrator of a long-term care facility asks the OT who works there to help develop a job description so he may hire a second OT. What would an OT job description MOST likely contain? a. previously established mutual goals, quality of patient care, achievement of predicted outcomes, and evidence of relationship building b. a summary of primary and secondary job functions, references, and physical exertion requirements c. organizational relationships, personality characteristics, and accomplishments desired in a candidate d. job title, employers expectations, productivity expectations, and how performance will be measured.

d. job title, employers expectations, productivity expectations, and how performance will be measured. Items that are not required but may complement the job description are personality characteristics, past experience, and accomplishments. A are characteristics of a performance review. Answers B and C include items more appropriately listed on a resume.

An individual with guillian barre acute syndrome was recently admitted to a rehab unit and is expected to remain for 3-4 weeks. At what point in the rehabilitative process should the OT order adaptive equipment for this individual? a. after the patient and family have accepted the individuals disability b. as soon as the insurance provider approves it c. within the first week of therapy d. just before discharge

d. just before discharge An intensive interdisciplinary rehab program is typically implemented during the recovery phase as the client begins to regain physical movement. Equipment should be provided just before discharge to accurately determine what the individual needs. B and C may not be necessary at the time the individual is discharged. A may not necessarily correspond with the appropriate time of ordering the equipment.

When the OT implements intervention to support a child with neuromotor disorder to increase her participation in home and community activities, which approach should be the PRIMARY focus? a. securing assistive tech devices that promote participation in everyday activity, followed by training to develop her competent use in the AT. b. developmental approaches that maximize skill development and prevent further ROM limitations c. assurance that a peer buddy is available in all contexts and settings to provide assistance with unanticipated needs d. knowing the child and family's priorities, modifying contexts to promote participation, enhancing performance, and educating family.

d. knowing the child and family's priorities, modifying contexts to promote participation, enhancing performance, and educating family Best practice utilizes a multicomponent top-down approach. Answer A and B may be a part of the approach, yet they do not describe an adequate program approach. C is incorrect as this emphasis in an OT program encourages the child to rely on others and may lead to persistent dependence.

an OT is preparing to complete an initial eval on an individual diagnosed with OCD. which strategy for modifying the environment is likely to be most effective while interviewing this individual? a. minimize environmental distractions b. utilize open ended questions c. instruct the individual to take her time and not rush d. limit time available to answer each question interrupting when possible

d. limit time available to answer each question interrupting when possible individuals with OCD frequently experience time consuming compulsions. Limiting time and using highly structured interviews and close-ended questions will help the evaluation process. B is more useful for eliciting information. C may be beneficial for a highly anxious individual. Individuals with attention deficits would most likely benefit from decreasing environmental distractions.

The OT manager is training colleagues about the use of electronic health records as the outpatient rehab department transitions from a handwritten documentation system. Some important considerations must be taken to assure the client health record, such as: a. legibility of entries b. creating updated care plans c. completing a discontinuation summary d. logging in and out each time an electronic entry is made

d. logging in and out each time an electronic entry is made steps must be taken to assure the security and confidentiality of all patients. The other answers do not pertain to electronic documentation, but serve as forms of documentation requirements. Answer A refers to handwritten documentation.

a child has difficulty controlling food in her mouth when swallowing. In helping the parents plan snacks, what type of nipple would the OT most likely recommend? a. habermann b. single hole c. broad based d. long, thin

d. long and thin a long, thin nipple can encourage a recessed tongue forward during the feeding process. Answer A was developed specifically for children with cleft palates. B is most effective fro children who perform better with a steady flow of liquid versus bursts of fluid from multi-hole nipples. C is most effective for a cleft lip as well.

an individual in overall good health has expressed the overarching goal to remain as functional and independent as possible after recently being diagnosed with OA of the UE and LEs . Using a client centered approach to prevent further complications, what is the OT most likely to recommend to the client? a. lifting weights 3 times a week for 1 hour b. listening to relaxation tapes 3 times a week before bed c. vocational training d. low impact aerobics 3 times a week for 1 hour

d. low impact aerobics 3 times a week for 1 hour treatment of clients with arthritis must take into account the progressive nature of the disease. The overarching goal of therapy is to decrease pain, protect joints, and increase function. A is considered to be an activity that promotes hyperextension and can increase pain and further joint damage. B is appropriate for coping with arthritis but not for prevention of complications. C is not indicated.

an OT is employed by a school district to provide consultation to vocational instructors in a high school program for students with ID. Which approach is most appropriate for the OT to provide? a. developing an in-house prevocational program b. bringing in outside speakers from different job settings c. teaching the vocational instructor different assessment tools and scoring procedures d. meeting the vocational instructor weekly to discuss adaptations to work tasks

d. meeting the vocational instructor weekly to discuss adaptations to work tasks OT consultation is a collaborative process used to identify and solve problems. A and B are activities typically completed by the vocational teacher. A vocational teacher should already be able to perform assessment

While making brownies, an individual is able to obtain all the supplies from the cabinet and check the oven temperature periodically. However, when the TV is turned on halfway through the activity, she becomes involved with the program and burns the brownies. This individual is showing signs of a deficit in which area? a. sustained attention b. detecting and reacting c. shifting of attention d. mental tracking

d. mental tracking Mental tracking is the ability to simultaneously keep track of two or more stimuli during an ongoing activity. Sustained attention is the ability to consistently engage in an activity over time. Detecting and reacting is the ability to detect and react to gross changes in the environment such as the phone ringing or name being called. Shifting attention is the ability to alternate attention between tasks with different cognitive or motor requirements.

the BEST way to screen quickly for orientation, attention, and memory in older adults is to administer what tool? a. LOTCA b. Allen diagnostic model c. ACL test d. Mini Mental

d. mini Mental It is a widely used screening tool for assassin cognitive function older adults. It takes approximately 10 minutes to administer. answer a takes much longer and is more comprehensive. answer c screens for cognitive level but not specifically orientation, attention, and memory. answer B provides more in-depth information about cognitive level

the OT has received a referral for a patient recently admitted to the psych unit. what is the best method to quickly screen for overall cognitive functioning? a. barth time construction b. AMPS c. ACL d. Mini mental

d. mini Mental this is a widely screening tool for cognitive function that can be done in approximately 10 minutes. answer a requires an individual to complete a color-coded chart to demonstrate how the individual spends their time during the day. answer be assesses ADL performance, scoring the individual on both motor and process abilities. answer c uses leather lacing to assess problem solving skills.

While evaluating an individuals ADL status on the first day of treatment following open heart surgery, it is MOST important for the OT to: a. keep the MET level below 3 b. observe for shortness of breath c. listen for complaints of chest pain d. monitor HR, BP, and symptoms

d. monitor HR, BP, and symptoms It is important to measure these vitals in individuals with cardiac conditions. Answers B and C are symptoms associated with exercise intolerance and are subsumed within answer D. Answer A may be too high for individuals on the first day of therapy following open heart surgery.

which is most likely to provide relevant eval info about how a child with motor delays completes self-care activities? a. standardized tests of dressing skills b. review of the medical record c. a developmental screening test d. observation of a child during self-care activities

d. observation of a child during self-care activities Interview and structured and naturalistic observations are eval methods typically used to measure ADL performance in OT. The other answers provide necessary info about performance components, development, and other parameters, but are not as effective in helping the evaluator learn about the childs self care function

when performing a naturalistic observation of dressing skills with a young child with DD in an outpatient setting, what should the OT do first? a. provide oversized clothing to ensure success b. have the child dress and undress in a distraction-free corner of the clinic c. give assistance as needed to minimize frustration d. observe the child entering the clinic and taking off his coat and shoes

d. observe the child entering the clinic and taking off his coat and shoes This represents naturalistic observation in which the therapist gathers information. A and C describe situations in which assistance is provided, therefore not allowing the child to demo his true skills.

an individuals weight has changed during the course of hospitalization, and there is now a space of 2.5 inches between the individuals hips and the sides of the wheelchair. Which is the best recommendation regarding wheelchair fit? a. obtain a wider wheelchair because this one is now too narrow b. encourage the individual to lose weight c. pad the sides of the wheelchair to improve the fit d. obtain a narrower wheelchair because this one is now too wide

d. obtain a narrower wheelchair because this one is now too wide the wheelchair as described is too wide, not too narrow. The need to gain or lose weight should be discussed first with an individuals physician. Padding the sides is a less desirable option because a narrower wheelchair is more effective.

an OT is training an OTA to administer a standardized assessment. Which of the following is the best method to eval the OTAs services competency with that assessment tool? a. observe a competent OTs performance of the standardized test b. observe a competent OT practice then teach another how to administer the test c. follow procedures exactly as outlined in the test manuals d. obtain the same results as another OT does who has demonstrated service competency

d. obtain the same results as another OT does who has demonstrated service competency service competency is the process of teaching, training, and evaluating in which the OT determines that the OTA performs tasks in the same way that the OT would and achieves the same outcome.

An OT is beginning to work with individuals in recovery in a community-based program. The OT needs an evaluation tool that can be administered in less than 20 minutes, identifies occupational performance limitations, can be used to establish goals based on client priorities, and measure outcomes. Which is the BEST tool for this purpose? a. Occupational Performance History Interview 2 b. Role checklist c. COPM d. Occupational self-assessment

d. occupational self-assessment This is a self report measure that identifies client's problem areas in occupations, their competence in the occupations, and the importance they assign to them. It takes 10-20 minutes to administer, and goals can be based on the results. It can also be used as an outcome measure. The OPHI-II is a semistructred interview focused on occupational adaptation in three areas: occupational identity, occupational competence, and impact of occupational environment. It takes about an hour to complete and generates a narrative of the individual's life history allowing the therapist to compare performance over time. The role checklist takes approximately 15 minutes to complete, however it does not provide a way to measure occupational performance. The COPM takes about 30-40 minutes to complete. It is a semistructured interview of self-care, productivity, and leisure. It can be useful for goal setting, intervention planning, and measuring outcomes, but it takes longer than the occupational self-assessment.

after a variety of interventions have not led to the improvement of a parent anticipated in her 4 y/o social participation, she asks the OT for info about the benefit of vitamins, special diets, and other alternatives for children with autism. What is the best response for the OT to give this parent? a. advise the parent that these are not research-based interventions, so the family should not pursue them b. encourage the mother to be patient and allow her son to participate in the current therapy program for a longer time c. suggest the parent contact her insurance company to learn more about their policies on reimbursement for these types of treatments d. offer to bring summaries on the research into various complementary and alternative approaches for children with autism and review them during the next home visit.

d. offer to bring summaries on the research into various complementary and alternative approaches for children with autism and review them during the next home visit. this reflects a collaborative and well-informed approach. OT should be open minded and support the family as they make decisions. The other answers refer the parents to other resources and do not respond to the parents need for information and support.

An OT completing a home assessment has recommended a hospital bed, lightweight wheelchair, bedside commode, reachers, long handled sponge, shower chair, and a hand-held shower. The family states they can only afford the items that can be billed as DME. What should the OT do next? a. order all of the equipment b. order the reacher and long handled sponge c. order the shower chair and hand held shower d. order the lightweight wheelchair and hospital bed

d. order the lightweight wheelchair and hospital bed Generally, Medicare payments are based on services provided and Part B also covers some DME. Part B does not cover elevated toilet seats, grab bars, or AE because they are not considered medically necessary. Answers B and C are incorrect because they include items that aren't considered DME. Depending on the persons medical condition, a bedside commode may be covered.

an OT is part of a team developing a policy for discontinuation of services to students in school based settings. When is the most appropriate time to end school based OT services? a. failure to achieve long term goals by the end of the school year b. inability to accomplish short term goals within one marking period c. transition from middle school to high school d. participation in all classroom activities and routines with agreed upon adaptations

d. participation in all classroom activities with agreed upon adaptations. when a student has acquired the needed skills and can use them during school, the IEP services are no longer necessary. A and B would most likely indicate to the OT to change the approach in order to make more successful goals. C is not a reason to end services, as services continue into highschool.

an individual with RA complains of joint pain and inflammation. The individual reports continuing with the home program despite the pain and demos a series of briskly executed AROM. after eval, the OT should instruct the individual to: a. continue performing the program as demod b. perform gentle AROM with weights as tolerated c. eliminate all ROM for a week d. perform only gentle AROM

d. perform only gentle AROM this allows the patient to control the movement and avoid overstretching inflammed joint tissues. Brisk AROM and the addition of resistance are likely to cause further damage. C would result in further joint stiffness and loss of ROM.

An OT observes a child climb into a high chair and jump up and down on the floor. However, when presented with a new rocking horse, the child is unable to mount the horse and ride. This MOST likely indicates a problem with which area? a. muscle strength b. motor control c. reflex integration d. praxis

d. praxis praxis is the ability to conceive of and plan a motor sequence and adapt quickly to a novel environment. A is incorrect because there is evidence to reflect sufficient motor control and integrated primitive reflexes.

An individual who sustained partial thickness burns is ready for the rehab phase of treatment after 4 weeks in ICU. The wounds have all closed and there are no open areas remaining. Prior to performing ADL training, what is the most important action for the OT to take? a. debride the wound b. remove compression garments c. complete UE strengthening activities d. perform skin conditioning techniques

d. perform skin conditioning techniques These are used to improve scar integrity and durability against minor trauma caused by pressure or shearing forces, decrease hypersensitivity, and moisturize dry, newly healed skin. These techniques should be used for any burned or surgical sites that took longer than 2 weeks to heal. Lubrication and massage should be performed 3-4 times per day. Wound debridement is completed by nurses or doctors and begins in acute phase of treatment. B are worn constantly after wounds are healed, not prior, and should not be removed for anything other than bathing and skin care. Answer C may be part of the treatment plan, but does not need to be performed prior to ADL activities.

A first-grade child has difficulty with mature pincer grasp and handwriting. The MOST appropriate activity the outpatient OT would recommend to the parent to complete at home would be: a. crayon drawing on sandpaper b. copying shapes from the blackboard c. rolling out clay with a rolling pin d. picking up raisins with a pair of tweezers

d. picking up raisins with a pair of tweezers This activity is the only one that targets isolated finger use. Research suggests that small muscle development and the ability to hold utensils are among the prerequisite skills for handwriting. Drawing on sandpaper can be used to increase kinesthetic awareness and finger strength. Copying shapes is primarily a perceptual motor task. Rolling out clay is an activity that promotes bilateral hand use and the development of palmar arches.

part of an OTs documentation reads, continue social skills training program and encourage child to attend one new after school club activity within the next week. Which selection of a SOAP note is the most appropriate to place this statement? a. subjective b. objective c. assessment d. plan

d. plan This section includes statements related to continuing intervention, the frequency and duration of treatment, and the need for further evaluation or new goals.

An OT practitioner is planning a program for an individual who needs to increase shoulder strength, ROM, and endurance. Which of the following activities is MOST suitable for periodic upgrading? a. blowing up and tying balloons of various sizes. b. shooting a game of balloon darts c. painting faces on balloons d. playing balloon volleyball

d. playing balloon volleyball This may be graded by adding resistance to the arm in the form of weights. Endurance may be increased by adding repetitions. Raising the height of the net could increase ROM. A and B are primarily fine motor activities.

the OT is selecting activities for a school age child with postural control limitations. Which activity would best promote postural control? a. sliding down a playground slide b. playing simple simon says c. swimming laps d. playing on a trampoline

d. playing on a trampoline postural control refers to balance during functional movements. A stimulates vestibular processing. B facilitates perceptual processing. C is a good activity to improve endurance.

A therapist is discussing discharge plans with the parents of a 7 y/o child with sensory defensiveness. What is the MOST appropriate strategy to introduce the proprioceptive input to heighten the childs arousal? a. walking barefoot on textured surfaces b. slow, repetitive rocking over a large therapy ball c. playing in a large box of styrofoam peanuts d. playing tug-of-war with rope.

d. playing tug of war with rope This affords alternating pushing and pulling tasks that are constantly changing their proprioceptive demands to facilitate increased arousal. Answers A and C are incorrect because they emphasize additional tactile input. B is incorrect because it emphasizes slow vestibular input.

an OT is running a social skills group and the activity involves planning a trip to the aquarium. The OT starts with a warm-up activity. Next,the OT introduces the goal for the session, which is determining what time they would need to get the bus from their building in the morning in order to arrive at the aquarium by 10 am. What would be the next step for the OT to take? a. provide feedback to group members who are not respecting the opinions of other group members b. assign homework involving working with a partner to determine when to get the bus back to the center c. ask the group members whether anyone has ever been to the aquarium, and see how many fish they can name d. point out where on the schedule to find times for buses leaving the center in the morning.

d. point out where on the schedule to find times for buses leaving the center in the morning. social skills training is founded on the principles of operant conditioning. The goal for this session has already been introduced, so the next step is demonstration. Answer D is correct because it follow the behaviorally based instructions. This would then be followed by corrective feedback. Then a homework assignment would be used to reinforce learning. C would proceede the demonstration.

during a meal planning group, an individual with a brain injury is frequently observed looking out the window instead of engaging in the planning task. Which of the following strategies would have the most effective and immediate impact on the individuals participation in the meal planning activity? a. teach the individual how to use self instruction statements b. instruct the client in the use of orienting procedures c. have the individual use time pressure management strategies d. position the individual with their back to the window

d. position the individual with their back to the window This behavior is indicative of an attention deficit. Strategies aimed at modification of task and environment have proved to be beneficial for this environment. A and B are effective strategies but would require too much time to instruct the individual to perform. C is used for individuals who have deficits in speed of processing info.

a resident of a long-term care facility is receiving OT because of difficulties with eating. The FIRST step the OT should perform at meal time is to: a. provide skid-proof placements, plate guard, and utensils with built-up handles b. observe for swallowing after each bite of food c. instruct the caregivers about a special eating set up for the resident d. position the person in an upright posture, making sure head is flexed slightly and in midline.

d. position the person in an upright posture, making sure head is flexed slightly and in midline. making sure that the resident is correctly positioned is the first step in addressing eating problems. Providing adaptive devices may or may not be helpful, but would not be the first step without assessment of need. B and C would be important but would occur later in the intervention process.

An OT is teaching several older adult individuals with COPD energy conservation techniques in home-management skills. Following learning principles for older adults, the MOST effective way to present the info would be to: a. present all the important principles to be covered together in a single presentation b. keep the presentation loosely structured, rather than highly organized. c. attempt to persuade individuals about the importance of those points on which they do not seem to agree d. present important principles in small units that are spaced at a slower than normal pace.

d. present important principles in small units that are spaced at a slower than normal pace. according to learning principles for older adults, learning will be more effective if info is presented in smaller units. A is incorrect because learning will be too quick. B is incorrect because highly organized presentations increase retention.

a child with ADHD, hyperactivity type, is receiving OT to increase his attention span. Keeping this in mind, the OT introduces a construction activity. When a puzzle with many pieces is placed in front of the child, the child sweeps many of them onto the floor and starts throwing the remaining ones around the room. How can the OT MOST effectively restructure the activity to facilitate a successful experience for the child? a. use soft foam pieces b. provide puzzle pieces of only one color c. use larger interlocking puzzle pieces d. present only a few puzzle pieces at a time.

d. present only a few puzzle pieces at a time This method allows the OT to help the child focus on a few relevant stimuli to make it possible to complete a short term task. This experience will then increase attention span. Answer A is less likely to cause injury if through but not likely to increase attention span. B may reduce visual stimulation somewhat and C may make manipulation of the pieces easier, but the overwhelming stimuli would make these strategies irrelevant.

after fabricating an antispasticity splint/orthotic, the MOST important thing for the OT to determine is: a. the fingers are flexed b. the thumb is in an opposable position c. the patient is wearing it at all times d. pressure marks or redness from the splint disappear within 20 minutes

d. pressure marks or redness from the splint disappear within 20 minutes a universal concern is that orthotic devices should be checked for correct fit. Most orthoses are not worn all the times, but are removed for activities such as self care or exercise. The positions of antispasticity splints should be without fingers flexed or thumb opposed

An OT is educating the family of an individual admitted to the ICU after sustaining full thickness dorsal hand burns. Which of the following MOST accurately explains the purpose of burn hand orthosis? a. decrease pain and allow for AROM b. minimizes hypertrophic scarring c. diminishes the need for skin grafting d. prevents stress on the extensor tendons and ligaments while decreasing edema

d. prevents stress on the extensor tendons and ligaments while decreasing edema Burn hand orthoses prevent stress on the superficial tendons and ligaments, decreasing edema secondary to the avoidance of dependent position. It does not allow for ROM because it is an immobilizing static splint. The other answers are not primary reasons for using this splint.

A child running in the playground trips and falls forward, landing on an outstretched arm. How is this reaction BEST described? a. attitudinal reflex b. righting reaction c. equilibrium reaction d. protective extension response

d. protective extension response This is a postural reaction used to stop a fall or prevent injury when equilibrium reactions fail to do so. Equilibrium reactions maintain center of gravity over the BOS. Righting reactions bring the head and trunk into an upright position. A are automatic movements that appear within the first year and are designed to align the head with the body. Most of these reflexes are suppressed within infancy.

as discharge time approaches for an individual hospitalized for mental illness, the OT is approached by the individuals parents who express their desire to help their adult child. What is the best action for the OT to take? a. explain that HIPPA does not allow discussion of the clients situation with family members b. explain that recovery occurs best when family members are not involved c. emphasize the importance of putting their adult childs needs first d. provide them with info about NAMI

d. provide them with info about NAMI Providing info about NAMI incorporates several competencies including a collaborative relationship with the family and education. Nothing is said about C. B is not best practice. The situation at hand does not involve confidentiality, and thus A is incorrect.

an OT is fabricating a dynamic orthosis for an individual who sustained a low-level radial nerve injury while slicing lunchmeat. How should the orthosis be designed? a. provide wrist extension, MCP flexion, and thumb flexion b. prevent wrist extension, MCP extension, and thumb extension c. prevent wrist extension, MCP flexion, and thumb flexion d. provide wrist extension, MCP extension, and thumb extension

d. provide wrist extension, MCP extension, and thumb extension A low-level radial nerve injury results in decreased extension of the MP joints, thumb, and fingers. The purpose of this orthosis is to prevent the extensor tendons for overstretching as well as provide proper positioning of the hand for functional use.

an individual who is hospitalized with schizophrenia is attending OT in order to learn how to manage stress. the OT has decided to use a behavioral approach. which of the following interventions is the best example of a behavioral approach? a. journaling about situations that produce anxiety b. reducing auditory stim and dimming lights c. participating in a deep breathing and muscle relaxation group d. providing tokens for attendance at an excerise group

d. providing tokens for attendance at an exercise group In token economies, clients are given direct reinforcement for positive behavior. Journaling is an example of a psychodynamic object relations approach. Answer B may help reduce overwhelming stimuli but it is an environmental adaptation. C is an approach for stress relief.

A child has difficulty controlling food in her mouth when swallowing. In helping the parents plan snacks, what would the OT MOST likely recommend? a. tapioca pudding b. peanut butter c. carrot sticks d. pudding

d. pudding foods with smooth, even, cohesive textures are easier to manage when a child has oral, sensory, and oral motor impairments. Foods with crunchy, sticky, dense, or uneven in consistency are more difficult to manage.

The OT researcher would like to strengthen a study related to CIMT protocol. The research committee recommended learning more about experimental design, as their current methodology is quasi-experimental. One of the PRIMARY differences between an experimental design and a quasi-experimental design relates to: a. dependent variables b. control group c. manipulation d. randomization

d. randomization a true experimental design refers to the class to-groups design in which subjects are randomly selected and randomized. Quasi-experimental designs are characterized by the presence of some type of comparison group and manipulation but do not contain random group assignment.

the goal of a work program for homeless youths is to develop job skills that will improve housing status. The OT plans to eval each participant. Based on the transtheoretical model, which area of assessment/eval should the OT begin with? a. environmental b. social skills c. work skills d. readiness for change

d. readiness for change The transtheoretical model states that an individual must be ready to change prior to beginning the stages. The other answers would all come later.

An OT hypothesizes that a childs hand writing difficulty is related to a writing posture and posture ergonmics that limit automaticity in handwriting skill. Which option does the OT most likely pursue? a. promote work habits such as positioning paper in midline area of table, use of nondominant hand to stabilize paper, and use of eraser when needed b. implement activities to refine grasp and manipulation of writing tool, gauge speed of work, and avoid fatigue when writing c. help student learn strategies to ensure correct spacing, orientation, uniform size of letters drawn. d. recommend student work station arrangement that supports pelvis/shoulder/head alignment and optimizes arm/hand position for endurance in writing tasks

d. recommend student work station arrangement that supports pelvis/shoulder/head alignment and optimizes arm/hand position for endurance in writing tasks. a functional writing posture considers both body position and fatigue. Answer A is incorrect as it represents concerns related to difficulty in using writing tools. B is incorrect as it represents concerns related to difficulty in writing grasp. C is incorrect as it represents concerns related to difficulty with writing legibility.

The OT is administering the ACL screen to an elderly individual diagnosed with dementia. The OT determines that the individual is functioning at ACL 4.8 (goal-directed activity-personalizing). The family reports that the individual has been in multiple car accidents within the past few months and they are concerned about his driving safety. When taking the client centered approach, what is the MOSDT appropriate intervention for the OT to recommend? a. instruct the family to hide the car keys b. explain to the client the danger that is presented with driving c. initiate OT intervention that focuses on memory and problem solving skills d. recommend that a family member ride in the car when the client is driving

d. recommend that a family member ride in the car when the client is driving. It is unsafe for individuals at ACL level 4.6 or below to drive. Individuals at level 4.8 can benefit from cueing while driving. Hiding the car keys is not a client centered approach. It is likely that the client has already heard about the danger he presents and unlikely that telling him will make him drive any safer. He may benefit from intervention promoting memory and problem solving, but his cognition will continue to decline, so he would still need to be cued.

An OT is working with a 17 y/o girl with impaired mobility, dexterity and communication skills. The teen has low average cognitive abilities. Which would the OT MOST likely recommend to the family regarding emergency alert systems in the event of a fire? a. position a wireless cell phone within the girls reach b. establish an exit routine in order to get out of the house quickly in the event of a fire c. review fire prevention within the home literature d. recommend that the girl wear an emergency alert system pendent around her neck

d. recommend that the girl wear an emergency alert system pendant around her neck. This would be most effective because it does not require a great deal of dexterity to manipulate. A may be effective but would not be of much assistance if her dexterity is limited. B is something that the OT and clients family should address, but in the event that no family members are home. C is something that should be reviewed but does not address immediate needs.

An OT is working with an elderly patient with mild to moderate severity of Alzheimers disease who was admitted to the hospital after accidentally setting a kitchen fire. What is the MOST appropriate info to provide caregivers with after discharge regarding cooking? a. recommend outpatient OT services to teach kitchen safety b. seek volunteer companion services for home supervision c. identify transportation services to bring the person to a community meal site d. refer to home delivered meal services.

d. refer to home-delivered meal services. Answer A is incorrect because individuals with this disease decline over time and have difficulty managing instructions. B provides companionship but not skilled supervision. C could be inconvenient. D would be the best and safest option.

When selecting activities for an 8 y/o with Ducheene MD, which developmental issue is most important to consider? a. establishment of basic trust b. freedom to use his initiative c. development of self identity d. reinforcement of competence

d. reinforcement of competence an 8 yr old is usually at the stage of industry versus inferiority. For a child who is expected to lose motor function gradually, a treatment plan that provides ongoing competence is especially relevant. The other answers describe other developmental issues identified by Erikson that are typically achieved at other ages. A is developed in infancy, B during toddler years, and C during adolescence.

An OT is working with an 8 y/o child with mild spastic CP. The eval has shown that the child has poor in-hand manipulation skills. What would most likely improve the development of this skill? a. grasping blocks to build a building b. placing pegs from one pegboard to another c. carrying books in a bag with a handle d. removing nuts from a bolt

d. removing a nut from a bolt unscrewing a nut from a bolt requires in hand manipulation skills called simple rotation. The other answers are incorrect because they describe activities with no in-hand manipulation skill and the hand especially keeps the object in a certain position as it is grasped, released, or carried.

a 6 y/o student is confusing the direction in which letters of the alphabet are organized. What is the BEST way for the OT to advise the teacher? a. it is suggestive of a problem and warrants further eval of visual perceptual skills b. there is a need to develop visual memory skills to improve spatial recognition c. a formal vision screening is indicated d. research suggests that position in space development is complete at 7-9 years of age

d. research suggests that position in space development is complete at 7-9 years of age A is incorrect as this age group is typically developing their understanding of position in space concept. Therefore developing visual memory is not warranted and there is insufficient info available to justify a referral for vision screening.

when fabricating an orthotic device for an individual with RA, which type of splint or orthoses is most appropriate for the purpose of resting the joints, decreasing pain, and preventing contracture? a. MP joint orthosis b. wrist stabilization c. ulnar drift positioning d. resting hand orthosis

d. resting hand a resting hand orthotic device id indicated for the wrist, fingers, and thumb during acute synovitis. B would be indicated for wrist pain and to protect the extensor tendons from rupture. C would be used to prevent ulnar drift while maintaining joint alignment for grasp and pinch activities. A would assist in keeping the MP joints in normal alignment while preventing volar subluxation.

A new parent recently returned to work and reports difficult concentrating at work because of thoughts about the baby. When at home, the individual feels distracted by thoughts about work. This MOST likely suggests the need to help the individual in what area? a. parenting skills b. attention span c. assertiveness d. role performance.

d. role performance Role performance involves identifying, maintaining, and balancing functions one assumes in society. Changes in important roles may result in feelings of anger, frustration, confusion, boredom, and fear. OTs help people construct or reconstruct their roles when they have experienced dysfunction. This individual is having difficulty balancing roles and is feeling stressed and conflicted. This stress may result in difficulty maintaining attention; however, the larger issue is related to the role performance. Evaluation by an OT could include assessment of parenting and assertiveness skills to determine whether these are areas of need, and if so, interventions could be designed to address these areas to support role performance. However, these have not been identified as areas of need in the scenario.

the OT is encouraging playfulness in a 7 y/o girl with developmental coordination disorder. The child experiences difficulties with motor tasks and often complains that no one likes her. After establishing rapport with the child, what activity would the OT most likely introduce? a. playing a game of go fish b. playing a game of checkers c. jumping rope d. role playing with dolls

d. role playing with dolls This provides opportunities for playfulness through pretend play. The other answers focus on the end product and may be frustrating due to the motor deficits.

The OT is working with the parents of a 4 y/o boy who demonstrates a strong tonic bite reflex when eating. What type of utensils will the OT MOST likely recommend to the child's parents? a. weighted universal grips b. curved utensils c. swivel utensils d. rubber-coated spoons

d. rubber coated spoons Rubber coated spoons will allow the parent to remove the utensil from the child's mouth more easily. The other answers would not be recommended for a child with a strong bite reflex but would be for other children whose feeding is impacted by incoordination, apraxia, or tremors.

a child with athetoid CP is working in OT to develop self-feeding skills. When the child attempts to pick up food, it is inadvertently pushed off the plate. Which adaptation should the OT provide to solve this problem? a. swivel spoon b. nonslip mat c. mobile arm support d. scoop dish

d. scoop dish This is correct as its high rim provides a surface against which to push the food. The child would have less difficulty with coordinating movement of food because the sides of the dish would provide a shape that aids scooping of food. A swivel spoon helps primarily when supination is limited. A nonslip mat helps stabilize the plate itself. Mobile arm support positions the arm to help weak shoulder and elbow muscles to position the hand.

The OT is thinking ahead to a childs rehab following a TBI. His current program, while he is in acute care, is focus on positioning and movement to maintain joint movement, sensory stim, and family education. Which is the OT MOST likely to consider to help identify when the childs awareness increases to a point at which his OT program can shift its focus. a. the childs muscle tone increases b. childs visual disturbances have diminished c. parents have prioritized ADL performance as a goal d. scores on the Ranchos Los Amigos Cognitive recovery scale

d. scores on the Rancho scale this scale lists ten levels of cognitive performance with 1 being no response and 10 being purposeful, appropriate, modified independence. Answers A and B are incorrect as each change is not necessarily evident in all children with TBIs. Answer C is important for all families and may be stressed immediately after the childs accident and throughout the acute phase. However, this priority itself does not enable a child to begin rehab when medical concerns are too great.

The OT is teaching the parents of a child with spastic quadriplegia how to effectively position the child sitting in a wheelchair so that the participation in family tabletop games is facilitated. What is the OT MOST likely to recommend? a. keep the head upright b. place arms on the armrest c. ensure the back is straight d. secure the pelvis in the seat

d. secure the pelvis in the seat. Key points in achieving functional seating are the position and stability of the pelvis.

an 8 y/o boy with conduct disorder is disruptive, uncooperative, and occasionally combative during therapy. From a behavioral point of view, what is the MOST appropriate strategy to use to address the child's behavior? a. allow the child to express his anger without restraint for a short period to vent his frustration b. ignore the behavior and continue with therapy with or without the childs cooperation c. attempt to reason with the child to get his cooperation d. set clear expectations for behavior and enforce consequences.

d. set clear expectations for behavior and enforce consequences. Many children respond best when rules are clearly established. Answers A and B would not help the child learn new behaviors and could cause an increased loss of control . Answer C is incorrect because a child who is out of control will not respond to an insight-based approach.

A patient with a SCI is on a rehab unit and constantly flirts with the OT. What is the most appropriate way for the OT to respond? a. firmly reject the patients advances and stop treatment b. acknowledge the patients actions and mildly flirt back in order to promote the patients self-esteem c. request that the supervising OT discuss the effects of SCI and sexual functioning with the patient d. set personal boundaries appropriate to the therapist-patient relationship

d. set personal boundaries appropriate to the therapist-patient relationship it is important to acknowledge an individuals needs for sexual expression , but these behaviors should be dealt with appropriately by setting boundaries. A may cause an individual to believe he or she is undesirable. B may imply that a sexual relationship is possible.

An OT is working with 3 individuals in a cooking group who demo difficulty attending to tasks, frequently as to leave the room, and do not interact with each other. Based on the developmental group FOR, which of the following is the most appropriate goal for this group for each group member? a. experiment with trying one different group role b. share materials and tools with at least one other group member c. express two positive feelings about onesself within the group session d. share space while working on a task without disrupting each other for 15 minutes

d. share space while working on a task without disrupting each other for 15 minutes Parallel groups are most appropriate for individuals who can tolerate being with more than one person at a time but do not have the ability to interact successfully with other group members. A is a goal consistent with an ego-centric group. B is a project group goal. C is a cooperative group goal .

the OT is seeking a noncompetitive employment environment for a client with an ID who wants to develop basic work skills. What would be the MOST appropriate community environment? a. a clubhouse program b. supported employment c. a shop with a job coach assistance d. sheltered workshop

d. sheltered workshop these are noncompetitive employment settings intended to provide many of the benefits of a work atmosphere for individuals with disabilities. Supported employment and working with a job coach are both competitive and integrated work environment. Club house models offer a range of employment options but are typically used in psych rehab.

an OT in a residential community mental health setting is developing a psychoeducational program to promote healthier eating habits among the residents. To accomplish the groups goals using this approach, what is the OT most likely to do? a. have each client make a healthy good collage b. have the group plan and shop for a meal c. designate 1 day a week for residents to be responsible for cooking dinner d. show a video about nutrition and keep a meal diary for a week.

d. show a video about nutrition and keep a meal diary for a week. the psychoeducational model utilizes a teacher-student format as opposed to a learning-by-doing approach. It is used to teach specific info to clients and their families. The other answers can all be used to promote healthier eating but they all involve learning through occupation based activities rather than psychoeducational activities

what is the best position in which the OT can place an 8 month old baby to provide the opportunity to further develop reaching skills for play? a. supine b. prone c. sidelying d. sitting

d. sitting this is correct because an upright sitting position provides the child with the opportunity to develop a variety of skills. A and B are incorrect because head and neck stability have most likely developed primarily in the prone and side-lying position before the child attains a seated position. D is incorrect as arm reaching out into space is limited by the floor surface on which the infant lies.

following hospitalization for an acute schizophrenic episode, a college student in a psych hospitalization program is uncomfortable in social settings, has difficulty sustaining conversations, is unable to maintain eye contact, and responds to others with bizarre comments. Which of the following would be the MOST effective treatment approach? a. vestibular stimulation and gross motor exercises b. modifications of the social environment c. activities that do not require conscious attention to movement d. social skills training

d. social skills training This is the correct answer because it can be used to develop the ability to appropriately and effectively relate to others. Answer A is a sensory integration treatment approach. Answer C is best suited for individuals with chronic schizophrenia. Answer B will not help in developing social skills and is most appropriate for individuals with cognitve deficits.

an OT iss planning a group that will include some teens who pose a suicide risk. In selecting craft media, what activity would most likely be safest? a. leather checkbook cover with leather lacing b. macrame plant hanger c. ceramic vase d. stenciling greeting cards

d. stenciling greeting cards it is the safest choice because it is free of sharp, toxic, and cordlike materials. A ceramic object can be broken into sharp, weapon-like pieces, cord can be used for hanging, and needles are required for lacing crafts.

an OT is assessing hand function in an individual with arthritis. While making a PB&J, the individual is unable to remove the lid from a peanut butter jar but is able to stand at the counter, spread the peanut butter on bread with a knife, and replace the lid. These observations would most likely reflect a deficit in which of the following? a. ROM b. coordination c. endurance d. strength

d. strength exerting enough pressure to twist off a jar lid requires strength making answer D correct.

an OT has been asked to perform an ergonomic evaluation and provide ergonomic interventions to a job site where the rate of cumulative trauma disorders is unusually high. which of the following actions best addresses this request? a. introduce relaxation seminars for the employees to decrease stress while on the job b. treat corporate clients for cumulative trauma disorders c. provide work simulation activities d. suggest furniture and accessories that promote better positioning at work

d. suggest furniture and accessories that promote better positioning at work Answers A and C are not considered ergonomic interventions. Answer B is an example of a treatment intervention

an OT asks an individual in a manic state what he would like to do in a craft group. The individual answers I am really good at carpentry so I am going to build my kids a club house. What is the OTs best response? a. support him in his choice b. tell him he doesnt have the necessary attention span at this time c. redirect him towards an activity that does not require sharp tools d. suggest trying a small birdhouse first

d. suggest trying a small birdhouse first The client will benefit from direction providing parameters of acceptable behavior and an honest appraisal of behavior presented in a gentle, yet firm,way. Manic individuals are often unaware of their problem areas and therefore make grandious or inappropriate choices. the lack of awareness may make it difficult for the individual to recognize or accept that his attention span is too limited. Whereas, depressed individuals may need support for his choices, a manic individual may need redirected. Although sharp tools would be contraindicated for individuals with suicidal ideation, in this situation it is the time, space, and complexity of this project that makes answer D the correct one.

An OT has been working with an individual with schizophrenia who has been living in a homeless shelter. The individual has been consistent about taking medication and his symptoms are currently well controlled. The individual has expressed an interest in renting an apartment in the community, where he can have responsibility for his own meals and housekeeping, develop social and leisure activities, and still receive necessary social and rehabilitation services. Which type of housing should the OT recommend in order to BEST meet this individuals preferences? a. partial hospitalization b. assisted living c. custodial housing d. supportive housing

d. supportive housing Supportive housing is described as independent housing coupled with provision of community based mental health services. Staff provide case management, support, and/or rehab in a variety of different housing types, including halfway houses, group homes, and supervised apartments. C would not provide the rehabilitation and social services the individual would require. B is typically considered for older adults who require assistance with medication management, light housework, meals, and activities, and would provide more structure than this individual desires. A is an outpatient mental health service for individuals who still need intense daily treatment but not the level of safety provided with inpatient rehab.

While evaluating an individual with arthritis, the OT observes PIP joint hyperextension and DIP joint flexion in the digits. The OT will MOST LIKELY document this as a: a. boutonniere deformity b. mallet finger c. congenital deformity d. swan neck deformity

d. swan neck deformity. A boutonniere deformity is characterized by PIP flexion and DIP hyperextension. C is related to deformity present at birth, and B is characterized by DIP joint flexion and loss of active extension.

An individual with a TBI is able to pick up a toothbrush and apply toothpaste independently, but takes 15 minutes to rush his teeth. This behavior most likely indicates difficulty in which of the following areas? a. sequencing b. following directions c. problem solving d. terminating an activity

d. terminating activity this individual demonstrated difficulty ending the task evidenced by perseveration. Trying to put toothpaste on the toothbrush before taking off the cap would have been an example of deficits in sequencing. Difficulty following directions would have been evidenced by the individual doing a different task than the one presented. Impaired problem solving would be an individual trying to squeeze toothpaste out too softly and giving up before achieving the goal.

When working within a school system to help include child with spina bifida within a gen ed first grade, the OT consults with the classroom teacher. What is the OTs priority outcome of this consultation? a. the OT learns about the first grade curriculum and daily schedule of first grade students b. the teacher understands more about how OT can support the students in the classroom c. the OT evals the student and writs a complete summary report for the school record d. The OT and the teacher identify new strategies to help the student access and participate in the classroom learning centers

d. the OT and the teacher identify new strategies to help the student access and participate in the classroom learning centers a consultation is a collaborative process that takes place between a practitioner and clients, families, or other professionals to identify and solve problems with occupations, prevent future problems, and achieve identified goals. The other answers are incorrect as they represent learning and outcomes for only one person engaged in the collaborative relationships.

an OT working with adults with serious persistent mental illness is planning a weight management program. Based on a lifestyle redesigned model, the OT will begin with a didactic component. Which of the following is most appropriate for the first session? a. participants share stories about past experiences, challenges, and successes when trying to lose weight b. participants identify one meal that has personal meaning to them and list substitutes for the least healthy ingredients c. participants form a cooking group in which they make low-calorie snacks d. the OT presents info on nutrition, diet, and their impact on health and occupations.

d. the OT presents info on nutrition, diet, and their impact on health and occupations lifestyle redesign programs emphasize power of occupations by educating participants about the health related consequences of their occupations. Each module begins with a didactic presentation and occupational analysis. In addition, modules include peer exchange and direct experience.

Which of the following is the BEST example of the plan section of a discharge summary when using the SOAP format? a. the patient reports intentions to continue to practice proper body mechanics at work b. the patient demos independence in performing the HEP c. the patient expressed a desire to return to work but does not yet demo the capacity for the required sitting tolerance. d. the OT recommends use of lumbar support and regular performance of HEP.

d. the OT recommends use of lumbar support and regular performance of HEP The plan section of a discharge summary contains the patients discharge disposition, recommendations for additional therapy, actions for the patient, equipment needs, and equipment provided. Answer A is a subjective report. B is an example of the objective portion of a discharge summary, and C belongs in the assessment area.

An OT documenting the progress of a client using the SOAP note format would include which of the following in the subjective info section? a. the OT will establish a daily self feeding routine using verbal and physical cues to encourage the client to open containers on the lunch tray b. the client has been able to identify closed liquid beverage containers on the meal tray for 4-6 presentations c. the client is able to identify and drink liquids presented in cups without lids but leaves beverages in closed containers untouched d. the client asks for more beverages during the meals and appears surprised when the OT indicated beverages in closed containers that are on the meal tray.

d. the client asks for more beverages during the meals and appears surprised when the OT indicated beverages in closed containers that are on the meal tray. The subjective of the SOAP should contain info gained through a chart review or communication with the patient, family, or staff. This information is not measurable. Answer A would be in the plan. B and C would be the objective portion of the notes because they are either measurable or based on specific observations.

an individual is beginning to demo return in the RUE following a CVA, but has mildly impaired sensation in the right hand, which results in the ability to identify objects. Which of the following would be the best method to improve stereognosis? a. the OT verbally describes how an object looks prior to touching it b. the individual looks at the object before touching it c. the individual works with putty to strengthen her hand d. the individual attempts to find coins in a pocket while occluding vision.

d. the individual attempts to find coins in a pocket while occluding vision. steroegnosis is the inability to identify objects through proprioception, cognition, and the sense of touch such as identifying coins in a pocket. A and B do not encourage the use of proprioception and touch where as C may be used for strengthening.

The OT researcher wishes to strengthen a study. What does employment of random assignment assume? a. there is no need for inclusion criteria b. research subjects always receive the experimental treatment c. the sample is not representative of the larger population d. there is equivalency between the study group and the control group

d. there is equivalency between the study group and the control group In experimental-type design, procedures to establish control are implemented to minimize the influences of extraneous variables on the outcome of the dependent variable. The other answers are incorrect and have no relation to the process of ensuring random assignment/equivalence among group members.

A sales executive being treated for anxiety is participating in OT to develop time management skills. Which of the following would be the expected outcome for the individual? a. to control anxiety when arriving late for a meeting b. to take responsibility when late with reports c. to cope with feelings of inadequacy when missing a deadline d. to eliminate late arrival to work

d. to eliminate late arrival to work Learning effective time management techniques is a useful strategy for people with anxiety disorders. The focus of this training is to provide the individual with strategies so that they are able to live successful and productive lives. The other answers are ways of coping with being late, not strategies for the time management goal of being on time.

At an initial IFSP meeting for a 1 month old infant with Down syndrom, the team determines that OT will serves as the primary therapist implementing home visits and interacting with the family. Which team approach for young children does this represent? a. unidisciplinary b. multidisciplinary c. interdisciplinary d. transdisciplinary

d. transdisciplinary this is when one team member provides the direct intervention and the other team members function in collaborative consultant roles. Unidisciplinary is not really a team. In a multidisciplinary approach, each profession is responsible for his or her own discipline. Interdisciplinary teach is incorrect because this is composed of members identifying goals and plans collaboratively

an OT is instructing an individual with arthritis how to maintain functional ROM while performing household activities Which of the following activities will most effectively accomplish this goal? a. use short strokes with the vacuum cleaner b. keep elbows flexed when ironing c. place lightweight objects on low shelves d. use a dust mit to keep fingers fully extended

d. use a dust mit to keep fingers fully extended an effective means of maintaining function with arthritis is having the client perform daily occupations. A requires long strokes which promotes good elbow and shoulder ROM. B should attempt to keep the elbow in full flexion to maintain ROM. C is opposite of what is necessary to increase or maintain ROM.

A young child exhibits tactile defensiveness with all dressing tasks. What is the MOST effective technique to facilitate the task? a. tickle the child prior to dressing and undressing b. play loud music when undressing the child c. lightly stroke the childs arms and legs during undressing tasks d. use deep pressure when holding and moving the child during dressing.

d. use deep pressure when holding and moving the child during dressing This is correct as most individuals with tactile defensiveness feel comfortable with deep touch. Answers A and C represent light touch which is uncomfortable or intolerable for children with this diagnosis. B causes further discomfort during a time when the child is extremely vulnerable to sensation of light touch.

An instructor from a local nursing school has asked an OT to speak about OT to a class of first-year nursing students. The practitioner feels uncertain about giving the lecture because of lack of resources. What is the MOST appropriate action to take? a. recommend that the nursing course instructor call AOTA and obtain info to share b. decline to do the lecture but send information to the instructor c. decline to do the lecture and try to find another OT to present the lecture d. use the AOTA website to obtain resources about OT.

d. use the AOTA website to obtain resources about OT It is every OTs responsibility to promote the profession.

an OT has been asked to develop a skills training program for individuals with cognitive limitations who will be working on a dairy farm. What is the first step in designing an intervention that will effectively address the needs of this population? a. interview consumers to identify solutions to the problems using open-ended questions b. create a skills training curriculum based on identified problems and solutions c. develop a list of problems and solutions based on the literature d. using consumer input, create a list of potential problems

d. using consumer input, create a list of potential problems designing a skills training intervention begins with problem identification followed by solution identification and finally module development. In this situation it would be particularly important to involve the consumers in problem and solution identification, because the likelihood of finding literature pertaining to this area is slim.

during an initial visit with a 5-year-old child with a suspected learning disability, the OT observes the child run across the room, hop around on one foot, pick up a pencil, and draw a stick figure using a tripod grasp. when asked to complete a 10-piece puzzle, the child gives up after several unsuccessful attemp. which type of assessment is most appropriate to gather additional information to understand this child's difficulty? a. fine motor b. gross motor c. developmental d. visual perceptual

d. visual perceptual visual perceptual deficits may result in problems with cutting, coloring, constructing with blocks, doing puzzles, or construction toys. running hopping using a tripod grass, drawing a stick figure, and putting together a 10-piece puzzle are all developmentally appropriate skills for a 5-year-old. although this child cannot put together the puzzle, the gross and fine motor skills required are observed to be intact. No fine motor evaluation is necessary because A tripod grasp is achieved. No gross motor evaluation is necessary because running and hopping skills are evident. Because the child's abilities appear developmentally appropriate, developmental evaluation is not indicated.

when assessing an individual who is suspected of having carpal tunnel syndrome, the OT tests for tinel sign by gently tapping the median nerve at what level? a. elbow b. midforearm c. palmar crease d. volar wrist

d. volar wrist tapping at the volar wrist elicits tinels sign. The other answers are incorrect.

an individual diagnosed with Guillain barre acute syndrome exhibits good UE strength. The activity MOST appropriate for further strengthening and endurance building would be: a. peeling potatoes b. bed making c. polishing furniture d. washing windows

d. washing windows This is a repetitive activity that involves resisted, elevated UE activity. Rests can be taken as needed. The other answers do not provide adequate resistance to achieve UE strengthening.

An OT is assessing a 3 y/o child born with fetal alcohol syndrome. In addition to the parent interview, which is the NEXT strategy to gather assessment data? a. complete an activities and interests survey with the child b. record the childs developmental milestones on a standardized chart for comparison c. refer to the childs medical records and reports from early intervention series d. watch the child engage in play, eating, and bathing activities.

d. watch the child engage in play, eating, and bathing activities. this is the only choice that reflects the components of skilled observation. Answer A is most representative of interviewing. Although answer B is considered a comparative measurement assessment method, the question is asking for the next strategy that the OT would use. Answer C is most closely linked to record review, not skilled observation.

A short-term goal of the treatment plan of a child with CP is to improve the ability to manipulate objects during play, and increased flexor tone interferes with the childs ability to grasp objects. What activity would be MOST appropriate to meet this objective? a. building a block tower b. active release of blocks into a container c. traction on the finger flexors d. weight-bearing over a small bolster in prone position

d. weight bearing over a small bolster in prone position research has shown that children with CP demo increased use of wrist extension and finger extension after UE weight bearing. Answers A and B are incorrect because they require voluntary control of the release of objects. C is incorrect because traction on the finger flexors may increase spasticity in the flexor muscles.

The OT is working on handwriting skills with a school aged child with decreased proprioception in the hand and wrist. Which adaptation would the OT recommend for writing? a. wide pen b. attach a rubber band to the eraser and the childs wrist c. soft triangle grip d. weighted utensil holder

d. weighted utensil holder This increases proprioceptive feedback to the joints adding to the children's awareness of the position and movement of the hand, wrist, and fingers. Answers A and C would assist a child with decreased grip, whereas answer B would encourage the student to use a slanted and relaxed pencil positioning for writing.


Related study sets

Banzai Plus Post test study guide

View Set

BA 3350 Chapter 6 Test Prep THIDA 80

View Set

BIOL 2120 Chapter 5: An Introduction to Carbohydrates

View Set

Chp. 23 Genomics II: Functional genomics, proteomics and bioinformatics

View Set

UNIT 1 -13 Level 1 certification

View Set